1 名前:132人目の素数さん [2017/11/01(水) 22:25:02.36 ID:lCaf2vX7.net] さあ、今日も1日頑張ろう★☆ 前スレ 分からない問題はここに書いてね434 https://rio2016.5ch.net/test/read.cgi/math/1507993404/
2 名前:132人目の素数さん [2017/11/01(水) 22:30:50.35 ID:4z8K/pt+.net] 削除依頼を出しました
3 名前:¥ mailto:sage [2017/11/01(水) 22:33:59.63 ID:cSPyhj3J.net] ¥
4 名前:¥ mailto:sage [2017/11/01(水) 22:34:17.70 ID:cSPyhj3J.net] ¥
5 名前:¥ mailto:sage [2017/11/01(水) 22:34:36.06 ID:cSPyhj3J.net] ¥
6 名前:¥ mailto:sage [2017/11/01(水) 22:34:53.32 ID:cSPyhj3J.net] ¥
7 名前:¥ mailto:sage [2017/11/01(水) 22:35:10.20 ID:cSPyhj3J.net] ¥
8 名前:¥ mailto:sage [2017/11/01(水) 22:35:28.06 ID:cSPyhj3J.net] ¥
9 名前:¥ mailto:sage [2017/11/01(水) 22:35:44.97 ID:cSPyhj3J.net] ¥
10 名前:¥ mailto:sage [2017/11/01(水) 22:36:07.12 ID:cSPyhj3J.net] ¥
11 名前:¥ mailto:sage [2017/11/01(水) 22:36:24.50 ID:cSPyhj3J.net] ¥
12 名前:¥ mailto:sage [2017/11/01(水) 22:36:42.80 ID:cSPyhj3J.net] ¥
13 名前:132人目の素数さん [2017/11/01(水) 22:37:33.69 ID:SCGDdq8H.net] 前スレの>>1000 の人。 ありがとうございます。
14 名前:132人目の素数さん [2017/11/01(水) 22:42:56.70 ID:SCGDdq8H.net] ただ、f(x)= sinx-2x/πにしてグラフを調べるのは分かるのですが、不等式を証明せよという問題でグラフを調べれば証明した事になりますかね?
15 名前:132人目の素数さん mailto:sage [2017/11/01(水) 22:52:51.48 ID:gL9JwERl.net] グダグダ書き込む前に実際にグラフを書いた方が良いでしょうね
16 名前:132人目の素数さん mailto:sage [2017/11/01(水) 23:52:53.23 ID:xrV7qa8b.net] [前スレ.997] Jordan の不等式(微分を使わない方法) 円c(半径r)の直径をABとする。 A,Bを通るもう一つの円C(半径 R >r)がある。 このとき線分ABの長さは 2R sin(x)= 2r, また横方向のズレ幅からみて、明らかに 弧AcB > 弧ACB, πr > 2R x, 辺々掛けて sin(x)> 2x/π,
17 名前:132人目の素数さん [2017/11/02(木) 00:04:13.34 ID:H3Q9xkUT.net] 次の微分
18 名前:方程式の解法が全く分かりません。ご教授お願いします。 y=-xdy/dx+x^4(dy/dx)^2 [] [ここ壊れてます]
19 名前:132人目の素数さん mailto:sage [2017/11/02(木) 00:46:37.78 ID:FM0nIjMB.net] >>17 x = 1/t とおくと y = t・(dy/dt)+(dy/dt)^2 これは Claireaut の方程式なので、tで微分して {t +2(dy/dt)}(d^2 y/(dt)^2)= 0, ・d^2 y/(dt)^2 = 0 のとき y = c(t+c)= c(1/x +c), (cは任意定数) ・t + 2(dy/dt)= 0 のとき y = -tt/4 = -1/(4xx), …包絡線 頑張ってクレロー
20 名前:¥ mailto:sage [2017/11/02(木) 01:17:40.53 ID:23MnTxXU.net] ¥
21 名前:¥ mailto:sage [2017/11/02(木) 01:18:03.38 ID:23MnTxXU.net] ¥
22 名前:132人目の素数さん [2017/11/02(木) 01:21:01.02 ID:H3Q9xkUT.net] >>18 ありがとうございました!
23 名前:132人目の素数さん [2017/11/02(木) 09:15:35.19 ID:pIX/ZAmh.net] 距離空間 X の任意の部分集合 A に対し、 A の内部の閉包は A の閉包に含まれる ことを証明せよ。 お願いします。
24 名前:132人目の素数さん mailto:sage [2017/11/02(木) 09:40:56.35 ID:DIS6M2o1.net] int(A) ⊂ A より cl( int(A) ) ⊂ cl(A) A ⊂ B → cl(A) ⊂ cl(B) が分からんて事? cl(B) は B を含む、つまり Aを含む閉集合である。 cl(A) は Aを含む閉集合の族の共通集合 (Aを含む最小の閉集合)である。 Aを含む閉集合には cl(B) が含まれるので、cl(A) ⊂ cl(B) である。
25 名前:132人目の素数さん mailto:sage [2017/11/02(木) 09:47:05.10 ID:pIX/ZAmh.net] >>23 ありがとうございました。
26 名前:132人目の素数さん mailto:sage [2017/11/02(木) 11:20:59.99 ID:swN/GnGJ.net] 閉包の定義から証明するんじゃねーかよ
27 名前:132人目の素数さん [2017/11/02(木) 11:40:19.21 ID:pIX/ZAmh.net] 以下の問題の解答ですが、もっと簡単になりませんか? 距離空間 X において部分集合 A の集積点全部の集合を A' で表すことにする。 A' は閉集合であることを証明せよ。 A の孤立点の集合を A'' で表すことにする。 A の内部を A^i で表すことにする。 A の外部を A^e で表すことにする。 A の閉包を cl(A) で表すことにする。 点 a を中心とする半径 r の開球を B(a ; r) で表すことにする。 A' = cl(A) - A'' = cl(A) ∩ (A'')^c である。 (A')^c = [cl(A) ∩ (A'')^c]^c = cl(A)^c ∪ A'' = A^e ∪ A'' a ∈ (A')^c とする。 a ∈ A^e ならば、 A^e は開集合だから、 B(a ; r) ⊂ A^e ⊂ (A')^c となるような r > 0 が存在する。 ∴ a ∈ ((A')^c)^i a ∈ A'' ならば、 B(a ; r) ∩ cl(A) = {a} となるような r > 0 が存在する。 ∴ B(a ; r) ⊂ cl(A)^c ∪ {a} ⊂ cl(A)^c ∪ A'' = (A')^c ∴ a ∈ ((A')^c)^i 以上より、 (A')^c は開集合である。 ∴ A' は閉集合である。
28 名前:132人目の素数さん [2017/11/02(木) 12:23:52.24 ID:kG1AS/N8.net] 「無」は至高ですか?
29 名前:132人目の素数さん mailto:sage [2017/11/02(木) 14:29:30.82 ID:DIS6M2o1.net] >>26 【 A' := { x ∈ X | 任意のU∈V(x) に関して (U-{x}) ∩ A ≠ φ } (V(x)は xの開近傍族) 】 任意の x ∈ X - A' について、定義より ある U∈V(x) が存在し (U-{x}) ∩ A = φ である 。 任意の y ∈ (U-{x}) について、 yの開近傍 U' で U' ⊂ U かつ U' ∩ {x} = φ となるものが存在する。 (∵ Uは開集合であり、距離空間Xはハウスドルフ空間である) (U'-{y}) ∩ A ⊂ U' ∩ A = (U'-{x}) ∩ A ⊂ (U-{x}) ∩ A = φ である。 つまり、ある U'∈V(y) が存在し (U'-{y}) ∩ A = φ である。xの件と合わせて、 「任意の x ∈ X - A' について、ある U∈V(x) が存在し U ⊂ X - A' である」事が示せた。 よって X - A' は開集合、つまり A' は閉集合である。 "簡単" と感じるかどうかは人によるかも
30 名前:132人目の素数さん mailto:sage [2017/11/02(木) 14:33:22.21 ID:DIS6M2o1.net] 回りくどい事しないで U' = U - {x} としてもよかった....。
31 名前:132人目の素数さん mailto:sage [2017/11/02(木) 17:02:06.60 ID:G6iiPKxf.net] 前スレで塗りつぶされた正三角形の個数を聞いた者です 教えていただいたことを参考に、正六角形で初期配置を近似する方法で極限が1になる証明ができました! 先生曰くほとんど白紙提出だったとのことですが、ありがとうございました!
32 名前:132人目の素数さん mailto:sage [2017/11/02(木) 17:25:21.97 ID:swN/GnGJ.net] >>26 x をA'の集積点とし x に収束するA'の点列を{a_n}とすると |b_n − x|<1/n となる部分列{b_n}⊂{a_n}が存在する {b_n}⊂{a_n}⊂A'だから b_n∈A'であり b_nはAの集積点だから b_nに収束するAの点列{c(n)_m}が存在し |c(n)_m − b_n|<1/m となる部分列{d(n)_m}⊂{c(n)_m}が存在する d(n)_n∈Aであり |c(n)_n − x|<|c(n)_n − b_n| + |b_n − x|<2/n だから x はAの集積点であり x∈A' となる ∴ A'はA'の集積点全部を含むから閉集合である
33 名前:132人目の素数さん [2017/11/02(木) 19:08:37.87 ID:kG1AS/N8.net] 「無」になってもう二度と「有」になりたくない。 それが唯一にして最大の願い。
34 名前:132人目の素数さん mailto:sage [2017/11/02(木) 19:17:50.32 ID:pQ8LPgM/.net] >>32 あきらめるのは、まだ早い https://ameblo.jp/monsters55/entry-12003176219.html ちょっと感動した
35 名前:132人目の素数さん mailto:sage [2017/11/02(木) 19:51:18.03 ID:GMAJI1p+.net] この6次元とか10次元ってのはどこからくるんですか?一般の公式もあるのでしょうか? ファインマンbot? @feynmannnn 3次元空間の中に埋め込んだ2次元曲面は、曲がった空間の簡単な例として考えられた。 しかし3次元空間の曲率を同じように表現するには6次元空間に埋め込む必要があり、 また4次元空間の場合には、10次元空間に埋め込んで考える必要がある。時空の曲率は、面の曲率よりもかなり複雑なのである。
36 名前:132人目の素数さん mailto:sage [2017/11/02(木) 20:02:12.26 ID:86VaCdO4.net] 超弦理論の話です、多分 まだ未完成の理論ですから、話半分に聞いておけば良いでしょう
37 名前:132人目の素数さん mailto:sage [2017/11/02(木) 20:07:15.02 ID:kG1AS/N8.net] やっぱり数学って才能が必要なんですかね・・・? 受験数学レベルなら才能はもしかして必要ないかもしれないけど、 東大の院で博士号を取得するレベルになると、もはや才能無しでは太刀打ちできない気がするのですが・・・・・。 東京大学理学部数学科卒 → 東京大学大学院数理科学研究科数理科学専攻修士課程修了 → 東京大学大学院数理科学研究科数理科学専攻博士課程修了 というルートを辿りたいのですが、どうすれば良いですか? やっぱり猛烈に努力するしかないのでしょうか? それでも絶対に無理ですか?
38 名前:132人目の素数さん mailto:sage [2017/11/02(木) 20:14:02.10 ID:86VaCdO4.net] >>36 内容云々ではなく、東大でそういう道を辿るのは難しいでしょうね まず受験という壁がありますし、進振りとかいう制度のせいで、大学入った後も競争に勝つための勉強をしなければなりません そこで失敗すると自分の好きな学科に行けないそうです まああなたの場合は白チャートもできないんですから、杞憂というやつですね
39 名前:132人目の素数さん mailto:sage [2017/11/02(木) 20:22:33.61 ID:pQ8LPgM/.net] >>36 >>33 にも書いておいたが、まず到達可能な目標をたててみるのがいいでしょう。 日本で一番難しい資格試験・司法試験を攻めてみてはいかが?それなりに評価が高いと思います
40 名前:132人目の素数さん mailto:sage [2017/11/02(木) 20:36:00.25 ID:kG1AS/N8.net] >>38 すいません。 司法試験は、まったく興味がないというわけではないのですが、 やはり>>36 に示したルートを辿りたいという夢の方が遥かに大きいのです。 なんとしてでも>>36 に示したルートを辿りたいです。 なんとかならないでしょうか?
41 名前:132人目の素数さん mailto:sage [2017/11/02(木) 20:39:19.01 ID:5snpMa5u.net] 用語についての質問なのです 特性方程式と決定方程式では何か違いはあるのでしょうか? 常微分方程式の級数解法にて、フロベニウス級数解を持つと仮定して式変形をして出てきた式が 決定方程式と呼ばれていたのですが、特性方程式と大きく変わらないような気がしました ただ私が勝手に大した違いは無いだろうと思い込んでしまっているかもしれないので 質問してみることにしました
42 名前:132人目の素数さん [2017/11/02(木) 22:11:52.49 ID:PYfJKPhX.net] >>39 最終目的って何なの? 世の中には、目的達成のため正面きって真っ正直に難しい道を行く奴はバカだ・・ って価値観の人もいるし、目的が同じなら他大学の理学部数学科じゃだめかよ
43 名前:132人目の素数さん mailto:sage [2017/11/02(木) 22:38:06.31 ID:FM0nIjMB.net] >>27 色即是空 空即是色 (大意) 色(物質:フェルミオン)と空(力:ボゾン)とが同等(超対称)だということ。
44 名前:132人目の素数さん mailto:sage [2017/11/02(木) 22:42:07.57 ID:OMY3pab6.net] SUSYってどうなんですかね? 実験的にはそんなのがあるかけらも無いみたいですけど
45 名前:132人目の素数さん mailto:sage [2017/11/02(木) 22:44:07.26 ID:FM0nIjMB.net] 「色」ってクォークの color のことか("^ω^)・・・
46 名前:132人目の素数さん [2017/11/02(木) 23:03:33.16 ID:kb3Y9mL5.net] >>26 xが集積点 ⇔∀e>0 U(x,e)∩A≠φ xが集積点でない ⇔∃e>0 U(x,e)∩A=φ ⇔∃e>0 U(x,e)⊂X-A ⇔X-Aは開 ⇔Aは閉
47 名前:132人目の素数さん [2017/11/02(木) 23:10:34.20 ID:kb3Y9mL5.net] あちゃ >>45 >⇔X-Aは開 >⇔Aは閉 ⇔x∈(X-A)^iは開 xは集積点 ⇔x∈X-(X-A)^iは閉
48 名前:132人目の素数さん mailto:sage [2017/11/02(木) 23:42:23.66 ID:sgSl9kXV.net] ↑これが数学板の実力です 専門板なのに異常にレベルが低い せいぜい数学の少しできる高校生レベル
49 名前:132人目の素数さん [2017/11/02(木) 23:59:38.33 ID:kb3Y9mL5.net] >>47 もう止めたら?
50 名前:132人目の素数さん mailto:sage [2017/11/03(金) 00:05:21.90 ID:T1A7RIRi.net] >>48 集積点と触点が一致することを証明してください
51 名前:132人目の素数さん [2017/11/03(金) 00:12:04.43 ID:aWvOAd2K.net] >>49 あそうか
52 名前:132人目の素数さん [2017/11/03(金) 00:12:46.37 ID:aWvOAd2K.net] >>49 じゃ 証明してください
53 名前:132人目の素数さん mailto:sage [2017/11/03(金) 00:17:24.89 ID:T1A7RIRi.net] 既に上に回答がいくつか上がってますよね?
54 名前:132人目の素数さん [2017/11/03(金) 00:19:17.67 ID:aWvOAd2K.net] >>52 もっと簡単な奴でどうぞ
55 名前:132人目の素数さん [2017/11/03(金) 00:51:06.53 ID:aWvOAd2K.net] xが集積点 ⇔∀e>0 U(x,e)-{x}∩A=U(x,e)∩A-{x}≠φ xが集積点でない ⇔∃e>0 U(x,e)∩A-{x}=φ ⇔∃e>0 U(x,e)⊂X-A∨U(x,e)∩A={x} ⇔∃e>0 ∀y∈U(x,e) ∃d>0 U(y,d)⊂U(x,e)⊂X-A∨U(y,d)∩A={y} ⇔∃e>0 ∀y∈U(x,e) yは集積点でない ⇔集積点でない点の全体は開 ⇔集積点の全体は閉
56 名前:132人目の素数さん [2017/11/03(金) 00:56:45.09 ID:aWvOAd2K.net] >>54 >⇔∃e>0 U(x,e)⊂X-A∨U(x,e)∩A={x} >⇔∃e>0 ∀y∈U(x,e) ∃d>0 U(y,d)⊂U(x,e)⊂X-A∨U(y,d)∩A={y} ⇔∃e>0 U(x,e)∩A=φor{x} ⇔∃e>0 ∀y∈U(x,e) ∃d>0 U(y,d)∩A=φor{y} のがいいや}
57 名前:132人目の素数さん [2017/11/03(金) 00:58:42.26 ID:aWvOAd2K.net] >>55 >⇔∃e>0 ∀y∈U(x,e) ∃d>0 U(y,d)∩A=φor{y} ⇒
58 名前:132人目の素数さん [2017/11/03(金) 01:03:00.55 ID:aWvOAd2K.net] xが集積点 ⇔∀e>0 U(x,e)-{x}∩A=U(x,e)∩A-{x}≠φ xが集積点でない ⇔∃e>0 U(x,e)∩A-{x}=φ ⇔∃e>0 U(x,e)∩A=φor{x} ⇒∃e>0 ∀y∈U(x,e) ∃d>0 U(y,d)∩A=φor{y} ⇔∃e>0 ∀y∈U(x,e) yは集積点でない ⇒集積点でない点の全体は開 ⇔集積点の全体は閉
59 名前:132人目の素数さん mailto:sage [2017/11/03(金) 01:25:01.41 ID:T1A7RIRi.net] 誤答なんちゃらさんを思い出しますね
60 名前:132人目の素数さん [2017/11/03(金) 01:30:47.21 ID:aWvOAd2K.net] >>58 何もできないんですね
61 名前:132人目の素数さん [2017/11/03(金) 01:38:01.19 ID:aWvOAd2K.net] >>57 >U(x,e) 距離でなくて開近傍で十分
62 名前:132人目の素数さん [2017/11/03(金) 02:35:32.38 ID:fhtpXXZ8.net] 神と無はどっちの方が凄いですか?
63 名前:132人目の素数さん mailto:sage [2017/11/03(金) 03:59:15.27 ID:VPk+2qO6.net] 代数方程式の解求める問題 A1x^n+A2x^n-1 ... Anx + An+1 = 0 みたいなものがあるとします。 A2~An+1は実数なんだけど、A1だけ1から2の区間?の場合はどうすれば解の集合が求まるか分かる人いますか? 無数の方程式の求根をしてその全てを包含するような解の区間を求めたいのですが… 先生は一番難しい問題と言っていましたし、自分自身では何も思いつきません…
64 名前:132人目の素数さん [2017/11/03(金) 07:56:47.91 ID:7q8eT4re.net] それ複素数全体にならん?
65 名前:132人目の素数さん [2017/11/03(金) 10:44:03.66 ID:f/oY5xQT.net] この本ってどうですか? 現代数学序説 集合と代数 (ちくま学芸文庫) 松坂 和夫 固定リンク: amzn.asia/hbjyymd
66 名前:132人目の素数さん [2017/11/03(金) 10:48:53.55 ID:aWvOAd2K.net] A2〜An+1は与えられた実数定数でA1だけ幅があるってことだろ
67 名前:132人目の素数さん [2017/11/03(金) 10:49:43.15 ID:f/oY5xQT.net] 荒井秀男って岩波書店の人ですよね。 まだ生きていたんですね。 内容紹介 『集合・位相入門』などの名教科書で知られる著者による、懇切丁寧な入門書。 組合せ論・初等数論を中心に、現代数学の一端に触れる。解説 荒井秀男
68 名前:132人目の素数さん mailto:sage [2017/11/03(金) 10:59:51.67 ID:d8bRV0BU.net] >>61 神無月 ぢゃなくて 「神無一族の氾濫」です。 (詰将棋パラダイスに掲載されたフェアリー詰)
69 名前:132人目の素数さん [2017/11/03(金) 11:00:06.19 ID:f/oY5xQT.net] 次の(1), (2), (3)をみたす R 上の C^∞ 関数 f(x) と g(x) が存在する。 (1) lim_{x → ∞} f(x) = lim_{x → ∞} g(x) = +∞ (2) lim_{x → ∞} f'(x)/g'(x) は存在して有限値 (3) lim_{x → ∞} f(x)/g(x) は存在しない 例 f(x) = x + sin(x)*cos(x) g(x) = exp(sin(x)) * f(x) と書いてあるのですが、 g'(x) = exp(sin(x))*(f(x) + 2*cos(x))*cos(x) なので、 lim_{x → ∞} f'(x)/g'(x) は考えられないと思います。 これはどういうことなのでしょうか? lim_{x → ∞} f'(x)/g'(x) = a の定義は、 任意の正の実数 ε に対して、 K < x ⇒ |f'(x)/g'(x) - a| < ε となる実数 K が存在する です。 K < x かつ g'(x) = 0 となるような x がかならず存在しますので問題ではないでしょうか?
70 名前:132人目の素数さん [2017/11/03(金) 11:01:22.17 ID:HJ4C+ncY.net] f(x)=1/2*(1-(-1)^x)*(-1)^((x-1)((1-(-1)^x))/4)
71 名前:¥ mailto:sage [2017/11/03(金) 12:04:45.99 ID:fn4ojm6A.net] ¥
72 名前:¥ mailto:sage [2017/11/03(金) 12:05:03.74 ID:fn4ojm6A.net] ¥
73 名前:¥ mailto:sage [2017/11/03(金) 12:05:22.36 ID:fn4ojm6A.net] ¥
74 名前:¥ mailto:sage [2017/11/03(金) 12:05:39.89 ID:fn4ojm6A.net] ¥
75 名前:¥ mailto:sage [2017/11/03(金) 12:05:57.99 ID:fn4ojm6A.net] ¥
76 名前:¥ mailto:sage [2017/11/03(金) 12:06:15.13 ID:fn4ojm6A.net] ¥
77 名前:¥ mailto:sage [2017/11/03(金) 12:06:33.97 ID:fn4ojm6A.net] ¥
78 名前:¥ mailto:sage [2017/11/03(金) 12:06:53.14 ID:fn4ojm6A.net] ¥
79 名前:¥ mailto:sage [2017/11/03(金) 12:07:10.28 ID:fn4ojm6A.net] ¥
80 名前:¥ mailto:sage [2017/11/03(金) 12:07:28.56 ID:fn4ojm6A.net] ¥
81 名前:132人目の素数さん mailto:sage [2017/11/03(金) 12:20:20.47 ID:ghuUoCu2.net] >>68 f'/g'を具体的に書いてみろよ
82 名前:132人目の素数さん [2017/11/03(金) 12:24:16.53 ID:f/oY5xQT.net] f'(x) / g'(x) = 2*(cos(x))^2 / exp(sin(x))*(f(x) + 2*cos(x))*cos(x) ですね。 分母に cos(x) があるため、 f'(x) / g'(x) が定義されない x の値が無数にありますね。
83 名前:132人目の素数さん [2017/11/03(金) 12:28:15.60 ID:fhtpXXZ8.net] 神 vs 全 vs 無 ファイッ!!!!!!!!!!!!!!!!!!!!!!!!!!!!!!!!!!!
84 名前:¥ mailto:sage [2017/11/03(金) 16:15:10.92 ID:fn4ojm6A.net] ¥
85 名前:¥ mailto:sage [2017/11/03(金) 16:15:29.17 ID:fn4ojm6A.net] ¥
86 名前:¥ mailto:sage [2017/11/03(金) 16:15:46.68 ID:fn4ojm6A.net] ¥
87 名前:¥ mailto:sage [2017/11/03(金) 16:16:03.35 ID:fn4ojm6A.net] ¥
88 名前:¥ mailto:sage [2017/11/03(金) 16:16:20.02 ID:fn4ojm6A.net] ¥
89 名前:¥ mailto:sage [2017/11/03(金) 16:16:37.03 ID:fn4ojm6A.net] ¥
90 名前:¥ mailto:sage [2017/11/03(金) 16:16:53.96 ID:fn4ojm6A.net] ¥
91 名前:¥ mailto:sage [2017/11/03(金) 16:17:12.06 ID:fn4ojm6A.net] ¥
92 名前:¥ mailto:sage [2017/11/03(金) 16:17:29.46 ID:fn4ojm6A.net] ¥
93 名前:¥ mailto:sage [2017/11/03(金) 16:17:49.10 ID:fn4ojm6A.net] ¥
94 名前:132人目の素数さん [2017/11/03(金) 16:24:10.47 ID:f/oY5xQT.net] リチャードって誰ですか? https://page.auctions.yahoo.co.jp/jp/auction/v524715453 微積分形式って何ですか? https://page.auctions.yahoo.co.jp/jp/auction/k278143925
95 名前:132人目の素数さん mailto:sage [2017/11/03(金) 16:43:01.16 ID:sh4/Y9Em.net] この証明の例が分かりません。明らかに間違っている(正しくMPが使われていない)と思うのですが…… https://i.imgur.com/JReZQZ2.jpg https://i.imgur.com/4zLyYdy.jpg
96 名前:132人目の素数さん mailto:sage [2017/11/03(金) 17:31:36.25 ID:5s5hiJkM.net] 最近調べたらなかなか難しくて感心した(呆れた)問題 W. Rudin, Real and Complex Analysis, 3rd.ed. pp. 229, Chapter 10 Exercise 23. 指数関数e^zのテイラー展開のn次までの和をP_n(z) P_n(z) = 1 + z + z^2 / 2! + ... + z^n / n! Q_n(z)=1-P_n(z)とするとき、P_n(z)、Q_n(z)の零点の位置についてどのようなことが言えるか (stackexchangeを探すと答えがいくつかあります)
97 名前:132人目の素数さん [2017/11/03(金) 19:09:20.30 ID:f/oY5xQT.net] 松坂和夫著『解析入門3』を読んでいます。 「X をコンパクトとし、 A を X の任意の無限部分集合とする。もし A が X の中に集積点をもたないとすれば、 X の任意の点 a に対し、適当な正の実数 r(a) をとれば、開球 B(a ; r(a)) は A の点をたかだか有限個しか 含まない。(前節12.1の命題6(b)参照。)」 と書かれています。参照先の命題6(b)は以下です。 「命題6(b) a ∈ X が A の集積点ならば、任意の r > 0 に対して B(a ; r) は無限に多くの A の点を含む。」 この命題を参照させるのはおかしいですよね。 任意の正の実数 r に対して、開球 B(a ; r(a)) が A の点を無数に含むとすれば、 a は明らかに A の集積点である。仮定により、 X の任意の点 a は A の集積点ではないから適当な正の実数 r(a) をとれば、開球 B(a ; r(a)) は A の点をたかだか有限個しか含まない。 ということですよね。
98 名前:132人目の素数さん [2017/11/03(金) 19:10:51.04 ID:f/oY5xQT.net] 松坂和夫さんの解析入門シリーズは、かなり詳しく位相について書かれていますね。 杉浦光夫の解析入門よりも細かいことが書いてあるのではないでしょうか?
99 名前:132人目の素数さん mailto:sage [2017/11/03(金) 22:22:32.38 ID:3k8GLaBb.net] >>95 それなりに検索しても答え見つからなかった。 よければリンク先教えて。
100 名前:132人目の素数さん [2017/11/03(金) 22:32:55.39 ID:bNV/YL/F.net] >>81 それ極限のこととは別
101 名前:132人目の素数さん [2017/11/03(金) 22:40:55.09 ID:bNV/YL/F.net] てゆか詰まらない
102 名前:132人目の素数さん mailto:sage [2017/11/03(金) 23:23:05.31 ID:d8bRV0BU.net] >>95 むかしむかし、P_n(z)の零点を Wolframalpha に書かせて twitter に上げてる人がいました。("^ω^)・・・ 馬蹄形にきれいに並んでましたよ。 nが奇数のときの負根は -(0.28125n+0.85)ぐらいで遠ざかってましたね。
103 名前:132人目の素数さん mailto:sage [2017/11/03(金) 23:40:33.63 ID:5s5hiJkM.net] >>98 ほい https://math.stackexchange.com/questions/1976891/the-location-of-zeros-of-partial-sum-of-exponential-series https://mathoverflow.net/questions/4329/roots-of-truncations-of-ex-1/16500
104 名前:132人目の素数さん [2017/11/03(金) 23:56:46.67 ID:bNV/YL/F.net] >>101 イヤ別にいいんだけどさ n次までの近似というものを特別視する 積極的な理由ってあるのかなあ
105 名前:132人目の素数さん mailto:sage [2017/11/04(土) 00:49:13.40 ID:b2rnI0sj.net] >>103 それもそうだけどさ nが増すほどピカール性が増して 零点を追っ払うのが目に見える。
106 名前:132人目の素数さん [2017/11/04(土) 01:37:18
] [ここ壊れてます]
107 名前:.88 ID:QtnaBUMR.net mailto: アメリカ合衆国大統領とダライ・ラマはどっちの方が偉いですか? [] [ここ壊れてます]
108 名前:132人目の素数さん mailto:sage [2017/11/04(土) 02:43:28.05 ID:+mzx1ILZ.net] 次の問題が分かりません。 (2)はn=3のときで楽勝です。(1)はk=1で固定して楽勝と思いきや、k=1で固定して良いことをどう記述するか悩みます。 (3)は色々実験しましたが手が出ません。解の公式に代入したり規則性をつかもうとしたのですがよく分かりません。 n,kを自然数とする。xの2次方程式 x^2-(n!)x+n^k=0 について、以下の問に答えよ。 (1)次の条件をみたすnの範囲を、理由をつけて答えよ。「この方程式が実数解を持つようにkの値をとれる」 (2)この方程式が整数解をもつような(n,k)の組を1つ求めよ。 (3)この方程式が整数解をもつような(n,k)の組は無数に存在するか。
109 名前:132人目の素数さん [2017/11/04(土) 02:49:06.54 ID:QtnaBUMR.net] 数学者は天才ですよね?
110 名前:132人目の素数さん mailto:sage [2017/11/04(土) 06:13:42.85 ID:dhzc9Rtr.net] >>106 n^1≦n^kなのだから、 判別式を考えれば あるkで実数解をもてば必ずk=1の時に実数解をもつ 従って実数解を持つようにkが取れることとk=1の時に実数解を持つことは同値
111 名前:132人目の素数さん mailto:sage [2017/11/04(土) 06:18:11.56 ID:dhzc9Rtr.net] >>106 (3)に関しては、 解と係数の関係などを考えて 整数解を持つというのは片方だけ持つのか、両方とも整数になっていなくてはならないのか とか 両方整数なら、素因数を考えてどういう形をしてるか、 等を考えいくとできる
112 名前:132人目の素数さん [2017/11/04(土) 08:12:03.74 ID:OJrl3+LI.net] 楕円x^2+2y^2=1 放物線4y=2x^2+a 2つの式からxを消去して, 4y^2+4y-(a+2)=0・・・@ このyについての2次方程式の判別式をDとすると, D=4a+12 ここでD≧0ならば@は実数解をもち,この2つの曲線は共有点をもつと考えたのですが,4a+12≧0を解くとa≧-3 これは2つの曲線が共有点をもつための条件になっていません。 なぜこんなことが起こるのか,間違いはどこにあるのか,教えてください
113 名前:132人目の素数さん [2017/11/04(土) 08:18:59.13 ID:99rphagA.net] https://i.imgur.com/C93pCWC.png
114 名前:132人目の素数さん mailto:sage [2017/11/04(土) 08:25:42.46 ID:wShE/PHI.net] ん
115 名前:132人目の素数さん mailto:sage [2017/11/04(土) 08:39:05.60 ID:IN31GEoq.net] >>110 @がyの方程式として実数解をもっても、もう一方の未知数xが実数になるとは限らない。
116 名前:132人目の素数さん [2017/11/04(土) 09:02:18.95 ID:OJrl3+LI.net] >>113 感動しました。ありがとうございます
117 名前:132人目の素数さん mailto:sage [2017/11/04(土) 10:45:05.85 ID:b2rnI0sj.net] >>101 -(0.28125n +0.865 -0.6/n + …)ぐらい
118 名前:132人目の素数さん [2017/11/04(土) 10:59:58.35 ID:iK/EeUea.net] 距離空間 X はコンパクトでなくかつ全有界であるとする。 (U_λ)λ∈Λ を X の開被覆とし、 X が U_λ のうちの有限個では被覆されないと仮定する。 X は全有界であるから、半径 1/2 の有限個の開球によって被覆される。もし、これらの有限個の 開球がすべて U_λ のうちの有限個で被覆されるならば X 自身被覆可能となるから、これらの 開球のうちには U_λ のうちの有限個では被覆されないものが存在する。その1つを B_1 とする。 次に、 X は半径 1/2^2 の有限個の開球によって被覆されるが、上と同様の考察からわかるように、 それらのうちに、 B_1 と空でない共通部分をもち、かつ U_λ のうちの有限個では被覆されない ものが存在する。その1つを B_2 とする。 「B_1 と空でない共通部分をもち、かつ U_λ のうちの有限個では被覆されない ものが存在する。」とありますが、それはなぜでしょうか?
119 名前:132人目の素数さん mailto:sage [2017/11/04(土) 11:01:52.71 ID:b2rnI0sj.net] >>110 a<-3 なし a=-3 2個(x,y)=(±1/√2,1/2) -3<a<-2√2 4個 a=-2√2 3個(x,y)=(0,-1/√2)(±√{2(√2 -1)},1/√2 -1) -2√2<a<2√2 2個 a=2√2 1個(x,y)=(0,1/√2) 2√2<a なし
120 名前:132人目の素数さん [2017/11/04(土) 11:20:24.80 ID:iK/EeUea.net] >>116 あ、分かりました。 X ⊃ B_1 は半径 1/2^2 の有限個の開球によって被覆される。 B_1 と空でない共通部分をもつ半径 1/2^2 の(有限個の)開球がすべて U_λ のうちの有限個で 被覆されるならば B_1 も U_λ のうちの有限個で被覆されることになる。
121 名前:132人目の素数さん mailto:sage [2017/11/04(土) 11:27:41.22 ID:b2rnI0sj.net] >>101 -(0.282n +0.835 -0.312/n +…)ぐらい
122 名前:98 mailto:sage [2017/11/04(土) 12:26:05.91 ID:tlOJv18D.net] >>102 ありがとうがざいます。 これで練習問題とか Rudin は何考えてたんだ...
123 名前:132人目の素数さん [2017/11/04(土) 12:30:38.34 ID:PARh0rbO.net] 簡単な手の運動以外は練習問題に載せるなってことか?
124 名前:132人目の素数さん mailto:sage [2017/11/04(土) 13:38:10.24 ID:tlOJv18D.net] 載せるなら載せるで多少の配慮が欲しいところ。 よくある、ちょい難し目の問題には ✳ マークみたいなので 2、3日考えて分かんない時の見切り判断にはなるでしょう。延々と心残りにしなくて済むし。
125 名前:132人目の素数さん [2017/11/04(土) 14:23:31.44 ID:V33uwbBg.net] >>120 いえいえ ちょっと難易度高すぎよね 深く考えると面白い結論を自分で再発見できるから良い演習問題なんだろうけど Szegőの1920年代の結果と知って昔の人はこの手の方向では 物知りだったんだろうなぁと思ったわ
126 名前:¥ mailto:sage [2017/11/04(土) 15:45:11.13 ID:ayXvmvB0.net] ¥
127 名前:¥ mailto:sage [2017/11/04(土) 15:45:28.58 ID:ayXvmvB0.net] ¥
128 名前:¥ mailto:sage [2017/11/04(土) 15:45:43.19 ID:ayXvmvB0.net] ¥
129 名前:¥ mailto:sage [2017/11/04(土) 15:45:58.03 ID:ayXvmvB0.net] ¥
130 名前:¥ mailto:sage [2017/11/04(土) 15:46:11.74 ID:ayXvmvB0.net] ¥
131 名前:¥ mailto:sage [2017/11/04(土) 15:46:42.79 ID:ayXvmvB0.net] ¥
132 名前:¥ mailto:sage [2017/11/04(土) 15:46:59.97 ID:ayXvmvB0.net] ¥
133 名前:¥ mailto:sage [2017/11/04(土) 15:47:15.78 ID:ayXvmvB0.net] ¥
134 名前:¥ mailto:sage [2017/11/04(土) 15:47:32.39 ID:ayXvmvB0.net] ¥
135 名前:¥ mailto:sage [2017/11/04(土) 15:47:58.14 ID:ayXvmvB0.net] ¥
136 名前:132人目の素数さん [2017/11/04(土) 16:41:53.00 ID:nneIbVRy.net] ガウス積分の解法ってこれでええの?→ ∫[-∞→∞]e^-x^2dx=I とおいて I^2=∫[-∞→∞]e^-x^2dx∫[-∞→∞]e^-y^2dy =∫[-∞→∞]e^(-x^2-y^2) x=rcosθ y=rsinθ と置換するとヤコビアンはrなので I^2=∫[0→∞]∫[0→2π]e^-r^2 rdθdr =2π∫[0→∞]e^-r^2 rdr =(計算していくと…) =π ∵I=√π
137 名前:132人目の素数さん [2017/11/04(土) 17:09:44.09 ID:x3IALP1l.net] 津田
138 名前:132人目の素数さん [2017/11/04(土) 18:17:47.98 ID:nneIbVRy.net] 自作問なんですけど f(x)=xlogx-100=0の根をニュートンの方法を使って求めなさい。 御教授お願いします。
139 名前:∵I mailto:はい駐車 [2017/11/04(土) 18:19:36.94 ID:I/NX3QRO.net] いいとおもうけど よけいなこといわないで =2π∫[0→∞]e^-r^2 rdr (=(計算していくと…) ) =2π∫[0→∞]e^-r^2 d(r^2)/2=π∫[0→∞]e^-x dx =π
140 名前:132人目の素数さん [2017/11/04(土) 18:34:45.55 ID:I/NX3QRO.net] x-f(x)/f'(x)=x - (-100 + x log(x))/(1 +log(x)) を繰り返し適用する x=29.5366.... に収束する。
141 名前:132人目の素数さん [2017/11/04(土) 19:14:21.08 ID:ou5PcGSA.net] 大日如来とアルキメデスはどっちの方が凄いですか?
142 名前:132人目の素数さん mailto:sage [2017/11/04(土) 20:03:12.51 ID:QS066o32.net] 無意味な事書いて楽しい?
143 名前:132人目の素数さん [2017/11/04(土) 20:16:11.78 ID:ou5PcGSA.net] 数学板の皆さんは、「最強妄想キャラクター議論スレ」というのをご存知でしょうか? あれで暫定1位のキャラを作れますか? 数学板の皆さんの知力で暫定1位のキャラを作ってみてください。 最強妄想キャラクター議論スレ32 https://mao.5ch.net/test/read.cgi/ranking/1494767095/ これで暫定1位のキャラを作るのってかなり難しいんですよ。 フィールズ賞を受賞するのより難しいかもしれません。 是非挑戦してみてください。
144 名前:132人目の素数さん [2017/11/04(土) 21:29:32.51 ID:nneIbVRy.net] >>137 ありがとうございます。
145 名前:132人目の素数さん [2017/11/04(土) 21:30:06.79 ID:nneIbVRy.net] >>138 分かりました。ありがとうございます。
146 名前:¥ mailto:sage [2017/11/04(土) 21:59:09.30 ID:ayXvmvB0.net] ¥
147 名前:¥ mailto:sage [2017/11/04(土) 21:59:26.46 ID:ayXvmvB0.net] ¥
148 名前:¥ mailto:sage [2017/11/04(土) 21:59:43.83 ID:ayXvmvB0.net] ¥
149 名前:¥ mailto:sage [2017/11/04(土) 22:00:02.43 ID:ayXvmvB0.net] ¥
150 名前:¥ mailto:sage [2017/11/04(土) 22:00:20.97 ID:ayXvmvB0.net] ¥
151 名前:¥ mailto:sage [2017/11/04(土) 22:00:37.26 ID:ayXvmvB0.net] ¥
152 名前:¥ mailto:sage [2017/11/04(土) 22:00:55.31 ID:ayXvmvB0.net] ¥
153 名前:¥ mailto:sage [2017/11/04(土) 22:01:12.92 ID:ayXvmvB0.net] ¥
154 名前:¥ mailto:sage [2017/11/04(土) 22:01:31.91 ID:ayXvmvB0.net] ¥
155 名前:¥ mailto:sage [2017/11/04(土) 22:01:49.51 ID:ayXvmvB0.net] ¥
156 名前:132人目の素数さん mailto:sage [2017/11/04(土) 22:50:26.76 ID:jZgAiEKg.net] ヒルベルト形式
157 名前:フ証明体系の話です(命題論理)。 系の証明が分かりません。無矛盾の定義もかなり特殊なような気がするのですが……どなたかお願いします。 https://i.imgur.com/YRtu8Sg.jpg [] [ここ壊れてます]
158 名前:132人目の素数さん mailto:sage [2017/11/04(土) 22:57:46.33 ID:b2rnI0sj.net] >>134 余計なこと言いますが ∫[-R→+R]e^(-xx)dx = I(R), とおいて I(R)^2 =∫[-R→+R]e^(-xx)dx∫[-R→+R]e^(-yy)dy ≧∫[半径Rの円]e^{-(xx+yy)}dxdy =π{1 -e^(-RR)}, I(R)^2 =∫[-R→+R]e^(-xx)dx∫[-R→+R]e^(-yy)dy ≦∫[半径R√2の円]e^{-(xx+yy)}dxdy =π{1-e^(-2RR)}, にて挟み撃ち。
159 名前:132人目の素数さん mailto:sage [2017/11/04(土) 23:23:06.18 ID:qn/JBcju.net] >>154 読めません
160 名前:132人目の素数さん mailto:sage [2017/11/04(土) 23:29:29.12 ID:b2rnI0sj.net] >>120 >>123 Rudin(の問題)のツボが割れた
161 名前:132人目の素数さん mailto:sage [2017/11/04(土) 23:30:45.29 ID:jZgAiEKg.net] >>156 申し訳ない https://i.imgur.com/p9pzQBk.jpg
162 名前:132人目の素数さん mailto:sage [2017/11/04(土) 23:39:24.11 ID:rPV7C8r3.net] 劣等感婆の香りがする
163 名前:132人目の素数さん mailto:sage [2017/11/04(土) 23:45:46.27 ID:qn/JBcju.net] >>158 対偶を取り、充足可能ではないならば矛盾していることを示します Γは充足可能ではないので、Γのある論理式の集まりφが存在して、φがトートロジーとなるようなφ∈Γが存在します 完全性定理により|-¬φすなわちφ|-すなわちΓ|-証明可能となり、Γは矛盾しています
164 名前:132人目の素数さん mailto:sage [2017/11/04(土) 23:46:14.75 ID:qn/JBcju.net] φは集まりではなく論理式ですね、ただの
165 名前:132人目の素数さん mailto:sage [2017/11/04(土) 23:47:03.19 ID:qn/JBcju.net] >>158 対偶を取り、充足可能ではないならば矛盾していることを示します Γは充足可能ではないので、Γのある論理式の集まりφが存在して、¬φがトートロジーとなるようなφ∈Γが存在します 完全性定理により|-¬φすなわちφ|-すなわちΓ|-証明可能となり、Γは矛盾しています
166 名前:132人目の素数さん mailto:sage [2017/11/04(土) 23:47:39.58 ID:qn/JBcju.net] >>158 対偶を取り、充足可能ではないならば矛盾していることを示します Γは充足可能ではないので、Γのある論理式φが存在して、¬φがトートロジーとなるようなφ∈Γが存在します 完全性定理により|-¬φすなわちφ|-すなわちΓ|-証明可能となり、Γは矛盾しています あーもう
167 名前:132人目の素数さん mailto:sage [2017/11/04(土) 23:57:51.96 ID:jZgAiEKg.net] >>163 素早い回答ありがとうございます。とても分かりやすく理解できました!
168 名前:132人目の素数さん mailto:sage [2017/11/04(土) 23:58:12.12 ID:qn/JBcju.net] 今日も「解いた側」の圧勝かぁ・・・。 毎日毎日、ラクラク解ける問題ばかりだから常勝なんだよね・・・。 たまには、解けない解けないっと悩んで負けてみたい、それが今の切実な悩み。
169 名前:132人目の素数さん [2017/11/04(土) 23:59:20.09 ID:rc8qWf13.net] 関数f(x)がf(x)=0→1(x+t)f(t)dtという関係を満たすとき、f(x)を求めよ
170 名前:132人目の素数さん mailto:sage [2017/11/05(日) 00:08:08.45 ID:AyBYVZcE.net] 0
171 名前:132人目の素数さん mailto:sage [2017/11/05(日) 00:10:51.99 ID:9es9BySf.net] nを自然数とする。(n^n)/k!が整数になるような自然数kの最小値をf(n)とおく。 lim(n→∞)f(n)/n^aが0でない実数値に収束する実数aを求めよ。また、f(n)を求めよ。
172 名前:132人目の素数さん mailto:sage [2017/11/05(日) 00:15:14.54 ID:U+hsGOZU.net] >>168 kの最小値ではなく最大値では
173 名前:132人目の素数さん mailto:sage [2017/11/05(日) 00:17:31.43 ID:U+hsGOZU.net] そもそも命題はただしいか?これ
174 名前:132人目の素数さん [2017/11/05(日) 00:20:39.65 ID:WxqRvsvz.net] アルキメデスは人類史上トップクラスの天才なのでしょうか?
175 名前:132人目の素数さん mailto:sage [2017/11/05(日) 00:23:14.48 ID:toHRlwqr.net] >>168 a=0、f(n)=1
176 名前:132人目の素数さん [2017/11/05(日) 00:35:55.27 ID:WxqRvsvz.net] 秘密曼陀羅十住心論を独力で読破するのは不可能ですか?
177 名前:132人目の素数さん mailto:sage [2017/11/05(日) 01:18:38.62 ID:9es9BySf.net] >>169 すいません最大値です
178 名前:132人目の素数さん mailto:sage [2017/11/05(日) 01:18:59.08 ID:9es9BySf.net] >>172 最大値でした、すいません
179 名前:132人目の素数さん mailto:sage [2017/11/05(日) 05:23:06.85 ID:Eblk41OG.net] 段階的な質問(何度かに分けた質問)になるかもしれません 同じ大きさ(面積S1とします)、形のn枚の紙に合計面積がS2となるように複数の穴を開けます(穴の数、大きさ、分布はある適当な法則にしたがってそれぞれの紙でランダムに決まります) うちk枚の紙を重ねて1枚の紙にしたときに出来る穴の面積をS(k)とします 実際に数値としてS(1)〜S(n)が与えられた時、それぞれの数値から近似関数S(x)を求めたいです その際はどのような手法を用いるのが適切でしょうか n、S1、S2によって全く異なるとは思うのですが、一応下記を想定しております nは十分に大きく、S1>S2かつS1≒S2、S(n)≒0 非常に要領を得ない質問かと思いますが、何かヒントを頂けますと幸いです
180 名前:132人目の素数さん [2017/11/05(日) 09:14:42.22 ID:pAiKdsy1.net] >>166 ?となっている部分はインテグラルです
181 名前:132人目の素数さん [2017/11/05(日) 10:29:00.00 ID:bJHsCCAe.net] >>159 鋭い
182 名前:132人目の素数さん [2017/11/05(日) 10:34:21.67 ID:bJHsCCAe.net] >>166 f(x)=(∫[0,1]f(t)dt)x+(∫[0,1]tf(t)dt) という1次関数だから a=∫[0,1]f(t)dt b=∫[0,1]tf(t)dt と置いて a=∫[0,1](at+b)dt b=∫[0,1]t(at+b)dt の連立1次方程式を解く
183 名前:132人目の素数さん [2017/11/05(日) 10:42:04.54 ID:bJHsCCAe.net] >>168 nが奇数ならf(n)=1でしょ だったらa=0しかあり得ない
184 名前:132人目の素数さん [2017/11/05(日) 11:05:42.51 ID:3IMErAk8.net] ユークリッド空間 R^n は可分であることを証明せよ。 以下の解答であっていますか? 可算集合 Q^n が R^n において密であることを証明する。 Q^n が R^n において密である ⇔ R^n の任意の空でない開集合 U に対して U ∩ Q^n ≠ 空集合 U を R^n の任意の空でない開集合とする。 a = (a_1, …, a_n) を U の任意の元とする。 ∃r > 0 s.t. B(a ; r) ⊂ U. R における Q の稠密性により、 |x_i - a_i| < r/n をみたす x_i ∈ Q が存在する。 x = (x_1, …, x_n) ∈ Q^n. sqrt( (x_1 - a_1)^2 + … + (x_n - a_n)^2 ) ≦ |x_1 - a_1| + … + |x_n - a_n| < r であるから、 x ∈ B(a ; r) ⊂ U. ∴U ∩ Q^n ≠ 空集合
185 名前:132人目の素数さん [2017/11/05(日) 11:22:14.98 ID:bJHsCCAe.net] >>181 いんじゃね
186 名前:132人目の素数さん [2017/11/05(日) 11:25:54.69 ID:3IMErAk8.net] >>182 ありがとうございました。
187 名前:132人目の素数さん mailto:sage [2017/11/05(日) 11:29:32.69 ID:9cLr18hq.net] >>176 確率の問題とすれば 1枚で穴に当たる確率はS2/S1だから k枚で共通に穴になる確率は(S2/S1)^k
188 名前:132人目の素数さん [2017/11/05(日) 11:57:31.49 ID:pAiKdsy1.net] >>179 ありがとうございます
189 名前:132人目の素数さん [2017/11/05(日) 11:58:52.12 ID:uxMFapcs.net] 位数60未満の有限群は可解であることを示せ
190 名前:132人目の素数さん mailto:sage [2017/11/05(日) 12:30:29.09 ID:Eblk41OG.net] >>184 非常にクリアカットな解答ありがとうございます 恥ずかしながら確率の問題として捉える頭がなかったのですが、僕の求める解答に最も近いと思います それではもう一つお願いします 上記の問題でS1,n,S2の設定は同様です それとは別にある定数S3があります k枚の紙を重ねた時、開いている穴の数をm(k)とします そしてそれぞれの穴の面積をs(m)とします(m=1〜m(k)) その際、 s3<s(m)となるような穴の数N(k)が各kについて実際の数値として与えられた時、N(k)の近似関数を求めたい もう一つは S(k)=Σ[m=1,m(k)] [s(m)/s3] と定義して各kについてそれが実際の数値として与えられた時、その近似関数を求めたい([x]はxを超えない最大の整数(床関数)です) 言ってしまったらk枚の紙を重ねた時、面積s3の紙がどれだけ穴に収納できるかを知りたいです もちろん穴や当てはめる紙の形によって異なるんですが、もうそこまでは難しいような気がするので
191 名前:132人目の素数さん mailto:sage [2017/11/05(日) 13:23:39.84 ID:eHfaSYyq.net] 次の条件を満たす勝負を作成し、そのポイントと勝つ確率が正しいことを数学的に説明せよ。 ・二人間で勝ち負けの定義ができる勝負 ・引き分けを除いた勝つ確率が、1/2または2/3になる勝負 ・勝負の中で選択肢がある時、どちらも最善手を選択する、最善手が無ければ無作為に選択する ・理解しやすく、興味を引く勝負 ・勝つ確率の計算が数学的にみて質
192 名前:ェ高い また、結果に意外性があるとよい。 [] [ここ壊れてます]
193 名前:132人目の素数さん [2017/11/05(日) 13:58:36.12 ID:qiLs7GcW.net] 分からない問題だからスレチじゃないけど どこらへんが数学なの?
194 名前:132人目の素数さん [2017/11/05(日) 14:27:40.83 ID:3IMErAk8.net] 松坂和夫著『解析入門3』を読んでいます。 距離空間の「可分」という概念はどのような場面で役に立つのでしょうか? 解析入門にはその後、おそらく「可分」は使われないと思いますが。
195 名前:132人目の素数さん [2017/11/05(日) 15:43:56.69 ID:Cg8PcjKl.net] ダライ・ラマとオックスフォード大学総長はどっちの方が凄いですか?
196 名前:132人目の素数さん [2017/11/05(日) 15:52:46.66 ID:UUPpPjRU.net] ↑ マラとドブスのどっちがすごいのですか?
197 名前:132人目の素数さん [2017/11/05(日) 15:57:00.51 ID:2zo8aL1a.net] googleの人は伊藤さん?
198 名前:132人目の素数さん [2017/11/05(日) 16:02:18.09 ID:Cg8PcjKl.net] 世界の宗教権威の序列ってこんな感じですか? 1位、ローマ教皇 2位、コンスタンディヌーポリ総主教 3位、アル=アズハル大学総長 4位、ダライ・ラマ 5位、天皇
199 名前:132人目の素数さん mailto:sage [2017/11/05(日) 21:24:51.38 ID:L8HZuN+O.net] >>1 人類初!自分が作ったロボットに暴行を受けた男 https://youtu.be/pOdUO8DmPoU
200 名前:132人目の素数さん mailto:sage [2017/11/05(日) 21:29:47.42 ID:6KxvYVjk.net] >>193 赤池弘次さん生誕90周年らしい >>194 名誉博士号の数ならこの人? www.totetu.org/about/about-founder/1.html
201 名前:132人目の素数さん mailto:sage [2017/11/05(日) 22:43:03.70 ID:bakzJXn3.net] すみません x→x0の時に、(f(x)-f(x0))/(x-x0)が発散することはありますか? また、ないならそれを示すことはできますか? 証明の途中にそれを使いたくて、分かんなかったので、教えてください。
202 名前:132人目の素数さん [2017/11/05(日) 22:59:33.50 ID:RSWE/Y5W.net] x0 = 0 f(x) = 1 (x ≠ 0) f(0) = 0
203 名前:132人目の素数さん [2017/11/05(日) 23:10:53.03 ID:bJHsCCAe.net] f(x)=x^(1/3) x0=0
204 名前:132人目の素数さん [2017/11/05(日) 23:27:07.75 ID:z4nCSBHU.net] 微分係数が発散する(かもしれない)ことくらいはイメージ出来るようにね
205 名前:132人目の素数さん [2017/11/05(日) 23:57:40.22 ID:ESLgm1tN.net] ゴールドバッハ予想、6以上の偶数は二つの奇素数の和で表現できる。 3以上の整数Nに対して、ある正整数Kが存在して N−K N+K ともに素数となる、これは同じ命題で良いですか??
206 名前:132人目の素数さん [2017/11/06(月) 00:03:16.72 ID:bIi1g6CH.net] >>201 よいよ
207 名前:132人目の素数さん mailto:sage [2017/11/06(月) 00:58:34.22 ID:N1m+ckrX.net] 素数が無限にあるのを背理法で証明する問題ありますけど 素数みたいに範囲の有無というか範囲の内外にあるかないかで矛盾を指摘するみたいなものじゃなくて 量を測れないものの性質の有無を背理法で証明する具体例で適当なのなんかないですか? なんかイメージしにくくて…w
208 名前:132人目の素数さん [2017/11/06(月) 01:05:54.10 ID:vATXcroC.net] 「人生に飽きたから自殺したい」 これを数式で表すとどうなりますか?
209 名前:132人目の素数さん mailto:sage [2017/11/06(月) 01:07:50.51 ID:bJJhaPFC.net] >>203 背理法がわからないので素数以外の具体例が欲しい、ということで良いでしょうか?
210 名前:132人目の素数さん mailto:sage [2017/11/06(月) 01:09:32.59 ID:N1m+ckrX.net] >>203 一応分かってるつもりなんですが 他に具体例があれば…
211 名前:132人目の素数さん mailto:sage [2017/11/06(月) 01:14:27.89 ID:N1m+ckrX.net] >>204 やっぱアホな質問でしたw
212 名前:132人目の素数さん mailto:sage [2017/11/06(月) 01:15:56.00 ID:bJJhaPFC.net] >>206 今日の正午に公園で殺人事件が起きたとします 容疑者としてXがあがりました しかし、Xにはアリバイがあり、今日の正午には遊園地にいたとします Xが犯人ではな
213 名前:「ことを背理法を用いて証明します Xが犯人だと仮定します すると、Xは今日の正午には公園にいたことになります しかし、実際にはXは今日の正午には遊園地にいたので公園にはいませんでした よって矛盾が生じたため、最初の仮定は誤りだったことがわかります すなわち、Xは犯人ではありません [] [ここ壊れてます]
214 名前:132人目の素数さん mailto:sage [2017/11/06(月) 01:22:59.14 ID:N1m+ckrX.net] >>208 どうもわざわざありがと。 数学にも論理にも収まらないような 漠然とした背理法というか 背理法的だけど背理法と言っていいのか分からないような 感情的な背理法?につまずいてて…w
215 名前:132人目の素数さん mailto:sage [2017/11/06(月) 01:25:37.38 ID:bJJhaPFC.net] >>209 あんまりそういうことを深く考えると、数理論理学という分野の話が出てきたりしますから、なんとなくわかってればいいんですよ
216 名前:132人目の素数さん mailto:sage [2017/11/06(月) 01:28:11.57 ID:N1m+ckrX.net] >>210 とりまなんとなくわかっときますw さんくす。
217 名前:¥ mailto:sage [2017/11/06(月) 12:37:17.49 ID:StBgmS4d.net] ¥
218 名前:¥ mailto:sage [2017/11/06(月) 12:37:35.56 ID:StBgmS4d.net] ¥
219 名前:¥ mailto:sage [2017/11/06(月) 12:37:56.67 ID:StBgmS4d.net] ¥
220 名前:¥ mailto:sage [2017/11/06(月) 12:38:13.07 ID:StBgmS4d.net] ¥
221 名前:¥ mailto:sage [2017/11/06(月) 12:38:29.63 ID:StBgmS4d.net] ¥
222 名前:¥ mailto:sage [2017/11/06(月) 12:38:47.11 ID:StBgmS4d.net] ¥
223 名前:¥ mailto:sage [2017/11/06(月) 12:39:04.86 ID:StBgmS4d.net] ¥
224 名前:132人目の素数さん mailto:sage [2017/11/06(月) 12:39:18.84 ID:guoBG3/k.net] /⌒ヽ おっはお♪ ∩ ^ω^) | ⊂ノ おっはお♪ | _⊃ し ⌒ ⌒ ⌒ ⌒ ピョンピョン
225 名前:¥ mailto:sage [2017/11/06(月) 12:39:23.56 ID:StBgmS4d.net] ¥
226 名前:¥ mailto:sage [2017/11/06(月) 12:39:41.02 ID:StBgmS4d.net] ¥
227 名前:¥ mailto:sage [2017/11/06(月) 12:58:34.91 ID:StBgmS4d.net] ¥
228 名前:¥ mailto:sage [2017/11/06(月) 12:58:52.63 ID:StBgmS4d.net] ¥
229 名前:¥ mailto:sage [2017/11/06(月) 12:59:10.62 ID:StBgmS4d.net] ¥
230 名前:¥ mailto:sage [2017/11/06(月) 12:59:27.42 ID:StBgmS4d.net] ¥
231 名前:¥ mailto:sage [2017/11/06(月) 12:59:44.82 ID:StBgmS4d.net] ¥
232 名前:¥ mailto:sage [2017/11/06(月) 13:00:02.59 ID:StBgmS4d.net] ¥
233 名前:¥ mailto:sage [2017/11/06(月) 13:00:21.30 ID:StBgmS4d.net] ¥
234 名前:¥ mailto:sage [2017/11/06(月) 13:00:40.18 ID:StBgmS4d.net] ¥
235 名前:132人目の素数さん mailto:sage [2017/11/06(月) 13:07:30.95 ID:0DZubHvh.net] 惨めな奴
236 名前:132人目の素数さん [2017/11/06(月) 15:39:10.48 ID:zixy2BZu.net] 斎藤正彦著『線型代数入門』を読んでいます。 付録の代数学の基本定理の証明ですが、ひどい証明ですね。 正しいことは分かるが、何の勉強にもならない証明ですね。
237 名前:132人目の素数さん mailto:sage [2017/11/06(月) 15:57:41.15 ID:dClhKN+Q.net] 物理板と違ってNGできる数学板は幸せだ
238 名前:132人目の素数さん [2017/11/06(月) 18:05:44.33 ID:vATXcroC.net] 正直、東京大学理学部数学科に入りたい。
239 名前:¥ mailto:sage [2017/11/06(月) 19:18:57.75 ID:StBgmS4d.net] ¥
240 名前:¥ mailto:sage [2017/11/06(月) 19:19:17.62 ID:StBgmS4d.net] ¥
241 名前:¥ mailto:sage [2017/11/06(月) 19:19:35.12 ID:StBgmS4d.net] ¥
242 名前:¥ mailto:sage [2017/11/06(月) 19:19:53.94 ID:StBgmS4d.net] ¥
243 名前:¥ mailto:sage [2017/11/06(月) 19:20:14.72 ID:StBgmS4d.net] ¥
244 名前:¥ mailto:sage [2017/11/06(月) 19:20:33.40 ID:StBgmS4d.net] ¥
245 名前:¥ mailto:sage [2017/11/06(月) 19:20:53.45 ID:StBgmS4d.net] ¥
246 名前:¥ mailto:sage [2017/11/06(月) 19:21:12.78 ID:StBgmS4d.net] ¥
247 名前:¥ mailto:sage [2017/11/06(月) 19:21:32.46 ID:StBgmS4d.net] ¥
248 名前:¥ mailto:sage [2017/11/06(月) 19:21:52.76 ID:StBgmS4d.net] ¥
249 名前:132人目の素数さん [2017/11/06(月) 19:29:58.49 ID:hgxZ+vC7.net] 1以下、5ちゃんねるからVIPがお送りします2017/11/06(月) 19:11:25.756ID:S/Jsj/CRM P(n)=[cos^2(((n-1)!+1)π/n)] ([x]はxを超えない最大の整数) とすれば、 nが素数のときはP(n)=1 nが素数でないときはP(n)=0 に必ずなる 4以下、5ちゃんねるからVIPがお送りします2017/11/06(月) 19:12:15.114ID:S/Jsj/CRM さらにこのP(n)を使えば n以下の素数の個数が (k=2,n)P(k)と書けてしまう 10以下、5ちゃんねるからVIPがお送りします2017/11/06(月) 19:14:52.904ID:S/Jsj/CRM >>4 しかもこれを π(n)=(k=2,n)P(k)とおいてしまえば n番目の素数もπ(n)使って簡単に表せる これが正しいのか 正しいとして凄い発見なのか誰か教えてくれ
250 名前:132人目の素数さん [2017/11/06(月) 19:31:33.54 ID:kabw670x.net] nP(n)でよくね
251 名前:¥ mailto:sage [2017/11/06(月) 21:17:03.84 ID:StBgmS4d.net] ¥
252 名前:¥ mailto:sage [2017/11/06(月) 21:17:22.60 ID:StBgmS4d.net] ¥
253 名前:¥ mailto:sage [2017/11/06(月) 21:17:39.49 ID:StBgmS4d.net] ¥
254 名前:¥ mailto:sage [2017/11/06(月) 21:17:55.55 ID:StBgmS4d.net] ¥
255 名前:¥ mailto:sage [2017/11/06(月) 21:18:13.41 ID:StBgmS4d.net] ¥
256 名前:¥ mailto:sage [2017/11/06(月) 21:18:30.72 ID:StBgmS4d.net] ¥
257 名前:¥ mailto:sage [2017/11/06(月) 21:18:49.11 ID:StBgmS4d.net] ¥
258 名前:¥ mailto:sage [2017/11/06(月) 21:19:07.82 ID:StBgmS4d.net] ¥
259 名前:¥ mailto:sage [2017/11/06(月) 21:19:27.33 ID:StBgmS4d.net] ¥
260 名前:¥ mailto:sage [2017/11/06(月) 21:19:47.43 ID:StBgmS4d.net] ¥
261 名前:132人目の素数さん mailto:sage [2017/11/06(月) 21:21:09.93 ID:WTbJbUlz.net] >>231 勉強に なる/ならない の線引きは 松阪くんがするの?
262 名前:132人目の素数さん mailto:sage [2017/11/06(月) 21:55:20.82 ID:nBOos3E3.net] >>244 (n-1)!/nが、非整数か整数かは、nが素数か非素数かに一致する・・・・(☆) ただし、n=4(=2^2)は例外 というのとほぼ同値
263 名前:¥ mailto:sage [2017/11/06(月) 22:58:36.59 ID:StBgmS4d.net] ¥
264 名前:¥ mailto:sage [2017/11/06(月) 22:58:55.35 ID:StBgmS4d.net] ¥
265 名前:¥ mailto:sage [2017/11/06(月) 22:59:12.79 ID:StBgmS4d.net] ¥
266 名前:¥ mailto:sage [2017/11/06(月) 22:59:29.91 ID:StBgmS4d.net] ¥
267 名前:¥ mailto:sage [2017/11/06(月) 22:59:48.94 ID:StBgmS4d.net] ¥
268 名前:¥ mailto:sage [2017/11/06(月) 23:00:07.39 ID:StBgmS4d.net] ¥
269 名前:¥ mailto:sage [2017/11/06(月) 23:00:26.07 ID:StBgmS4d.net] ¥
270 名前:¥ mailto:sage [2017/11/06(月) 23:00:42.86 ID:StBgmS4d.net] ¥
271 名前:¥ mailto:sage [2017/11/06(月) 23:01:02.43 ID:StBgmS4d.net] ¥
272 名前:¥ mailto:sage [2017/11/06(月) 23:01:21.
] [ここ壊れてます]
273 名前:89 ID:StBgmS4d.net mailto: ¥ [] [ここ壊れてます]
274 名前:132人目の素数さん mailto:sage [2017/11/07(火) 00:20:39.21 ID:1/pN97hc.net] W.Rudin, Real and Complex Analysis, Exercise 11.11 I=[a,b]、ΩをIを含む連結な開集合、fをΩで連続かつΩ-Iで解析的な関数ならば、 fがΩ上で解析的となることを示せ 同じ結論が成り立つ集合はI以外にどのようなものが考えられるか (答え?) https://en.wikipedia.org/wiki/Analytic_capacity https://books.google.co.jp/books?id=Qhz0CAAAQBAJ
275 名前:132人目の素数さん [2017/11/07(火) 11:27:04.24 ID:Owsch49+.net] R の部分集合 I が区間であることとつぎの二条件をみたすことは同値である。 1) I は少なくとも二点を含む。 2) x, y ∈ I, x < z < y なら z ∈ I. これを証明してください。
276 名前:132人目の素数さん mailto:sage [2017/11/07(火) 12:14:51.18 ID:kJC0rJd6.net] >>269 実数は全順序集合 Iが上に(下に)有界ならば、Zornの補題から上限の(下限)存在がいえる 有界でなければ無限大を含む(半)直線になる
277 名前:132人目の素数さん mailto:sage [2017/11/07(火) 12:51:55.06 ID:0k/YWviz.net] フラッグマン・リンデレーエフの定理ではどうして、仮定では領域Dは有界とあるのでしょうか? 最大値の原理では有界は条件では無いのに、拡張されたフラッグマンの定理では有界が仮定されている理由が分かりません 勉強中で若輩者ですが、どなたか心優しい方教えて頂けないでしょうか?
278 名前:132人目の素数さん mailto:sage [2017/11/07(火) 13:01:12.82 ID:5o/NxnVm.net] >>270 そんなに詳しくないのですが、それZornの補題いりますか? 上限の(下限)存在は実数の性質から言えるかと...
279 名前:132人目の素数さん mailto:sage [2017/11/07(火) 13:22:53.76 ID:kJC0rJd6.net] >>272 確かに実数の性質っていった方がわかりやすいですね 僕も詳しくないですが、有界な実数集合の上限の存在がZornの補題から従うので 恐らく連続体公理を仮定するか、Zornの補題(選択公理と同値)を仮定するかの違いです
280 名前:132人目の素数さん mailto:sage [2017/11/07(火) 13:54:42.58 ID:1JSB0KNQ.net] は?
281 名前:132人目の素数さん [2017/11/07(火) 14:42:07.53 ID:Owsch49+.net] >>269 松坂和夫著『解析入門1』に書いてありました↓ でも、まずいところがありますよね。 A を少なくとも2つの数を含むような R の部分集合とし、 s, t ∈ A, s < t ならば、 s < c <t を満たす任意の c も A に属するとする。 そのとき、 A は1つの区間である。 証明 まず A が上下に有界であるとする。そのとき、 a = inf A, b = sup A とおけば、 A ⊂ [a, b] で、一方 a < c < b とすれば、下限、上限の定義によって a < s < c < t < b を満たす s, t ∈ A が存在する
282 名前:132人目の素数さん [2017/11/07(火) 14:47:50.78 ID:Owsch49+.net] ↓のように書かないとダメですよね。 まず A が上下に有界であるとする。 a = inf A b = sup A とおく。 A ⊂ [a, b] である。 a < c < b とする。 (1) a ∈ A であり, b ∈ A でない場合 上限の定義により、 c < t < b となる t ∈ A が存在する。 a < c < t だから、 c ∈ A である。 (2) a ∈ A でなく, b ∈ A である場合 下限の定義により、 a < s < c となる s ∈ A が存在する。 s < c < b だから、 c ∈ A である。 (3) a ∈ A でなく, b ∈ A でない場合 上限下限の定義により、 a < s < c < t < b となる s, t ∈ A が存在する。 s < c < t だから、 c ∈ A である。 (3) a ∈ A であり b ∈ A である場合 a < c < b だから、 c ∈ A である。 a < s < c < t < b を満たす s, t ∈ A が存在する
283 名前:132人目の素数さん [2017/11/07(火) 14:49:49.62 ID:Owsch49+.net] >>276 訂正します: ↓のように書かないとダメですよね。 まず A が上下に有界であるとする。 a = inf A b = sup A とおく。 A ⊂ [a, b] である。 a < c < b とする。 (1) a ∈ A であり, b ∈ A でない場合 上限の定義により、 c < t < b となる t ∈ A が存在する。 a < c < t だから、 c ∈ A である。 (2) a ∈ A でなく, b ∈ A である場合 下限の定義により、 a < s < c となる s ∈ A が存在する。 s < c < b だから、 c ∈ A である。 (3) a ∈ A でなく, b ∈ A でない場合 上限下限の定義により、 a < s < c < t < b となる s, t ∈ A が存在する。 s < c < t だから、 c ∈ A である。 (3) a ∈ A であり b ∈ A である場合 a < c < b だから、 c ∈ A である。
284 名前:132人目の素数さん mailto:sage [2017/11/07(火) 16:11:00.07 ID:BsVl5J45.net] Kを代数体 d(K)をKの判別式 d(a_0,a_1,•••,a_n)をはa_0,•••,a_nの判別式とする ただし、a_iはKの整数 このとき、d(K)がd(a_0,a_1,•••,a_n)を割る条件を求めよ これがわかりません。お願いします。
285 名前:¥ mailto:sage [2017/11/07(火) 17:17:11.16 ID:qfDf0bUe.net] ¥
286 名前:¥ mailto:sage [2017/11/07(火) 17:17:28.59 ID:qfDf0bUe.net] ¥
287 名前:¥ mailto:sage [2017/11/07(火) 17:17:44.16 ID:qfDf0bUe.net] ¥
288 名前:¥ mailto:sage [2017/11/07(火) 17:17:59.69 ID:qfDf0bUe.net] ¥
289 名前:¥ mailto:sage [2017/11/07(火) 17:1
] [ここ壊れてます]
290 名前:8:17.39 ID:qfDf0bUe.net mailto: ¥ [] [ここ壊れてます]
291 名前:¥ mailto:sage [2017/11/07(火) 17:18:33.99 ID:qfDf0bUe.net] ¥
292 名前:¥ mailto:sage [2017/11/07(火) 17:18:50.86 ID:qfDf0bUe.net] ¥
293 名前:¥ mailto:sage [2017/11/07(火) 17:19:07.10 ID:qfDf0bUe.net] ¥
294 名前:¥ mailto:sage [2017/11/07(火) 17:19:24.19 ID:qfDf0bUe.net] ¥
295 名前:¥ mailto:sage [2017/11/07(火) 17:19:42.58 ID:qfDf0bUe.net] ¥
296 名前:132人目の素数さん [2017/11/07(火) 20:42:22.56 ID:tIMewAzz.net] 斎藤正彦さんの『行列と群』という本の中で、 f(z) を実係数多項式とし、複素係数の多項式として f(z) = g(z)*h(z) と分解されるとする。 このとき、g(z) が実係数多項式ならば h(z) も実係数多項式である という論法が使われているのですが、証明を教えてください。
297 名前:132人目の素数さん mailto:sage [2017/11/07(火) 20:50:25.57 ID:3vuQU0Uc.net] ほとんど自明なのでは
298 名前:132人目の素数さん mailto:sage [2017/11/07(火) 20:51:42.15 ID:o8b3Czlh.net] わからないんですね(笑)
299 名前:132人目の素数さん mailto:sage [2017/11/07(火) 20:57:31.66 ID:C8Zmg5mj.net] 複素共役を使うのでは
300 名前:132人目の素数さん [2017/11/07(火) 21:00:04.52 ID:lN2mTJce.net] その本は読んだことないですが、全く自明ではありませんね
301 名前:132人目の素数さん [2017/11/07(火) 21:06:41.10 ID:tIMewAzz.net] f(z) の係数をすべてその共役複素数にした多項式を f'(z) と書くことにする。 f(z) = g(z) * h(z) f(z) は実係数多項式かつ f(z) ≠ 0 g(z) は実係数多項式かつ g(z) ≠ 0 とする。 明らかに、以下が成り立つ。 f'(z) = g'(z) * h'(z) f(z) = f'(z) g(z) = g'(z) だから f(z) = g(z) * h'(z) ∴ g(z) * h(z) = g(z) * h'(z) g(z) * (h(z) - h'(z)) = 0 g(z) ≠ 0 だから h(z) - h'(z) = 0 ∴ h(z) = h'(z) ∴ h(z) は実係数多項式である。
302 名前:132人目の素数さん [2017/11/07(火) 21:08:22.70 ID:tIMewAzz.net] >>289 今見てみたら、この論法が、佐武一郎さんの『線型代数学』の代数学の基本定理 のところで使われていました。
303 名前:132人目の素数さん [2017/11/07(火) 21:11:30.28 ID:lN2mTJce.net] >>294 は読まないことを強くおすすめします
304 名前:132人目の素数さん [2017/11/07(火) 21:25:13.50 ID:tIMewAzz.net] >>295 佐武一郎著『線型代数学(新装版)』のp.32の定理Bで、 >>289 の論法を使っています。 自明なことなのでしょうか?
305 名前:132人目の素数さん mailto:sage [2017/11/07(火) 21:28:27.78 ID:3vuQU0Uc.net] 背理法でzに適当な実数たくさん入れれば示せない?
306 名前:132人目の素数さん [2017/11/07(火) 21:28:46.64 ID:tIMewAzz.net] 具体的に言うと、 fx) = (x - ξ1) * (x - ξ1') * f1(x) f1(x) に対して再帰的に同じことを繰り返せば みたいな記述があります。 再帰的に同じことを繰り返すためには、 f1(x) も実係数多項式でなければなりません。
307 名前:132人目の素数さん mailto:sage [2017/11/07(火) 21:32:50.61 ID:C8Zmg5mj.net] >>294 横レスすまん 細かいが、zを実数変数x に置き換え h(x)=f(x) /g(x) としておく方が、すっきりしていると思う 複素共役の上線が書けないので、下記の上付きのアスタリスク (z*) を使う >>294 の屋上屋だが、h(x)の複素共役は h(x)* = {f(x) /g(x)}* = f(x)* /g(x)* =f(x) /g(x) =h(x) h(x)* = h(x) だから、h(x) も実係数多項式である (下記の複素共役 * の2つの性質1)2)を使った) https://ja.wikipedia.org/wiki/%E8%A4%87%E7%B4%A0%E5%85%B1%E5%BD%B9 複素共役 (抜粋) 複素共役を表すのには上線がよく使われる。上付きのアスタリスク (z*) なども使われるが、行列での随伴行列などとの混乱を避けるためにあまり使われない。 性質 1)z が実数 ←→ z*=z 2)(z/w)*=z*/w* (w ≠ 0)。 (引用終り)
308 名前:132人目の素数さん mailto:sage [2017/11/07(火) 21:34:58.07 ID:C8Zmg5mj.net] >>300 ああ、>>294 みたく、g(z) ≠ 0 を断っておく必要があるね
309 名前:132人目の素数さん mailto:sage [2017/11/07(火) 22:00:09.72 ID:1JSB0KNQ.net] 実数係数多項式環の除法の定理使えば一瞬だろ
310 名前:132人目の素数さん mailto:sage [2017/11/07(火) 22:38:24.16 ID:Dbgh/Bsh.net] a(b+c)2乗+b(c+a)2乗+c (a+b)2乗-4abc をaについて整理せよ。 解説読んでも理解できません。 お願いします。
311 名前:132人目の素数さん mailto:sage [2017/11/07(火) 23:17:30.24 ID:CXJzz6U5.net] >>303 整理せよの意味が分かってないのか、意味が分かってるけど整理する計算が分からないのか 参考書に計算過程書いてあるだろ それ丸写しして、どこがどう分かりませんくらい書けや
312 名前:132人目の素数さん mailto:sage [2017/11/07(火) 23:18:58.42 ID:ZVzkYw1Q.net] >>303 2乗の項を全部展開してからaについて整理すると (b+c)a^2+(b^2+2bc+c^2)a+bc^2+cb^2 となる。
313 名前:132人目の素数さん mailto:sage [2017/11/07(火) 23:19:07.44 ID:o8b3Czlh.net] >>304 わからないんですね
314 名前:132人目の素数さん mailto:sage [2017/11/07(火) 23:23:09.10 ID:CXJzz6U5.net] >>305 そいつの教育にならんのじゃ カスみたいなレスするヒマあったらそいつを追い込むレスの1つでも考えろ!
315 名前:132人目の素数さん [2017/11/07(火) 23:29:29.02 ID:F0P8QdZ3.net] >>273 >有界な実数集合の上限の存在がZornの補題から従う 切断で定義する実数では連続性に別に選択公理要らないよ
316 名前:132人目の素数さん [2017/11/07(火) 23:29:55.53 ID:F0P8QdZ3.net] >>306 もう止めなよ 可哀想な人
317 名前:132人目の素数さん mailto:sage [2017/11/07(火) 23:33:41.78 ID:6SAPjFtc.net] >>289 実係数多項式をR、複素係数多項式をCとすると、 R*R=R R*C=C C*R=C C*C=CまたはR のパターンしかない。 R*X=R なら、XはRで確定。
318 名前:132人目の素数さん [2017/11/07(火) 23:42:13.24 ID:KCPLSFoF.net] A(−3,1)を通り,傾き2の直線をLの方程式は y - 1 = 2(x + 3) が正解みたいなんですけど、y = 2x + 7 の可能性はどこで消えるんでしょうか?
319 名前:132人目の素数さん mailto:sage [2017/11/07(火) 23:42:42.43 ID:o8b3Czlh.net] どちらも同じですよね どっちも正解です
320 名前:132人目の素数さん [2017/11/07(火) 23:53:03.46 ID:KCPLSFoF.net] >>312 ありがとうございます。こういうケースに柔軟に対応できるよう慣れていこうと思います。
321 名前:132人目の素数さん [2017/11/07(火) 23:58:34.98 ID:F0P8QdZ3.net] >>310 うーん 弱いな
322 名前:132人目の素数さん mailto:sage [2017/11/08(水) 00:05:21.06 ID:scBnzmQg.net] >>307 ある無矛盾な公理系τの任意のモデルに対してある論理式φが常に真となるならば、τからφがLKにおいて証明可能となることを示せ、という問題がわかりません
323 名前:132人目の素数さん mailto:sage [2017/11/08(水) 00:13:02.05 ID:+FMC5OWu.net] 複素解析におけるポテンシャル論の有用性を教えて
324 名前:132人目の素数さん mailto:sage [2017/11/08(水) 00:15:54.66 ID:sc4+co92.net] >>313 @「y-1=2(x+3)」 意味を重視した式 ・x=-3、y=1で等式が成り立つ→点(-3,1)を通る ・2(x+○)→傾きが2 この意味を式中に埋め込んでいる 計算可能な部分が残っているから指示がない限り最終的な解答として不向き A「y=2x+7」 y=ax+b のように一次関数の基本形 一般的な解答としてはこちらが好まれる 答え方に迷ったらこちら B「2x-y+7=0」 直線の方程式の表示形式 多変数を扱う場合はこういう表記もあるけど、あまり気にしなくていい
325 名前:132人目の素数さん mailto:sage [2017/11/08(水) 00:17:54.64 ID:scBnzmQg.net] >>317 三段論法を用いる任意の数学の証明は、三段論法を用いない別証明を持つことを示せ、という問題がわかりません
326 名前:132人目の素数さん mailto:sage [2017/11/08(水) 00:34:18.22 ID:HCMYFA6+.net] 線形写像f:U→Vにおいてa∈Uに対して-f(a)=f(-a)ってどう証明すればいいんですかね f(a-b)=f(a)-f(b)と同じ方法で示そうとすると結局堂々巡りになってしまうのですが
327 名前:132人目の素数さん mailto:sage [2017/11/08(水) 00:36:10.82 ID:scBnzmQg.net] f(0)=0=f(a-a)=f(a)+f(-a) f(-a)=-f(a)
328 名前:132人目の素数さん [2017/11/08(水) 00:50:36.26 ID:KYal/F5W.net] >>318 下らん
329 名前:132人目の素数さん [2017/11/08(水) 00:51:06.36 ID:KYal/F5W.net] >>320 しょうも無
330 名前:132人目の素数さん [2017/11/08(水) 00:55:06.46 ID:rpzlc+ba.net] 今全財産が3000円しかありません 何とかして20万円まで増やしてiPhoneかギャラクシー買いたい その方法とは?
331 名前:132人目の素数さん [2017/11/08(水) 00:56:50.45 ID:NRmZ7qIa.net] 腎臓を売ります
332 名前:132人目の素数さん mailto:sage [2017/11/08(水) 00:58:32.75 ID:scBnzmQg.net] >>321 わからないんですね
333 名前:(笑) [] [ここ壊れてます]
334 名前:132人目の素数さん [2017/11/08(水) 00:58:46.52 ID:YGpWnrKo.net] 別宇宙・別世界・別次元・別階層にワープする技術を独力で構築したらノーベル賞9999不可説不可説転個貰えますか?
335 名前:132人目の素数さん [2017/11/08(水) 01:06:35.06 ID:YGpWnrKo.net] 新たな宇宙・世界・次元・階層を独力で構築したらノーベル賞9999不可説不可説転個貰えますか?
336 名前:132人目の素数さん mailto:sage [2017/11/08(水) 01:16:24.57 ID:mblwdtt/.net] >>318 〔ゲンツェンの基本定理〕(1935) シーケンスSが証明可能ならば、三段論法を用いないでもシーケンスSは証明可能である。 (大意) 第一階述語論理体系LKおよびLJにおいて、「シーケンスSを終式にもつ証明図」が与えられたとする。 この証明図から「同じシーケンスSを終式にもちかつ三段論法を含まない証明図」を構成する一連の手続きを、定義することができる。
337 名前:132人目の素数さん mailto:sage [2017/11/08(水) 01:27:02.20 ID:lphcYthw.net] (P(x),Q(x),R(x))がR[x]≦2の基底であるとするとき (P(x+1),Q(x+1),R(x+1))もR[x]≦2の基底であるか判定せよ また、そのとき(P(x),R(x),Q(x))から(P(x+1),Q(x+1),R(x+1))への変換行列はどうなるか求めよ 前スレでも書きましたが答えが頂けなかったのでお願いします
338 名前:132人目の素数さん mailto:sage [2017/11/08(水) 01:38:09.92 ID:mblwdtt/.net] >>328 ぢゃあ、後者の証明図に現れる論理式はどれも 終式Sに現れるいずれかの論理式の一部(部分論理式)になるな…
339 名前:132人目の素数さん mailto:sage [2017/11/08(水) 01:40:21.12 ID:scBnzmQg.net] どうせどっかのpdfからコピってきたんでしょうね
340 名前:132人目の素数さん mailto:sage [2017/11/08(水) 02:08:43.13 ID:mblwdtt/.net] そうだったのか。
341 名前:132人目の素数さん mailto:sage [2017/11/08(水) 02:34:36.24 ID:w+IqqMhv.net] >>329 > (P(x),Q(x),R(x))がR[x]≦2の基底であるとするとき まず、「R[x]≦2」の書きっぷりが、なんじゃこいつ、ということになったんだろうな。 「R[x]」は多項式環を表すのが普通の意味なので、 多項式環が2以下って何?ということになっちゃうね。 残念 (エスパーの暗闘は密かに繰り広げられているのかな?)
342 名前:132人目の素数さん mailto:sage [2017/11/08(水) 02:47:16.49 ID:6InMXqC3.net] >>329 次数2以下ってことなら適当に文字使って表して変換行列が正則かどうか見ればよくね
343 名前:132人目の素数さん [2017/11/08(水) 02:58:29.03 ID:YGpWnrKo.net] 大日如来とアメリカ合衆国大統領はどっちの方が偉いですか?
344 名前:132人目の素数さん [2017/11/08(水) 03:37:05.46 ID:YGpWnrKo.net] 全宇宙全次元全世界全階層になりたいのですが、どうすればなれますか?
345 名前:¥ mailto:sage [2017/11/08(水) 08:16:22.07 ID:r2EqUl6O.net] ¥
346 名前:¥ mailto:sage [2017/11/08(水) 08:16:39.84 ID:r2EqUl6O.net] ¥
347 名前:¥ mailto:sage [2017/11/08(水) 08:16:57.20 ID:r2EqUl6O.net] ¥
348 名前:¥ mailto:sage [2017/11/08(水) 08:17:13.31 ID:r2EqUl6O.net] ¥
349 名前:¥ mailto:sage [2017/11/08(水) 08:17:29.08 ID:r2EqUl6O.net] ¥
350 名前:¥ mailto:sage [2017/11/08(水) 08:17:44.93 ID:r2EqUl6O.net] ¥
351 名前:¥ mailto:sage [2017/11/08(水) 08:18:01.28 ID:r2EqUl6O.net] ¥
352 名前:¥ mailto:sage [2017/11/08(水) 08:18:17.50 ID:r2EqUl6O.net] ¥
353 名前:¥ mailto:sage [2017/11/08(水) 08:18:33.75 ID:r2EqUl6O.net] ¥
354 名前:¥ mailto:sage [2017/11/08(水) 08:18:50.95 ID:r2EqUl6O.net] ¥
355 名前:132人目の素数さん mailto:sage [2017/11/08(水) 09:09:25.27 ID:lphcYthw.net] >>333 R[x]_(≦2)と書けば分かりやすかったですかね [x]の添字に≦2が付いてる意味で書いてました 意味は次数が最大2の多項式環らしいです(自分もこの問題で初めてこの記号を見ました)
356 名前:132人目の素数さん mailto:sage [2017/11/08(水) 10:01:25.78 ID:w+IqqMhv.net] >>347 次数が2次以下の多項式の全体がなす加群、ね。 多項式「環」にはならない。 解答は>>334 さんが書いている
357 名前:132人目の素数さん [2017/11/08(水) 10:02:54.45 ID:KYal/F5W.net] >>329 T:R[x]→R[x]:f(x)→f(x+1)は線形同型(線形性は明かで逆写像の存在も明か) よって P,Q,Rが1次独立→TP,TQ,TRは1次独立 P,Q,Rが生成→TP,TQ,TRが生成 すなわち P,Q,Rが基底→TP,TQ,TRが基底 (TP,TQ,TR)=(P,Q,R)A (TP',TQ',TR')=(P',Q',R')A (TP'',TQ'',TR'')=(P'',Q'',R'')A A=(PQR/P'Q'R'/P''/Q''/R'')^-1(TPTQTR/TP'TQ'TR'/TP''TQ''TR'')
358 名前:132人目の素数さん [2017/11/08(水) 10:12:14.39 ID:KYal/F5W.net] 注:n個のn-1階微分可能関数が1次独立ならロンスキアンは0にならないので
359 名前:正則 [] [ここ壊れてます]
360 名前:132人目の素数さん mailto:sage [2017/11/08(水) 10:24:12.11 ID:w+IqqMhv.net] >>349 > >>329 > T:R[x]→R[x]:f(x)→f(x+1)は線形同型(線形性は明かで逆写像の存在も明か) え?
361 名前:132人目の素数さん mailto:sage [2017/11/08(水) 10:31:41.43 ID:w+IqqMhv.net] 失礼
362 名前:132人目の素数さん mailto:sage [2017/11/08(水) 12:04:38.21 ID:9GhzdmI+.net] >>289 >f(z) を実係数多項式とし、複素係数の多項式として >f(z) = g(z)*h(z) >と分解されるとする。 >このとき、g(z) が実係数多項式ならば h(z) も実係数多項式である もう話終わったかもしれんけど... f, g, h それぞれの係数を a[0...n+m] (∈ R), b[0...n] (∈ R), c[0...m] (∈ C) で表す。 h(z) に非実数係数項が存在すると仮定し、その最大次数 を k (≦m)とする。 a[n+k] = Σ{i+j=n+k} b[i]*c[j] = b[n]*c[k] + b[n-1] * c[k+1] + .... 仮定より和の初項は非実数、その他の項は実数である。これは和の a[n+k] が実数であることに矛盾する。 背理法により h(z) に非実数係数項 は存在しない。 よって h(z) は実係数多項式である。
363 名前:132人目の素数さん [2017/11/08(水) 12:09:34.24 ID:KYal/F5W.net] >>330 「三段論法」を凄く狭く定義してるだけよ
364 名前:132人目の素数さん mailto:sage [2017/11/08(水) 12:32:55.70 ID:YLC+7k7Q.net] >>354 >>330 が間違えだとでも言いたそうですね(笑)
365 名前:132人目の素数さん mailto:sage [2017/11/08(水) 12:55:48.85 ID:5ejE5iPD.net] カット以外の三段論法てあるんか?
366 名前:132人目の素数さん mailto:sage [2017/11/08(水) 12:58:45.50 ID:EG9EsJzj.net] 高校数学です lim[n→∞]((3^n)/n!)の極限を求めよ やり方がわからないので教えて下さい
367 名前:132人目の素数さん mailto:sage [2017/11/08(水) 13:19:37.87 ID:+pMn8nmi.net] >>357 an=(3^n)/n!とおく a(n+1)=(3/(n+1))an より n+1>3、つまりn≧4に対してa(n+1)<(3/4)an ここから0<an<(3/4)^(n-3)a3 n→∞で、はさみうちの原理からan→0
368 名前:132人目の素数さん [2017/11/08(水) 13:25:00.39 ID:NUCx5GNv.net] 距離空間 X において A, B を A ∩ B = 空集合である2つの閉集合とする。 A ⊂ U, B ⊂ V, U ∩ V = 空集合を満たす X の開集合 U, V が存在することを証明せよ。 証明: X 上で関数 φ(x) = d(x, A) - d(x, B) を考えれば、 φ : X → R は連続である。よって、 U = {x | φ(x) < 0} V = {x | φ(x) > 0} は開集合で、 A ⊂ U, B ⊂ V, U ∩ V = 空集合となる。
369 名前:132人目の素数さん [2017/11/08(水) 13:25:38.89 ID:NUCx5GNv.net] A ⊂ U, B ⊂ V となるのはなぜでしょうか? 証明をお願いします。
370 名前:132人目の素数さん [2017/11/08(水) 13:33:59.67 ID:NUCx5GNv.net] あ、分かりました。 d(a, B) = 0 となるような a ∈ A が存在したとすると、 a ∈ Bの触点 = B となり、 A ∩ B = 空集合に矛盾する。
371 名前:132人目の素数さん [2017/11/08(水) 14:44:59.81 ID:NUCx5GNv.net] 以下の問題に対する解答はあっていますか? 完備な距離空間に一様同相な距離空間は完備であることを証明せよ。 証明: X を距離空間 Y を完備な距離空間 f を X から Y への一様同相写像 とする。 f は一様連続だから、 ε を任意の正の実数とすると、 d(x, y) < δ ⇒ d(f(x), f(y)) < ε となるような正の実数 δ が存在する。 (x_n) を X のコーシー列とすると、 m, n ≧ N ⇒ d(x_m, x_n) < δ となるような自然数 N が存在する。 m, n ≧ N ⇒ d(f(x_m), f(x_n)) < εが成り立つ。 よって、 (f(x_n)) は Y のコーシー列である。 Y は完備であるから、 lim f(x_n) = b となるような b ∈ Y が存在する。 a := f^(-1)(b) とおく。 f^(-1) は連続写像だから、 lim x_n = lim f^(-1)(f(x_n)) = f^(-1)(b) = a である。 これは、 (x_n) が X の中で収束することを示す。 よって、 X は完備である。
372 名前:132人目の素数さん mailto:sage [2017/11/08(水) 15:53:40.67 ID:9qE2wZNV.net] 微分関係でわからない問題が3つありますのでどうか宜しくお願い致します ・関数 f:I→R を I 上で微分可能かつ単調非減少とするとき、I 上で f'(x)≧0 が成り立つことを示せ また、f が I 上で狭義単調増加ならば I 上で f'(x)>0 が成り立つか調べよ ・f:R→R を微分可能な関数とする 「f(x_0)=0 かつ f'(x)>f(x) (∀x∈R)
373 名前:」が成立してるならば ∀x>x_0 に対して f(x)>0 を示せ ・C^n級関数 f:(a,b)→R が α∈(a,b) とk=1,2,...,n-1 に対して f^(k)(α)=0 を満たし、f^(n)(α)≠0とするとき n が偶数ならば f は α で極値をとり、n が奇数ならば f は α で極値をとらないことを証明せよ [] [ここ壊れてます]
374 名前:132人目の素数さん mailto:sage [2017/11/08(水) 16:09:53.33 ID:ZIwTiOVm.net] こちらこそ宜しくお願い致します
375 名前:132人目の素数さん mailto:sage [2017/11/08(水) 16:23:04.36 ID:y1FBE2pI.net] 信号処理とかもここでいいかな? y(n)=x(n)+1/2y(n-1)-1/2y(n-2) の式をz平面で表したいんだけど、どうしたらいい?
376 名前:132人目の素数さん mailto:sage [2017/11/08(水) 17:59:11.83 ID:iXb4Tr2q.net] >>359 意味不明
377 名前:132人目の素数さん [2017/11/08(水) 18:05:38.42 ID:NUCx5GNv.net] >>353 直接的で分かりやすい証明ですね。 ありがとうございました。
378 名前:132人目の素数さん [2017/11/08(水) 18:40:02.50 ID:NUCx5GNv.net] 以下は R の任意の区間が連結であることの証明です。 「P は I の開集合、したがって R のある開集合と I との共通部分であるから」 は 「P は I の開集合であるから」 ではなぜ駄目なのでしょうか? R の任意の区間が連結であることを示す。 I を R の任意の区間とする。 I が2つの空でない I の開集合 P, Q の直和に分割されたと仮定して、矛盾を導こう。 P, Q からそれぞれ1つの点 a, b をとれば、 a ≠ b であるから a < b または a > b .どちらでも同じことであるから a < b と仮定する。 I は区間であるから、もちろん [a,b] ⊂ I である。いま M = [a, b] ∩ P とおく。そのとき a ∈ M であるから M ≠ 空集合 で、 b は M の1つの上界であるから、 sup M = c が存在して a ≦ c ≦ b である。 c ∈ I であるから c ∈ P または c ∈ Q .もし c ∈ P ならば、 c < b で、 P は I の開集合、したがって R の ある開集合と I との共通部分であるから、 ε > 0 を十分小さくとれば、 c < c + ε < b, c + ε ∈ P となる。 よって c + ε ∈ Mとなるが、これは c が M の上界であることに反する。
379 名前:132人目の素数さん [2017/11/08(水) 18:40:44.90 ID:NUCx5GNv.net] 訂正します: 以下は R の任意の区間が連結であることの証明です。 「P は I の開集合、したがって R のある開集合と I との共通部分であるから」 を 「P は I の開集合であるから」 としては、なぜ駄目なのでしょうか? R の任意の区間が連結であることを示す。 I を R の任意の区間とする。 I が2つの空でない I の開集合 P, Q の直和に分割されたと仮定して、矛盾を導こう。 P, Q からそれぞれ1つの点 a, b をとれば、 a ≠ b であるから a < b または a > b .どちらでも同じことであるから a < b と仮定する。 I は区間であるから、もちろん [a,b] ⊂ I である。いま M = [a, b] ∩ P とおく。そのとき a ∈ M であるから M ≠ 空集合 で、 b は M の1つの上界であるから、 sup M = c が存在して a ≦ c ≦ b である。 c ∈ I であるから c ∈ P または c ∈ Q .もし c ∈ P ならば、 c < b で、 P は I の開集合、したがって R の ある開集合と I との共通部分であるから、 ε > 0 を十分小さくとれば、 c < c + ε < b, c + ε ∈ P となる。 よって c + ε ∈ Mとなるが、これは c が M の上界であることに反する。
380 名前:132人目の素数さん [2017/11/08(水) 18:42:20.93 ID:NUCx5GNv.net] >>369 は松坂和夫著『解析入門3』に載っている証明です。
381 名前:132人目の素数さん mailto:ddd [2017/11/08(水) 18:51:05.66 ID:yVMmY+VD.net] >>365 Y=X+(1/2)Z^(-1) Y-1/2 Z^(-2)Y H(z)=Y/X= 2 Z^2/(1-Z+2 Z^2)=1+(Z-1)/(1-Z+2 +Z^2) この伝達関数をz平面(z=Exp(j w ))で書けばいいんじゃないの? ちょっと振動するのかな?
382 名前:132人目の素数さん [2017/11/08(水) 18:53:08.65 ID:YGpWnrKo.net] 仏教学と数学はどっちの方が真理に対する信憑性が高いですか?
383 名前:132人目の素数さん mailto:sage [2017/11/08(水) 19:22:43.10 ID:Ts1/tdBu.net] >>163 Γ={P,¬P}とするとΓは充足不可能ですが¬φがトートロジーとなるφ∈Γはとれないのでこの証明は間違ってませんか?
384 名前:132人目の素数さん mailto:sage [2017/11/08(水) 19:31:41.71 ID:rJQRhdQg.net] >>373 コンパクト性定理より、ある論理式の集まりφ1〜φnが存在して、¬φ1∨¬φ2∨...∨¬φnがトートロジーとなる、の間違えでした
385 名前:132人目の素数さん mailto:sage [2017/11/08(水) 19:54:24.73 ID:YGpWnrKo.net] 全=外側の絶対王者 無=内側の絶対王者 ですか?
386 名前:132人目の素数さん mailto:sage [2017/11/08(水) 19:56:17.89 ID:rJQRhdQg.net] >>375 絶対王者=神です
387 名前:132人目の素数さん mailto:sage [2017/11/08(水) 20:02:53.74 ID:9qE2wZNV.net] >>363 全部でなくともいいのでお願いします…
388 名前:132人目の素数さん mailto:sage [2017/11/08(水) 20:05:31.11 ID:Sb5mRlwP.net] >>369 相対位相で開は全体では開かわからんやろ ケチ付けるのが趣味のくせにレベル低いな
389 名前:132人目の素数さん mailto:sage [2017/11/08(水) 20:07:09.82 ID:YGpWnrKo.net] >>376 神は全に含まれるから、全>神 じゃないでしょうか?
390 名前:132人目の素数さん mailto:sage [2017/11/08(水) 20:18:04.53 ID:9GhzdmI+.net] >>369 P = P' ∩ I ( P' はRにおける開集合) と書ける c ∈ P, P⊂ P' なので、 (c-ε1, c+ε1) ⊂ P' となる ε1 が存在する また c < c+ε2 < b となる任意の ε2 について、 c+ε2 ∈ I ε = min(ε1, ε2) とおけば、c+ε ∈ P' ∩ I = P (分からなかったら部分空間の位相について読み直した方がいいかも)
391 名前:132人目の素数さん mailto:sage [2017/11/08(水) 20:31:31.72 ID:OXeR7kbv.net] >>363 >・関数 f:I→R を I 上で微分可能かつ単調非減少とするとき、I 上で f'(x)≧0 が成り立つことを示せ ∃a∈I s.t. 0>f'(a)=-εとします ε/2に対して∃δ>0 s.t.|h|<δ→ |(f(a+h)-f(a))/h+ε|<ε/2 -3ε/2<(f(a+h)-f(a))/h<-ε/2<0 h>0のときf(a+h)<f(a)となっており、これはfが単調減少ではないことに矛盾します >また、f が I 上で狭義単調増加ならば I 上で f'(x)>0 が成り立つか調べよ 反例 f(x)=x^3 y^3-x^3=(y-x)(y^2+xy+x^2)>0(y>x)ですが、f'(0)=0です
392 名前:132人目の素数さん mailto:sage [2017/11/08(水) 20:43:07.08 ID:M1XSfK8i.net] >>363 2問目は対偶とってロルの定理で証明できると思う
393 名前:132人目の素数さん mailto:sage [2017/11/08(水) 21:07:29.68 ID:5MwXlS3i.net] >>363 3問目はx=αで第(n-2)次テイラー展開して f(x)=剰余項f^(n-2)(α+θ(x-α))(x-α)^(n-2)/(n-2)! f'(α)=0と、nの偶奇によるf''(x)の0か否かの判定で極値の有無を言えると思う
394 名前:132人目の素数さん mailto:sage [2017/11/08(水) 21:15:29.74 ID:9qE2wZNV.net] >>381 >>382 >>383 ありがとうございます 2問目3問目も解への道筋が分かったので有難いです
395 名前:¥ mailto:sage [2017/11/08(水) 22:31:50.19 ID:r2EqUl6O.net] ¥
396 名前:¥ mailto:sage [2017/11/08(水) 22:32:07.17 ID:r2EqUl6O.net] ¥
397 名前:¥ mailto:sage [2017/11/08(水) 22:32:25.17 ID:r2EqUl6O.net] ¥
398 名前:132人目の素数さん mailto:sage [2017/11/08(水) 22:32:30.26 ID:nz1pZ2bE.net] Σ [k=1,n] nCk をnの整式で表すことはできますか?
399 名前:¥ mailto:sage [2017/11/08(水) 22:32:42.10 ID:r2EqUl6O.net] ¥
400 名前:¥ mailto:sage [2017/11/08(水) 22:32:59.45 ID:r2EqUl6O.net] ¥
401 名前:¥ mailto:sage [2017/11/08(水) 22:33:15.82 ID:r2EqUl6O.net] ¥
402 名前:¥ mailto:sage [2017/11/08(水) 22:33:36.06 ID:r2EqUl6O.net] ¥
403 名前:¥ mailto:sage [2017/11/08(水) 22:33:56.58 ID:r2EqUl6O.net] ¥
404 名前:132人目の素数さん [2017/11/08(水) 22:34:02.38 ID:KYal/F5W.net] >>355 ?
405 名前:¥ mailto:sage [2017/11/08(水) 22:34:15.60 ID:r2EqUl6O.net] ¥
406 名前:¥ mailto:sage [2017/11/08(水) 22:34:41.16 ID:r2EqUl6O.net] ¥
407 名前:¥ mailto:sage [2017/11/08(水) 22:35:03.20 ID:r2EqUl6O.net] ¥
408 名前:¥ mailto:sage [2017/11/08(水) 22:35:26.00 ID:r2EqUl6O.net] ¥
409 名前:¥ mailto:sage [2017/11/08(水) 22:35:45.93 ID:r2EqUl6O.net] ¥
410 名前:¥ mailto:sage [2017/11/08(水) 22:36:03.14 ID:r2EqUl6O.net] ¥
411 名前:¥ mailto:sage [2017/11/08(水) 22:36:23.88 ID:r2EqUl6O.net] ¥
412 名前:¥ mailto:sage [2017/11/08(水) 22:36:45.65 ID:r2EqUl6O.net] ¥
413 名前:¥ mailto:sage [2017/11/08(水) 22:37:07.90 ID:r2EqUl6O.net] ¥
414 名前:¥ mailto:sage [2017/11/08(水) 22:37:27.50 ID:r2EqUl6O.net] ¥
415 名前:132人目の素数さん mailto:sage [2017/11/08(水) 23:37:36.16 ID:mblwdtt/.net] >>388 正式には 2^n - 1 かなあ...
416 名前:132人目の素数さん [2017/11/08(水) 23:39:38.67 ID:KYal/F5W.net] >>388 無理 1=nC0足すと2項定理で2^n
417 名前:132人目の素数さん mailto:sage [2017/11/08(水) 23:41:36.29 ID:scBnzmQg.net] ↑これが数学板の実力です↑ 専門板なのに異常にレベルが低い せいぜい数学の少しできる高校生レベル
418 名前:132人目の素数さん [2017/11/09(木) 00:08:14.91 ID:1DPIZrTv.net] >>317 ありがとうございます。勉強になります。
419 名前:132人目の素数さん [2017/11/09(木) 00:24:39.63 ID:KmBKGqZp.net] >>407 もう止めたら?
420 名前:132人目の素数さん mailto:sage [2017/11/09(木) 00:42:34.89 ID:uCu6B49W.net] 整式の定義が分かっていない人に指摘をしただけですよ
421 名前:132人目の素数さん mailto:sage [2017/11/09(木) 01:34:13.57 ID:qWyw6S6d.net] 数学と法学ってどっちの方がムズイ?
422 名前:132人目の素数さん mailto:sage [2017/11/09(木) 01:57:58.63 ID:qWyw6S6d.net] 「無」ってほんと、考えれば考えるほどつくづく「究極」だなぁと思うよな。
423 名前:132人目の素数さん mailto:sage [2017/11/09(木) 02:08:11.39 ID:DmTD4nE0.net] 原点O. A(a.2) B.(a.6). C(8.8)の4点がある。 OA+ABBCが最短になる時の直線BCを表す式を求めなさい。 だれか教えてください。
424 名前:132人目の素数さん mailto:sage [2017/11/09(木) 02:09:41.81 ID:DmTD4nE0.net] OA+AB+BCの書き間違えです。 すいません
425 名前:132人目の素数さん mailto:sage [2017/11/09(木) 03:25:16.37 ID:4NtdL4/9.net] >>413 A(a,2),B(a,6),C(8,8) まずAB=4 次に点OをP(0,4)まで平行
426 名前:レ動するとOA=PB よって、OA+AB+BC=PB+4+BC=4+(PB+BC) すなわち折れ線PACの長さが最小になればいいので、3点P,B,C,が一直線上にあればよく、以下略 折れ線PBCを作るためにOを平行移動するのがミソ この手の最小値問題は必ず折れ線を作らせるので、折れ線を作るための工夫を考えれば出来る。大体平行移動。 [] [ここ壊れてます]
427 名前:132人目の素数さん mailto:sage [2017/11/09(木) 03:27:00.12 ID:4NtdL4/9.net] >>415 ☓折れ線PACの長さが最小 ○折れ線PBCの長さが最小
428 名前:132人目の素数さん [2017/11/09(木) 07:02:25.25 ID:KmBKGqZp.net] >>410 ぷ
429 名前:132人目の素数さん mailto:sage [2017/11/09(木) 07:03:54.18 ID:uCu6B49W.net] ↑nの整式といったときに定数が入ったら整式ではないと思ってるような人なんですね
430 名前:132人目の素数さん [2017/11/09(木) 07:04:08.05 ID:KmBKGqZp.net] アア分かった 2^nが整式じゃないって当たり前のことを指摘したのか ぷ
431 名前:132人目の素数さん mailto:sage [2017/11/09(木) 07:05:01.81 ID:uCu6B49W.net] >>419 あなたを殺すにはどうすれば良いでしょうか?
432 名前:132人目の素数さん [2017/11/09(木) 07:05:06.81 ID:KmBKGqZp.net] >>418 アラ違うのかな? なにそれ?
433 名前:132人目の素数さん mailto:sage [2017/11/09(木) 07:05:34.14 ID:uCu6B49W.net] >>421 あなたを殺す方法を教えてください
434 名前:132人目の素数さん mailto:sage [2017/11/09(木) 07:06:33.88 ID:uCu6B49W.net] ID:KmBKGqZpさんを殺害する方法がわかりません よろしくお願いします
435 名前:132人目の素数さん [2017/11/09(木) 07:09:13.47 ID:KmBKGqZp.net] ID:uCu6B49W 「nの整式といったときに定数が入ったら整式ではないと思ってるような人」 って 2^nの何を「定数と思っている」と指摘したの? 2?n? なにそれ?
436 名前:132人目の素数さん mailto:sage [2017/11/09(木) 07:09:37.77 ID:uCu6B49W.net] ここの回答者って、自分がどうやったら死ぬのかもわからないような、低レベルな人たちだったんですね
437 名前:132人目の素数さん [2017/11/09(木) 07:29:59.72 ID:Ep8PEh0S.net] 組合せの勉強してますが、 ある参考書にnC1=1と書いてましたが 間違ってないですか? 6C1=6ですよね?
438 名前:132人目の素数さん [2017/11/09(木) 07:54:41.68 ID:KmBKGqZp.net] >>425 ぷふぅ
439 名前:132人目の素数さん mailto:sage [2017/11/09(木) 07:56:29.97 ID:DmTD4nE0.net] >>415 ありがとうございます。 あともう1つ教えてください A(a.2)B.(a.6) これ4と割り出す式をお願いします。 息子のわからない問題で自分の頭では全くわかりません
440 名前:132人目の素数さん mailto:sage [2017/11/09(木) 08:09:17.55 ID:rjKfkEZ/.net] >>428 両方ともx座標がaだから その2点を結ぶ線分ABはy軸と平行→AB=6-2=4 機械的にやるなら AB^2=(a-a)^2+(6-2)^2=4^2 → AB=4
441 名前:132人目の素数さん mailto:sage [2017/11/09(木) 12:04:28.06 ID:Aitxjc9D.net] なぜ親が解こうとするのか 学校の先生に聞けで一蹴できる
442 名前:132人目の素数さん mailto:sage [2017/11/09(木) 13:23:05.58 ID:FBxIhJ9Y.net] そう書く事を気に入ってんだろ
443 名前:¥ mailto:sage [2017/11/09(木) 14:05:55.50 ID:9ddzKhaE.net] ¥
444 名前:¥ mailto:sage [2017/11/09(木) 14:06:12.53 ID:9ddzKhaE.net] ¥
445 名前:¥ mailto:sage [2017/11/09(木) 14:06:30.50 ID:9ddzKhaE.net] ¥
446 名前:¥ mailto:sage [2017/11/09(木) 14:06:49.08 ID:9ddzKhaE.net] ¥
447 名前:¥ mailto:sage [2017/11/09(木) 14:07:07.13 ID:9ddzKhaE.net] ¥
448 名前:¥ mailto:sage [2017/11/09(木) 14:07:26.77 ID:9ddzKhaE.net] ¥
449 名前:¥ mailto:sage [2017/11/09(木) 14:07:44.50 ID:9ddzKhaE.net] ¥
450 名前:¥ mailto:sage [2017/11/09(木) 14:08:08.43 ID:9ddzKhaE.net] ¥
451 名前:¥ mailto:sage [2017/11/09(木) 14:08:27.70 ID:9ddzKhaE.net] ¥
452 名前:¥ mailto:sage [2017/11/09(木) 14:08:45.93 ID:9ddzKhaE.net] ¥
453 名前:132人目の素数さん [2017/11/09(木) 19:33:11.00 ID:JfxwYVU8.net] https://i.imgur.com/Weg4eWj.jpg これの設題2の2番がさっぱりです 通信制の大学なので教授に聞くのも不可能なのですが、 わかる方、丁寧に教えていただけると助かります
454 名前:132人目の素数さん [2017/11/09(木) 19:37:27.58 ID:8rY1AWyi.net] 1/319のくじを3連続で引く確率を教えてください
455 名前:132人目の素数さん mailto:sage [2017/11/09(木) 19:51:43.32 ID:72WoZiVe.net] ベクトル解析でつまづいてます スカラーポテンシャルやベクトルポテンシャルが 何を意味しているのかさっぱりわからないのですが もしかしてこれは電磁気学をやらないとわからないという類のものでしょうか?
456 名前:132人目の素数さん [2017/11/09(木) 20:13:34.51 ID:ExFJ6f+/.net] >>380 丁寧にありがとうございました。 >>378 「P は I の開集合であるから」だけでも十分だと思います。 R のある開集合などという分かりにくいものを持ち出さなくてもOKです。 c ∈ P で P は I の開集合であるから、 (c - ε', c + ε') ∩ I ⊂ P となるような正の実数 ε' が存在する。 b ∈ P でないから、 ε' ≦ b - c でなければならない。 ε を 0 < ε < ε' を満たす実数とすると、 c < c + ε < c + ε' ≦ b であるから、 c + ε ∈ P である。
457 名前:132人目の素数さん [2017/11/09(木) 20:15:52.03 ID:ExFJ6f+/.net] >>378 I に端があり、その端を P が含まなければ明らかに P は R で開集合ではないでしょうか?
458 名前:132人目の素数さん [2017/11/09(木) 20:17:53.60 ID:ExFJ6f+/.net] R のある開集合など持ち出さなくても自明ですよね。 逆に、なぜ R のある開集合が登場するのかと疑問に思ってしまいますよね。
459 名前:132人目の素数さん mailto:sage [2017/11/09(木) 20:19:23.98 ID:LCf48bd/.net] >>444 そうです あくまでも「意味」を求めるなら、具体的な使われ方を見るのが一番早いでしょうね 数学的には定義が全てですから、意味も何もないわけです ただ、任意のベクトルVはV=∇φ+∇×Aと書き下せるなど、いろいろ興味深い性質を持っていることは確かですね
460 名前:132人目の素数さん mailto:sage [2017/11/09(木) 20:36:21.89 ID:Pdn9wNtv.net] >>447 アンタの存在が一番疑問
461 名前:132人目の素数さん mailto:sage [2017/11/09(木) 21:31:33.42 ID:AKM5yVrh.net] >>448 有難うございます grad・div・rotまでは大丈夫でだったんですが それらを組み合わせたdiv rot f =0 や rot grad φ = 0 辺りから理解が怪しくなりましたw アドバイス頂いたように意味については電磁気学で学ぶとして 今は計算や使いかたを学習すると割り切ってやっていきます
462 名前:132人目の素数さん mailto:ddd [2017/11/09(木) 22:25:57.10 ID:z5IbW63y.net] >>442 y2-y1=-2(x2-x1) (y1+y2)/2=(1/2)(x1+x2)/2 をといて {x2,y2}={{3,4},{4,-3}}/5 {x1,y1} から A={{3,4},{4,-3}}/5 B={{cos(Pi/4),-sin(Pi/4)},{sin(pi/4),cos(pi/4)}}={{1,-1},{1,1}}/Sqrt[2] f(g(X))<=> Ag == B so g=A^(-1)B ={{-1,7},{7,1}}Sqrt[2]/10
463 名前:132人目の素数さん mailto:sss [2017/11/09(木) 22:32:28.71 ID:z5IbW63y.net] g=A^(-1)B ={{7,1},{1,-7}}Sqrt[2]/10 に訂正してください。 >>451 はBA^(-1)でした。 失礼しました。
464 名前:132人目の素数さん mailto:sage [2017/11/09(木) 22:42:13.88 ID:HOAkGA6G.net] R上の実数値連続関数全体のなす集合の濃度が|R|であることを示せ 任意の有限集合である全順序集合は整列集合であることを示せ R-{0,1}においてx〜1-x, x〜1/x で生成される同値関係を定めるときのxの同値類を書け 複数問で申し訳ありませんが示し方がわからないのでお願いします
465 名前:¥ mailto:sage [2017/11/09(木) 23:24:19.49 ID:9ddzKhaE.net] ¥
466 名前:¥ mailto:sage [2017/11/09(木) 23:24:36.52 ID:9ddzKhaE.net] ¥
467 名前:¥ mailto:sage [2017/11/09(木) 23:25:00.66 ID:9ddzKhaE.net] ¥
468 名前:¥ mailto:sage [2017/11/09(木) 23:25:19.76 ID:9ddzKhaE.net] ¥
469 名前:¥ mailto:sage [2017/11/09(木) 23:25:38.29 ID:9ddzKhaE.net] ¥
470 名前:¥ mailto:sage [2017/11/09(木) 23:25:57.17 ID:9ddzKhaE.net] ¥
471 名前:¥ mailto:sage [2017/11/09(木) 23:26:16.47 ID:9ddzKhaE.net] ¥
472 名前:¥ mailto:sage [2017/11/09(木) 23:26:35.09 ID:9ddzKhaE.net] ¥
473 名前:¥ mailto:sage [2017/11/09(木) 23:26:52.49 ID:9ddzKhaE.net] ¥
474 名前:¥ mailto:sage [2017/11/09(木) 23:27:13.22 ID:9ddzKhaE.net] ¥
475 名前:132人目の素数さん mailto:sage [2017/11/10(金) 00:14:56.06 ID:pyRwbQRE.net] この荒らしっていつから、なんでいるんだ?
476 名前:132人目の素数さん mailto:sage [2017/11/10(金) 00:19:22.86 ID:NaWRmLAa.net] 10年前に痴漢で逮捕された元筑波大学の准教授さんです
477 名前:132人目の素数さん mailto:sage [2017/11/10(金) 00:20:38.28 ID:s39Cvbg6.net] 数学板を潰すことに生涯を懸けている人らしい。
478 名前:132人目の素数さん mailto:sage [2017/11/10(金) 00:49:06.23 ID:svfXj2o0.net] ゴールドバッハ予想(の類似)って有理整数環以外でも成り立つの?
479 名前:132人目の素数さん [2017/11/10(金) 01:59:35.52 ID:X/75Up1v.net] 全、無、空 これらを格付けするとどうなりますか?
480 名前:132人目の素数さん mailto:sage [2017/11/10(金) 02:00:48.06 ID:NaWRmLAa.net] 神=神=神ですね
481 名前:132人目の素数さん mailto:sage [2017/11/10(金) 02:12:05.49 ID:ISjW5goZ.net] >>468 無=日 よって旅客機会社となる
482 名前:132人目の素数さん [2017/11/10(金) 02:18:48.88 ID:X/75Up1v.net] >>469-470 真面目に教えてください。お願いします。
483 名前:132人目の素数さん mailto:sage [2017/11/10(金) 03:55:19.48 ID:zIX+6Ycy.net] >>464 平安時代からいるよ。 「吹くからに 秋の草木のしをるれば むべ山風を嵐といふらむ」 文屋康秀 「古今集」 秋下249 「小倉百人一首」 22
484 名前:¥ mailto:sage [2017/11/10(金) 04:46:54.94 ID:f5tI1p7u.net] ¥
485 名前:¥ mailto:sage [2017/11/10(金) 04:47:12.61 ID:f5tI1p7u.net] ¥
486 名前:¥ mailto:sage [2017/11/10(金) 04:47:33.18 ID:f5tI1p7u.net] ¥
487 名前:¥ mailto:sage [2017/11/10(金) 04:47:51.30 ID:f5tI1p7u.net] ¥
488 名前:¥ mailto:sage [2017/11/10(金) 04:48:09.43 ID:f5tI1p7u.net] ¥
489 名前:¥ mailto:sage [2017/11/10(金) 04:48:29.59 ID:f5tI1p7u.net] ¥
490 名前:¥ mailto:sage [2017/11/10(金) 04:48:49.57 ID:f5tI1p7u.net] ¥
491 名前:¥ mailto:sage [2017/11/10(金) 04:49:09.59 ID:f5tI1p7u.net] ¥
492 名前:¥ mailto:sage [2017/11/10(金) 04:49:29.76 ID:f5tI1p7u.net] ¥
493 名前:¥ mailto:sage [2017/11/10(金) 04:49:50.31 ID:f5tI1p7u.net] ¥
494 名前:132人目の素数さん mailto:sage [2017/11/10(金) 08:58:46.91 ID:9dR83lHU.net] (2)の答え50cm^2になるみたいですけど、どうやって誘導すればいいですか?ご教示いただけますと幸いです。 https://i.imgur.com/twt5ZXh.jpg
495 名前:132人目の素数さん mailto:sage [2017/11/10(金) 09:18:55.19 ID:+PZdZIEk.net] 解き方がわからないって素直に言えばいいのに △DEF:△BCF=1:4 △DEFの高さ5/2(cm) 全体の高さ25/2(cm) 面積50(cm^2)
496 名前:チンぽろり [2017/11/10(金) 09:35:32.65 ID:2FWaGhK9.net] 1と0.9999999永遠 って 偉い人は同じだって言うらしい しらべてみ そもそも分数と整数じゃ 情報が違う 分数は3つの情報があって整数は1つしかない 5/5なら 全部で5で そのうち5あって 0足りない 整数は どのくらいあるのかという情報だけ 無理に入れようとするから 足りない1を永遠に深く探しても見つからない というエラーが起こる そもそも分数を整数にするのはへん 科学の理屈はそんなもの 革命家 チンぽろり ◆pKQFD7Nv721S
497 名前:132人目の素数さん mailto:sage [2017/11/10(金) 09:38:18.84 ID:UeRbzcUC.net] 文字列と数は違います 文字列は単に文字並べただけですけど、数には複雑な計算規則の構造が入っています 合理的な規則を維持しようと思えば、その両者は同一視せざるを得ない、ただそれだけのことなのです
498 名前:132人目の素数さん [2017/11/10(金) 09:56:34.07 ID:uPW/Vk/t.net] 内部ノード数 n の 2色木の Black Height を h であらわすと以下の不等式が成り立つ。 2^(h-1) - 1 ≦ n ≦ 2^(2*h-1) - 1 よって、 h = O(log(n)) とある本に書いてあります。 これっておかしいですよね? h は n の関数ではありません。内部ノード数から2色木の Black Height は一意的にはきまらないからです。 もちろん、 O(log(n)) の左辺には n の関数が来るきまりです。 ですので、 h = O(log(n)) と書くのはおかしいのではないでしょうか?
499 名前:132人目の素数さん mailto:sage [2017/11/10(金) 10:02:14.58 ID:lMPuBN0e.net] 馬鹿アスペのマルチ糞 データ構造,アルゴリズム,デザインパターン総合スレ 3©2ch.net mevius.2ch.net/test/read.cgi/tech/1466315249/686
500 名前:132人目の素数さん mailto:sage [2017/11/10(金) 12:35:17.62 ID:UYWEJAKP.net] >>453 1問でもいいので分かる人いませんかね
501 名前:132人目の素数さん [2017/11/10(金) 12:47:57.37 ID:uPW/Vk/t.net] >>453 R上の実数値連続関数全体のなす集合の濃度が|R|であることを示せ これは松坂和夫著『解析入門3』のp.37に同じ問題があります。 任意の有限集合である全順序集合は整列集合であることを示せ これは背理法で簡単に示せそうですね。 R-{0,1}においてx〜1-x, x〜1/x で生成される同値関係を定めるときのxの同値類を書け 「x〜1-x, x〜1/x で生成される同値関係」って何ですか?
502 名前:132人目の素数さん mailto:sage [2017/11/10(金) 12:57:41.38 ID:gy7x46k5.net] 群について質問がある。 位数3(a1,a2,a3)の群Gがあるとして、これに一種の結合(×)を考えて下記のような九九の表ができるとする。 a1 a2 a3 a1 a1 a2 a3 a2 a2 a3 a1 a3 a3 a1 a2 このとき3個の要素
503 名前:の間に入れ替えを行う方法は6通りある。 {a1,a2,a3} ↓ @{a1,a2,a3} A{a1,a3,a2} B{a2,a3,a1} C{a2,a1,a3} D{a3,a1,a2} E{a3,a2,a1} その中でも入れ替えを行ったあとでも九九の表がそのままのものがある、それは@とA。 本に書いてあるこれの意味がよくわからない。 G={a1,a2,a3} の元を G={a2,a3,a1} に入れ換えたら a2×a2=a3 という結合は a3×a3=a1 (a2×a2=a2) になる。 という解釈でよいの? これなら確かにAは九九の表が変わらないはず。 でもこれいまいちしっくりこない。 九九の表って、a2とa2が結合したらa3になりますよ、っていう関係を示してるんだよね? なんで元の配置に影響を受けるんだろう……。 根本的なことを理解できてないから、なんかアドバイスでもいいからほしい。 夜にまたくる。 [] [ここ壊れてます]
504 名前:132人目の素数さん mailto:sage [2017/11/10(金) 12:59:36.92 ID:cdHEEJDU.net] その同値関係を含む同値関係のうちで最小の同値関係だな 含む,最小てのは同値類が集合として含む,最小の意味ね
505 名前:132人目の素数さん mailto:sage [2017/11/10(金) 13:06:48.49 ID:cdHEEJDU.net] >>491 「入れ替え」てのは配置を変える意味じゃなく 変数名を変える意味だよ
506 名前:¥ mailto:sage [2017/11/10(金) 14:29:37.30 ID:f5tI1p7u.net] ¥
507 名前:¥ mailto:sage [2017/11/10(金) 14:29:55.31 ID:f5tI1p7u.net] ¥
508 名前:¥ mailto:sage [2017/11/10(金) 14:30:12.09 ID:f5tI1p7u.net] ¥
509 名前:¥ mailto:sage [2017/11/10(金) 14:30:29.14 ID:f5tI1p7u.net] ¥
510 名前:¥ mailto:sage [2017/11/10(金) 14:30:48.95 ID:f5tI1p7u.net] ¥
511 名前:¥ mailto:sage [2017/11/10(金) 14:31:12.49 ID:f5tI1p7u.net] ¥
512 名前:¥ mailto:sage [2017/11/10(金) 14:31:31.79 ID:f5tI1p7u.net] ¥
513 名前:¥ mailto:sage [2017/11/10(金) 14:31:49.34 ID:f5tI1p7u.net] ¥
514 名前:¥ mailto:sage [2017/11/10(金) 14:32:05.77 ID:f5tI1p7u.net] ¥
515 名前:¥ mailto:sage [2017/11/10(金) 14:32:24.88 ID:f5tI1p7u.net] ¥
516 名前:132人目の素数さん mailto:sage [2017/11/10(金) 15:08:30.24 ID:UYWEJAKP.net] >>490 同値関係の問題は自分も意味分かってないんですよね ちなみに原文そのまま載せてます
517 名前:132人目の素数さん [2017/11/10(金) 15:17:17.80 ID:mftSfBFV.net] 親切に書いてある留意点をガン無視して、定義も分からず問題を解く気なのか
518 名前:491 mailto:sage [2017/11/10(金) 17:02:44.50 ID:WUMu27wj.net] >>493 簡潔で的確な答えをありがとう。 なるほど、変数名を入れ換えるという意味か。 それならすっきりする。 すっきりしたことで新たな疑問も浮上したけどそこはなんとか自力で。 独学だからほんと助かる、感謝。
519 名前:132人目の素数さん [2017/11/10(金) 18:48:16.84 ID:PEs00RSo.net] ガロア理論のとこです。正規拡大の定義で、 L/Kが正規 :⇔任意のK準同型σについてσ(L)=L となっているのですが、実際はσ(L)⊂Lだけわかればよいと言われました 逆の包含はなぜ調べなくてよいのでしょうか
520 名前:¥ mailto:sage [2017/11/10(金) 20:10:23.14 ID:f5tI1p7u.net] ¥
521 名前:¥ mailto:sage [2017/11/10(金) 20:10:39.86 ID:f5tI1p7u.net] ¥
522 名前:¥ mailto:sage [2017/11/10(金) 20:10:56.04 ID:f5tI1p7u.net] ¥
523 名前:¥ mailto:sage [2017/11/10(金) 20:11:12.58 ID:f5tI1p7u.net] ¥
524 名前:¥ mailto:sage [2017/11/10(金) 20:11:27.11 ID:f5tI1p7u.net] ¥
525 名前:¥ mailto:sage [2017/11/10(金) 20:11:58.94 ID:f5tI1p7u.net] ¥
526 名前:¥ mailto:sage [2017/11/10(金) 20:12:15.52 ID:f5tI1p7u.net] ¥
527 名前:¥ mailto:sage [2017/11/10(金) 20:12:33.49 ID:f5tI1p7u.net] ¥
528 名前:¥ mailto:sage [2017/11/10(金) 20:12:50.21 ID:f5tI1p7u.net] ¥
529 名前:¥ mailto:sage [2017/11/10(金) 20:13:07.04 ID:f5tI1p7u.net] ¥
530 名前:132人目の素数さん [2017/11/10(金) 21:31:49.45 ID:iwP+F9Zm.net] >>507 σ-1(L)⊂LからL⊂σ(L)となるから
531 名前:132人目の素数さん [2017/11/10(金) 21:45:21.84 ID:iwP+F9Zm.net] >>490 A={(x,1-x)} B={(x,1/x)} C=AUB C*(n+1)={(x,y)|∃z (x,z
532 名前:)∈C*n∧(z,y)∈C} D=U[n=1,∞]C*n x〜y⇔(x,y)∈D [] [ここ壊れてます]
533 名前:132人目の素数さん [2017/11/10(金) 22:06:53.19 ID:iwP+F9Zm.net] >>453 >R上の実数値連続関数全体のなす集合の濃度が|R|であることを示せ 折線の極限よな 分点が可算個だから折線は連続個 よって折線の列は連続個 その同値類だから連続個以下 連続個以上は自明なので連続個
534 名前:132人目の素数さん [2017/11/10(金) 22:09:11.03 ID:iwP+F9Zm.net] >>453 >任意の有限集合である全順序集合は整列集合であることを示せ 有限な全順序なら最小がある 部分集合も有限な全順序
535 名前:132人目の素数さん mailto:sage [2017/11/10(金) 23:23:10.82 ID:KYIvy0Uh.net] 無理数に関する以下の2題に答えよ。 なお、2つの問題には直接的な関連はない。 (1)初項0、末項1の等差数列の公差は無理数となれるか。 (2){a}でaの小数部分を表す。例えば{3.14}=0.14、{3}=0、{-27.01}=-0.01である。 いま、nを自然数として無理数xに対し実数の列 {x},{2x},…,{nx},… を考えると、列に含まれるどの2つの数も相異なることを示せ。
536 名前:132人目の素数さん mailto:sage [2017/11/10(金) 23:25:45.04 ID:zIX+6Ycy.net] ・長寿ランキング of 特別枠 108歳 蟹江ぎん(1892/08/01〜2001/02/28) 107歳 成田きん(1892/08/01〜2000/01/23) 皇寿(111歳)には届きませんでしたが… ぎんさんの娘・4姉妹がこれを追っています。 ? Y.年子(1914/04/14〜)103 ? T.千多代(1918/10/21〜)99 ? S.百合子(1921/05/18〜)96 ? 蟹江美根代(1923/10/01〜)94 [前スレ.643] ? 瀬戸内寂聴(1922/05/15〜) 95 の次に ? 佐藤愛子 (1923/11/05〜)94 を追加
537 名前:132人目の素数さん mailto:sage [2017/11/10(金) 23:28:30.28 ID:NaWRmLAa.net] >>522 (1)明らかです (2)明らかです ちなみに、普通小数部分、といったら、x-[x]を指すので、-27.01の小数部分は0.99となります
538 名前:132人目の素数さん mailto:sage [2017/11/10(金) 23:30:49.63 ID:Bjz8q8/+.net] >>522 何処が分からないの? 両方とも整数/整数にならないことを示せばいいだけよね
539 名前:132人目の素数さん mailto:sage [2017/11/10(金) 23:39:05.27 ID:KYIvy0Uh.net] >>525 何で分数が出てくるんですか?
540 名前:132人目の素数さん mailto:sage [2017/11/10(金) 23:39:44.72 ID:NaWRmLAa.net] わからないんですね(笑)
541 名前:132人目の素数さん mailto:sage [2017/11/10(金) 23:40:42.05 ID:KYIvy0Uh.net] >>524 すいません 答えられないゴミに用はないので 透明あぼーんくらい高校生でも知ってます
542 名前:132人目の素数さん mailto:sage [2017/11/10(金) 23:43:03.14 ID:NaWRmLAa.net] >>522 an=0+(n-1)d=1を満たす無理数dは存在しません {nx}={mx}だとします nx-mx=(n-m)x=Nとなりますが、これを満たす無理数xは存在しません でも、これ自作問題ですよね?
543 名前:132人目の素数さん mailto:sage [2017/11/10(金) 23:59:32.02 ID:pqxY2Wrd.net] >>526 無理数の定義を知ってたらそんなこと聞かない 何処がわからないか書いてない時点で臭いぞ
544 名前:132人目の素数さん [2017/11/11(土) 00:00:24.01 ID:hsVnMSJT.net] >>526 有理数(整数/整数)でないのが無理数だから そうならないことを示す
545 名前:132人目の素数さん [2017/11/11(土) 00:00:45.69 ID:hsVnMSJT.net] >>527 ぷ
546 名前:132人目の素数さん mailto:sage [2017/11/11(土) 00:07:32.27 ID:Xw9swpYT.net] >>531 ということは、あなたや私も無理数なのでしょうか?
547 名前:132人目の素数さん [2017/11/11(土) 00:09:27.02 ID:hsVnMSJT.net] >>453 >R-{0,1}においてx〜1-x, x〜1/x で生成される同値関係を定めるときのxの同値類を書け {x,1-x,1/x,1/(1-x),1-1/x,x/(x-1)} ちなみにS_3と同型
548 名前:132人目の素数さん [2017/11/11(土) 00:10:53.58 ID:hsVnMSJT.net] >>533 面白いことを書いてる人が居るぞ!
549 名前:132人目の素数さん mailto:sage [2017/11/11(土) 00:11:14.66 ID:Xw9swpYT.net] >>535 あなたは明らかに有理数ではないですから、無理数ですよね
550 名前:132人目の素数さん [2017/11/11(土) 00:20:40.71 ID:hsVnMSJT.net] >>536 いいぞ!
551 名前:132人目の素数さん mailto:sage [2017/11/11(土) 00:21:29.82 ID:Kk19NacV.net] Aは有理数ではない → Aは無理数 これは偽です
552 名前:132人目の素数さん [2017/11/11(土) 00:21:49.87 ID:aQhOn7fm.net] 超巨大宇宙文明を独力で築くのと仏になるのはどっちの方が難しいですか?
553 名前:132人目の素数さん [2017/11/11(土) 00:22:53.49 ID:hsVnMSJT.net] >>538 素晴らしい人も来たな!
554 名前:132人目の素数さん mailto:sage [2017/11/11(土) 00:48:55.36 ID:Xw9swpYT.net] >>538 なぜですか?
555 名前:132人目の素数さん mailto:sage [2017/11/11(土) 00:53:45.59 ID:TDDt/YME.net] 実数とは言ってないから
556 名前:132人目の素数さん mailto:sage [2017/11/11(土) 00:54:41.91 ID:Xw9swpYT.net] つまり、>>531 は間違えということでしょうか
557 名前:132人目の素数さん mailto:sage [2017/11/11(土) 00:59:47.36 ID:TDDt/YME.net] >>543 (1)は項数が自然数だから実数上の話 (2)も小数部分を定義できる時点で実数上の話 ただ実数でないのなら有理数でも無理数でもない
558 名前:132人目の素数さん mailto:sage [2017/11/11(土) 01:09:07.56 ID:Xw9swpYT.net] >>544 複素数かもしれませんよ? (2)で定義される小数部分は、通常のものとは異なりますから、実数部の「小数部分」を考えている可能性もあります -27.01の「小数部分」が-0.01になるような演算ですが、具体的に定義されてないわけですから、わかりませんね
559 名前:¥ mailto:sage [2017/11/11(土) 01:39:25.73 ID:4aYiSbvq.net] ¥
560 名前:¥ mailto:sage [2017/11/11(土) 01:39:43.08 ID:4aYiSbvq.net] ¥
561 名前:¥ mailto:sage [2017/11/11(土) 01:40:01.23 ID:4aYiSbvq.net] ¥
562 名前:¥ mailto:sage [2017/11/11(土) 01:40:19.82 ID:4aYiSbvq.net] ¥
563 名前:¥ mailto:sage [2017/11/11(土) 01:40:38.95 ID:4aYiSbvq.net] ¥
564 名前:¥ mailto:sage [2017/11/11(土) 01:40:57.25 ID:4aYiSbvq.net] ¥
565 名前:¥ mailto:sage [2017/11/11(土) 01:41:14.63 ID:4aYiSbvq.net] ¥
566 名前:¥ mailto:sage [2017/11/11(土) 01:41:37.80 ID:4aYiSbvq.net] ¥
567 名前:¥ mailto:sage [2017/11/11(土) 01:41:58.38 ID:4aYiSbvq.net] ¥
568 名前:¥ mailto:sage [2017/11/11(土) 01:42:19.55 ID:4aYiSbvq.net] ¥
569 名前:132人目の素数さん mailto:sage [2017/11/11(土) 01:42:58.04 ID:TDDt/YME.net] >>545 例えに正の数と負の数にしか触れていない { }をそれぞれの絶対値の小数部分と定義している つまり実数にしか定義していない もし仮に複素数の実部の小数部分を定義させるなら例に挙げなくてはいけない 何故ならば、{-27.01}=0.01となるので{ }の絶対値を取っているが ・複素数の実部の絶対値の小数部分 例:{-0.4+0.3i}={|-0.4|}={0.4}=0.4 ・複素数の絶対値の小数部分 例:{-0.4+0.3i}={|-0.4+0.3i|}={0.5}=0.5 このように複素数について一意に定義されていないのでこの問題中ではその範囲で考えることはできない
570 名前:132人目の素数さん mailto:sage [2017/11/11(土) 04:06:31.06 ID:Xw9swpYT.net] >>556 小数部分はx-[x]で定義されます -27.01の小数部分とは、0.99です それを0.01としているのですから、何か未知の演算だと考えるべきです 複素数の実数部を取るという計算を考えていない保証はどこにもないですね
571 名前:132人目の素数さん [2017/11/11(土) 04:53:11.82 ID:aQhOn7fm.net] 魔界と神界と全と無と空を格付けするとどうなりますか?
572 名前:¥ mailto:sage [2017/11/11(土) 06:55:09.13 ID:4aYiSbvq.net] ¥
573 名前:¥ mailto:sage [2017/11/11(土) 06:55:27.39 ID:4aYiSbvq.net] ¥
574 名前:¥ mailto:sage [2017/11/11(土) 06:55:45.69 ID:4aYiSbvq.net] ¥
575 名前:¥ mailto:sage [2017/11/11(土) 06:56:03.15 ID:4aYiSbvq.net] ¥
576 名前:¥ mailto:sage [2017/11/11(土) 06:56:19.75 ID:4aYiSbvq.net] ¥
577 名前:¥ mailto:sage [2017/11/11(土) 06:56:38.38 ID:4aYiSbvq.net] ¥
578 名前:¥ mailto:sage [2017/11/11(土) 06:56:56.44 ID:4aYiSbvq.net] ¥
579 名前:¥ mailto:sage [2017/11/11(土) 06:57:19.20 ID:4aYiSbvq.net] ¥
580 名前:¥ mailto:sage [2017/11/11(土) 06:57:36.96 ID:4aYiSbvq.net] ¥
581 名前:¥ mailto:sage [2017/11/11(土) 06:57:54.01 ID:4aYiSbvq.net] ¥
582 名前:132人目の素数さん mailto:sage [2017/11/11(土) 07:01:46.47 ID:PMyvgno+.net] >>487 意味を考えずに形式だけで追いかける人には 一生理解できないから諦めなさい
583 名前:132人目の素数さん [2017/11/11(土) 07:39:28.30 ID:hsVnMSJT.net] >>556 その ID:Xw9swpYT の人触んない方がいい人みたいよ
584 名前:¥ mailto:sage [2017/11/11(土) 09:19:24.38 ID:4aYiSbvq.net] ¥
585 名前:¥ mailto:sage [2017/11/11(土) 09:19:42.30 ID:4aYiSbvq.net] ¥
586 名前:¥ mailto:sage [2017/11/11(土) 09:20:01.39 ID:4aYiSbvq.net] ¥
587 名前:¥ mailto:sage [2017/11/11(土) 09:20:20.94 ID:4aYiSbvq.net] ¥
588 名前:¥ mailto:sage [2017/11/11(土) 09:20:38.60 ID:4aYiSbvq.net] ¥
589 名前:¥ mailto:sage [2017/11/11(土) 09:20:56.70 ID:4aYiSbvq.net] ¥
590 名前:¥ mailto:sage [2017/11/11(土) 09:21:13.75 ID:4aYiSbvq.net] ¥
591 名前:¥ mailto:sage [2017/11/11(土) 09:21:31.35 ID:4aYiSbvq.net] ¥
592 名前:¥ mailto:sage [2017/11/11(土) 09:21:48.99 ID:4aYiSbvq.net] ¥
593 名前:¥ mailto:sage [2017/11/11(土) 09:22:08.55 ID:4aYiSbvq.net] ¥
594 名前:132人目の素数さん mailto:sage [2017/11/11(土) 14:09:53.44 ID:1yWDMyzv.net] 惨めな奴
595 名前:132人目の素数さん [2017/11/11(土) 18:33:54.72 ID:QsPz7mgY.net] y=x ^ e^ xの微分を教えてください
596 名前:132人目の素数さん [2017/11/11(土) 20:05:55.18 ID:+XdJQyKl.net] n,mを正の整数とする。n個のさいころを同時にふり、すべてのさいころの目の和がn+1なら1点、n+2なら2点、n+3なら3点を、1回の得点として与えるゲームを 行う。2m回後の得点が3の倍数となる確率pを求めよ。
597 名前:132人目の素数さん mailto:sage [2017/11/11(土) 20:18:18.51 ID:050e/q1e.net] >>582 両辺に自然対数取ればいい
598 名前:¥ mailto:sage [2017/11/11(土) 21:00:58.14 ID:4aYiSbvq.net] ¥
599 名前:¥ mailto:sage [2017/11/11(土) 21:01:17.11 ID:4aYiSbvq.net] ¥
600 名前:¥ mailto:sage [2017/11/11(土) 21:01:35.61 ID:4aYiSbvq.net] ¥
601 名前:¥ mailto:sage [2017/11/11(土) 21:01:56.57 ID:4aYiSbvq.net] ¥
602 名前:¥ mailto:sage [2017/11/11(土) 21:02:14.66 ID:4aYiSbvq.net] ¥
603 名前:¥ mailto:sage [2017/11/11(土) 21:02:40.75 ID:4aYiSbvq.net] ¥
604 名前:¥ mailto:sage [2017/11/11(土) 21:02:58.74 ID:4aYiSbvq.net] ¥
605 名前:132人目の素数さん mailto:sage [2017/11/11(土) 21:03:15.16 ID:Xw9swpYT.net] しっかし誰も解けない難しい質問ばっかでつまんねえなぁ。 本当に「実際は解いている連中ばっか」状態になったこと一度もねえじゃんw もっと簡単な質問してこい、脳みそウンコまみ
606 名前:れの底辺層ども。 [] [ここ壊れてます]
607 名前:¥ mailto:sage [2017/11/11(土) 21:03:16.27 ID:4aYiSbvq.net] ¥
608 名前:¥ mailto:sage [2017/11/11(土) 21:03:33.03 ID:4aYiSbvq.net] ¥
609 名前:¥ mailto:sage [2017/11/11(土) 21:04:08.41 ID:4aYiSbvq.net] ¥
610 名前:¥ mailto:sage [2017/11/11(土) 21:04:27.59 ID:4aYiSbvq.net] ¥
611 名前:¥ mailto:sage [2017/11/11(土) 21:04:47.03 ID:4aYiSbvq.net] ¥
612 名前:¥ mailto:sage [2017/11/11(土) 21:05:05.70 ID:4aYiSbvq.net] ¥
613 名前:¥ mailto:sage [2017/11/11(土) 21:05:28.16 ID:4aYiSbvq.net] ¥
614 名前:¥ mailto:sage [2017/11/11(土) 21:05:47.85 ID:4aYiSbvq.net] ¥
615 名前:¥ mailto:sage [2017/11/11(土) 21:06:07.61 ID:4aYiSbvq.net] ¥
616 名前:¥ mailto:sage [2017/11/11(土) 21:06:25.16 ID:4aYiSbvq.net] ¥
617 名前:132人目の素数さん mailto:sage [2017/11/11(土) 21:07:19.90 ID:Xw9swpYT.net] 今日も「解いた側」の圧勝かぁ・・・。 毎日毎日、ラクラク解ける問題ばかりだから常勝なんだよね・・・。 たまには、解けない解けないっと悩んで負けてみたい、それが今の切実な悩み。
618 名前:132人目の素数さん [2017/11/11(土) 21:20:23.03 ID:QsPz7mgY.net] y=x ^ e^ xの微分を教えてください e= l i m (1+k)1/k k→0
619 名前:132人目の素数さん [2017/11/11(土) 21:22:14.87 ID:QsPz7mgY.net] 603の方、そんなに博識な方ならばどうか604の問題を解いていただけないでしょうか?
620 名前:132人目の素数さん [2017/11/11(土) 21:24:58.22 ID:QsPz7mgY.net] この問題は、基礎的な問題集の応用レベルの問題なので時間はくいはしませんから
621 名前:132人目の素数さん [2017/11/11(土) 21:29:51.01 ID:hsVnMSJT.net] >>605 彼にはそれは無理だろうし 触らない方がいい人だよw
622 名前:132人目の素数さん mailto:sage [2017/11/11(土) 21:31:40.02 ID:Xw9swpYT.net] >>606 基礎的な問題集の基礎的な問題のも違えですよね てか、これもまた自作問題ですよね 自作した跡が見えてますよ
623 名前:132人目の素数さん mailto:sage [2017/11/11(土) 21:32:05.66 ID:Xw9swpYT.net] >>607 ある無矛盾な公理系τの任意のモデルに対してある論理式φが常に真となるならば、τからφがLKにおいて証明可能となることを示せ、という問題がわかりません
624 名前:132人目の素数さん [2017/11/11(土) 21:33:45.54 ID:QsPz7mgY.net] 607ありがとう
625 名前:132人目の素数さん [2017/11/11(土) 21:33:54.49 ID:hsVnMSJT.net] >410 1 名前:132人目の素数さん Mail:sage 投稿日:2017/11/09(木) 00:42:34.89 ID:uCu6B49W >整式の定義が分かっていない人に指摘をしただけですよ
626 名前:132人目の素数さん mailto:sage [2017/11/11(土) 21:35:54.13 ID:Xw9swpYT.net] >>611 三段論法を用いる任意の数学の証明は、三段論法を用いない別証明を持つことを示せ、という問題がわかりません
627 名前:132人目の素数さん [2017/11/11(土) 21:37:05.32 ID:hsVnMSJT.net] >>610 時間を無駄にしない方がいいよね ちなみにその問題は x=e^logxを左のxに使って指数法則と合成関数の微分法を使うとすぐ
628 名前:132人目の素数さん [2017/11/11(土) 21:40:09.45 ID:hsVnMSJT.net] >420 名前:132人目の素数さん Mail:sage 投稿日:2017/11/09(木) 07:05:01.81 ID:uCu6B49W >>>419 >あなたを殺すにはどうすれば良いでしょうか? >422 名前:132人目の素数さん Mail:sage 投稿日:2017/11/09(木) 07:05:34.14 ID:uCu6B49W >>>421 >あなたを殺す方法を教えてください >423 名前:132人目の素数さん Mail:sage 投稿日:2017/11/09(木) 07:06:33.88 ID:uCu6B49W >ID:KmBKGqZpさんを殺害する方法がわかりません >よろしくお願いします >425 1 名前:132人目の素数さん Mail:sage 投稿日:2017/11/09(木) 07:09:37.77 ID:uCu6B49W >ここの回答者って、自分がどうやったら死ぬのかもわからないような、低レベルな人たちだったんですね
629 名前:132人目の素数さん mailto:sage [2017/11/11(土) 21:40:41.19 ID:Xw9swpYT.net] >>614 わからないんですか?
630 名前:132人目の素数さん [2017/11/11(土) 21:42:30.21 ID:QsPz7mgY.net] 本当に物分かりが悪くて申し訳ないのですが x= e^ log x を用いるのはどうしてでしょうか
631 名前:132人目の素数さん [2017/11/11(土) 21:44:04.78 ID:QsPz7mgY.net] 大変申し訳ないのですが細かな説明を添えていただけたらさいわいです
632 名前:132人目の素数さん mailto:sage [2017/11/11(土) 21:50:48.04 ID:Xw9swpYT.net] 答えがなかなか来ませんね わからないんでしょうか?
633 名前:132人目の素数さん mailto:sage [2017/11/11(土) 21:52:50.64 ID:V5IIyttr.net] y=x^(e^x) log(y)=log(x^(e^x)) log(y)=(e^x)log(x) 両辺をxで微分して (1/y)y'=(e^x)log(x)+(e^x)(1/x) y'=(x^(e^x))(e^x)(log(x)+(1/x))
634 名前:132人目の素数さん mailto:sage [2017/11/11(土) 21:53:59.35 ID:V5IIyttr.net] >>618 わからないんですね(笑)
635 名前:132人目の素数さん mailto:sage [2017/11/11(土) 21:55:42.97 ID:Xw9swpYT.net] >>614 さんは、結局、わからなかったというわけですね(笑)
636 名前:132人目の素数さん mailto:sage [2017/11/11(土) 21:56:27.14 ID:V5IIyttr.net] >>618 さんは高校数学すら判別できないんですね(笑)
637 名前:132人目の素数さん [2017/11/11(土) 21:56:28.18 ID:DC/G4Hnt.net] 連結集合の定義が直観的につながっているというイメージと一致することを説明してください。
638 名前:132人目の素数さん mailto:sage [2017/11/11(土) 22:00:48.63 ID:Xw9swpYT.net] >>623 2つの開集合の和として書き表すことができないというのが連結であるということです もし2つに分かれたら、それは繋がってないわけです しかし、連結の場合はどんなに頑張っても分けられないと言ってるわけですから、直感と一致しますよね
639 名前:132人目の素数さん [2017/11/11(土) 22:01:34.97 ID:QsPz7mgY.net] おかげさまでようやく理解しましたありがとうございました
640 名前:132人目の素数さん [2017/11/11(土) 22:02:55.
] [ここ壊れてます]
641 名前:95 ID:DC/G4Hnt.net mailto: 直観的につながっている ⇒ 連結 のほうはOKですが、 連結 ⇒ 直観的につながっている (直観的につながっていない ⇒ 連結でない) のほうはどうでしょうか? [] [ここ壊れてます]
642 名前:132人目の素数さん [2017/11/11(土) 22:06:22.11 ID:DC/G4Hnt.net] >>624 2つの「開集合」としているのに必然性はありますか? 2つの空でない共通部分をもたない「集合」に分けられない という定義がダメなのは分かりますが。
643 名前:132人目の素数さん mailto:sage [2017/11/11(土) 22:08:49.25 ID:Xw9swpYT.net] >>626 直観的に繋がっていること、の定義なんてできると思いますか? 我々が認知することができるのは、せいぜい距離空間だけでしょう それ以外の一般的な位相空間では、連結どころか開集合そのものだってイメージが曖昧になってしまいます 直観は数学をする上で大切ですが、直観そのものを数学の議論そのものに持ち込もうとすることは、危険です そんなことはできないのですよ 直観というわけのわからないものを排除するために、抽象化して、無機質な言葉による定義に置き換えてしまうわけです ですから、前者がなんとなくわかれば、イメージの説明は終わりなわけです イメージなんてのはその程度の扱いで十分です イメージは数学の対象物ではないのですから
644 名前:132人目の素数さん [2017/11/11(土) 22:11:13.77 ID:DC/G4Hnt.net] >>628 では、連結の定義を何か別の同値でない定義に数学者達がしなかった理由は 説明できないということですか?
645 名前:132人目の素数さん mailto:sage [2017/11/11(土) 22:11:33.12 ID:Xw9swpYT.net] >>627 集合としてしまうことには問題があるでしょうね [0,2]は繋がっているわけですが、仮に連結の定義を2つの集合にわけられない、としてしまうと、[0,2]=[0,1]∪(1,2]こういう風に分けられてしまいますよね また、先程の繰り返しですが、だからといってなんで開集合というものがつながっているイメージを説明することに必要十分なんだ、ということは意味のないことです
646 名前:132人目の素数さん mailto:sage [2017/11/11(土) 22:13:06.49 ID:Xw9swpYT.net] >>629 弧状連結という概念がありますよね これもまた、繋がっていることを示す一つの方法です 弧状連結→連結ですが、逆は成り立ちませんから、これらの概念は異なるものです しかしながら、やはり、どちらの概念が本当なんだ、というような議論は無意味なわけです
647 名前:132人目の素数さん mailto:sage [2017/11/11(土) 22:19:32.22 ID:Xw9swpYT.net] まあ、実数に限れば、連結成分は区間しか存在しないということが示せるわけですから、一つの答えにはなってるでしょうかね 直観的にも数直線上で繋がってるところは、区間しかないわけです
648 名前:132人目の素数さん mailto:sage [2017/11/11(土) 22:24:24.83 ID:N0kgKgqz.net] 位相多様体の場合、連結であることと弧状連結であることは同値な条件
649 名前:¥ mailto:sage [2017/11/12(日) 00:06:13.44 ID:AbMINYSr.net] ¥
650 名前:¥ mailto:sage [2017/11/12(日) 00:06:31.71 ID:AbMINYSr.net] ¥
651 名前:¥ mailto:sage [2017/11/12(日) 00:06:49.87 ID:AbMINYSr.net] ¥
652 名前:¥ mailto:sage [2017/11/12(日) 00:07:06.78 ID:AbMINYSr.net] ¥
653 名前:¥ mailto:sage [2017/11/12(日) 00:07:27.32 ID:AbMINYSr.net] ¥
654 名前:¥ mailto:sage [2017/11/12(日) 00:07:45.65 ID:AbMINYSr.net] ¥
655 名前:¥ mailto:sage [2017/11/12(日) 00:08:04.61 ID:AbMINYSr.net] ¥
656 名前:¥ mailto:sage [2017/11/12(日) 00:08:21.90 ID:AbMINYSr.net] ¥
657 名前:¥ mailto:sage [2017/11/12(日) 00:08:39.94 ID:AbMINYSr.net] ¥
658 名前:¥ mailto:sage [2017/11/12(日) 00:08:58.84 ID:AbMINYSr.net] ¥
659 名前:132人目の素数さん [2017/11/12(日) 00:28:29.23 ID:GGaVEi9w.net] >>625 一応終わったようだけど >>616 x^f(x)を微分する代わりにe^(f(x)logx)を微分する方が積の合成になるから楽ということ 合成されている所が対数微分法で出てくるところだからそれと同じことをやるのだけど
660 名前:132人目の素数さん mailto:sage [2017/11/12(日) 00:31:47.50 ID:HCnP3xyT.net] 連結だが弧状連結でない位相空間の例があるが、それは病的だと言われる 一般の位相空間は直観的に繋がってるかどうかを論じる対象ではないってことだ 位相幾何学の主たる研究対象は位相多様体となる
661 名前:132人目の素数さん mailto:sage [2017/11/12(日) 00:32:18.11 ID:cPywtfD3.net] >>644 x^f(x)はどのように微分するのでしょうか?
662 名前:132人目の素数さん [2017/11/12(日) 00:35:43.52 ID:GGaVEi9w.net] >>646 どうするんでしょうねえ
663 名前:132人目の素数さん mailto:sage [2017/11/12(日) 00:36:36.11 ID:cPywtfD3.net] >>647 あなたは、e^f(x)logを使うと楽になるといいましたね 別解があるということですよね
664 名前:132人目の素数さん [2017/11/12(日) 00:37:42.19 ID:GGaVEi9w.net] >>648 また可哀想な人だったか
665 名前:132人目の素数さん mailto:sage [2017/11/12(日) 00:38:53.53 ID:cPywtfD3.net] >>649 わからないんですか? 本当はわからないのに、嘘をついたということでしょうか? [] [ここ壊れてます]
667 名前:132人目の素数さん mailto:sage [2017/11/12(日) 00:40:00.55 ID:HCnP3xyT.net] 重箱の隅つつくことで満足する自尊心ってどんだけ安いのよ
668 名前:132人目の素数さん [2017/11/12(日) 00:40:37.00 ID:GGaVEi9w.net] だよねー
669 名前:132人目の素数さん mailto:sage [2017/11/12(日) 00:41:16.45 ID:cPywtfD3.net] ここの回答者って、息をするように嘘を付く人たちの集まりだったんですね 失望しました
670 名前:132人目の素数さん [2017/11/12(日) 00:43:22.64 ID:GGaVEi9w.net] たぶん ID:cPywtfD3 が失望して出て行ってくれれば 皆喜ぶんじゃないかな
671 名前:132人目の素数さん mailto:sage [2017/11/12(日) 00:59:21.80 ID:/4m9SASK.net] 別解 y=x^(e^x) y=(e^log(x))^(e^x) y=e^((e^x)log(x)) y'=(e^((e^x)log(x)))((e^x)log(x))' y'=(x^(e^x))((e^x)log(x)+(e^x)(1/x)) y'=(x^(e^x))(e^x)(log(x)+(1/x)) 満足?
672 名前:132人目の素数さん mailto:sage [2017/11/12(日) 01:06:15.08 ID:cPywtfD3.net] >>655 だから、それが>>654 の解法ですよね? >>654 は対数微分法を使ってませんから
673 名前:132人目の素数さん [2017/11/12(日) 01:09:26.90 ID:MPt+syL4.net] >>654 って解答書いたんだ
674 名前:132人目の素数さん mailto:sage [2017/11/12(日) 01:11:16.97 ID:cPywtfD3.net] >>644 書いてますよね?
675 名前:132人目の素数さん [2017/11/12(日) 01:27:40.07 ID:GGaVEi9w.net] >>658 書いてますか?
676 名前:132人目の素数さん mailto:sage [2017/11/12(日) 01:38:27.38 ID:w4fQieDK.net] 可哀想な人だとわかったら、もうレスしなきゃ良いのに
677 名前:132人目の素数さん mailto:sage [2017/11/12(日) 01:40:57.84 ID:+jphTJpC.net] >>42 般若parameter 心経  ̄ ̄ ̄ ̄ ̄ ̄ ̄ ̄ 観自在菩薩・行深 般若parameter 時、照見五蘊皆空、度一切苦厄。 舎利子。 色不異空、空不異色、色即是空、空即是色。 受・想・行・識・亦復如是。 舎利子。 是諸法空相、不生不滅、不垢不浄、不増不減。 是故空中、無色、無受・想・行・識、無眼・耳・鼻・舌・身・意、無色・声・香・味・触・法。 無眼界、乃至、無意識界。 無無明・亦無無明尽、乃至、無老死、亦無老死尽。 無苦・集・滅・道。 無智、亦無得。 以無所得故、菩提薩埵、依 般若parameter 故、心無罣礙、無罣礙故、無有恐怖、遠離・一切・顛倒夢想、究竟涅槃。 三世諸仏、依 般若parameter 故、得阿耨多羅三藐三菩提。 故知、般若parameter、是大神呪、是大明呪、是無上呪、是無等等呪、能除一切苦、真実不虚。 故説、般若parameter 呪。 即説呪曰、羯諦羯諦、波羅羯諦、波羅僧羯諦、菩提薩婆訶。 般若心経 * 般若parameter の内容は宗派によって違うんだろうな…
678 名前:132人目の素数さん [2017/11/12(日) 02:05:36.91 ID:nEVJN+VK.net] 1日10億円以上稼げるデイトレーダーになるにはどうすれば良いですか?
679 名前:132人目の素数さん mailto:sage [2017/11/12(日) 02:28:57.54 ID:Ol3q012R.net] 1円を100億倍レバして、それを1割増やすと(110億-100億-1)円の利益になる 1割減らしてしまうと(90億-100億-1)円の利益になる
680 名前:¥ mailto:sage [2017/11/12(日) 03:01:07.63 ID:AbMINYSr.net] ¥
681 名前:¥ mailto:sage [2017/11/12(日) 03:01:24.61 ID:AbMINYSr.net] ¥
682 名前:¥ mailto:sage [2017/11/12(日) 03:01:42.06 ID:AbMINYSr.net] ¥
683 名前:¥ mailto:sage [2017/11/12(日) 03:01:58.20 ID:AbMINYSr.net] ¥
684 名前:¥ mailto:sage [2017/11/12(日) 03:02:16.26 ID:AbMINYSr.net] ¥
685 名前:¥ mailto:sage [2017/11/12(日) 03:02:34.04 ID:AbMINYSr.net] ¥
686 名前:¥ mailto:sage [2017/11/12(日) 03:02:51.21 ID:AbMINYSr.net] ¥
687 名前:¥ mailto:sage [2017/11/12(日) 03:03:08.15 ID:AbMINYSr.net] ¥
688 名前:¥ mailto:sage [2017/11/12(日) 03:03:25.72 ID:AbMINYSr.net] ¥
689 名前:¥ mailto:sage [2017/11/12(日) 03:03:44.00 ID:AbMINYSr.net] ¥
690 名前:132人目の素数さん [2017/11/12(日) 04:52:51.24 ID:nEVJN+VK.net] ガウスとかオイラーみたいな超天才数学者とBNFみたいな超天才デイトレーダーはどっちの方が天才であると言えるのでしょうか?
691 名前:132人目の素数さん mailto:sage [2017/11/12(日) 07:04:59.65 ID:VL17AsZ4.net] 2つの二項係数の和mCr+nCsが1つの二項係数で表されるための必要十分条件を述べよ。 すなわち、mCr+nCs=aCbとなる自然数a,bが存在するために、自然数m,r,n,sが満たすべき必要十分条件を述べよ。 ただし、m≧2かつn≧2かつ1≦r≦m-1かつ1≦s≦n-1とする。
692 名前:132人目の素数さん mailto:sage [2017/11/12(日) 08:51:14.54 ID:+dIbkBl6.net] >>6
693 名前:75 1+1=2 [] [ここ壊れてます]
694 名前:¥ mailto:sage [2017/11/12(日) 09:22:06.45 ID:AbMINYSr.net] ¥
695 名前:¥ mailto:sage [2017/11/12(日) 09:22:24.63 ID:AbMINYSr.net] ¥
696 名前:¥ mailto:sage [2017/11/12(日) 09:22:41.90 ID:AbMINYSr.net] ¥
697 名前:¥ mailto:sage [2017/11/12(日) 09:22:59.60 ID:AbMINYSr.net] ¥
698 名前:¥ mailto:sage [2017/11/12(日) 09:23:16.57 ID:AbMINYSr.net] ¥
699 名前:¥ mailto:sage [2017/11/12(日) 09:23:33.92 ID:AbMINYSr.net] ¥
700 名前:¥ mailto:sage [2017/11/12(日) 09:23:54.27 ID:AbMINYSr.net] ¥
701 名前:¥ mailto:sage [2017/11/12(日) 09:24:11.28 ID:AbMINYSr.net] ¥
702 名前:¥ mailto:sage [2017/11/12(日) 09:24:28.56 ID:AbMINYSr.net] ¥
703 名前:¥ mailto:sage [2017/11/12(日) 09:24:46.50 ID:AbMINYSr.net] ¥
704 名前:132人目の素数さん mailto:sage [2017/11/12(日) 09:37:11.12 ID:cTg/FCp5.net] 問題(大学1年程度) Q1. [0,1]上至るところで不連続な関数を1つ示せ Q2. [0,1]上の有理数で不連続、無理数で連続な関数を1つ示せ Q3. [0,1]上の有理数で不連続、無理数で微分可能(当然連続)な関数を1つ示せ
705 名前:132人目の素数さん mailto:sage [2017/11/12(日) 09:42:58.58 ID:Ol3q012R.net] r;ァ'N;:::::::::::::,ィ/ >::::::::::ヽ . 〃 ヽル1'´ ∠:::::::::::::::::i i′ ___, - ,. = -一  ̄l:::::::::::::::l . ! , -==、´r' l::::::/,ニ.ヽ l _,, -‐''二ゝ l::::l f゙ヽ |、 ここはお前の自作問置き場じゃねえんだ レー-- 、ヽヾニ-ァ,ニ;=、_ !:::l ) } ト ヾ¨'7"ry、` ー゙='ニ,,,` }::ヽ(ノ チラシの裏にでも書いてろ :ーゝヽ、 !´ " ̄ 'l,;;;;,,,.、 ,i:::::::ミ ::::::::::::::::ヽ.-‐ ト、 r'_{ __)`ニゝ、 ,,iリ::::::::ミ ::::::::::::::::::::Vi/l:::V'´;ッ`ニ´ー-ッ-,、:::::`"::::::::::::::;゙ , な! :::::::::::::::::::::::::N. ゙、::::ヾ,.`二ニ´∠,,.i::::::::::::::::::::/// :::::::::::::::::::::::::::::l ヽ;:::::::::::::::::::::::::::::::::::::::::::/ / ::::::::::::::::::::::::::::::! :|.\;::::::::::::::::::::::::::::::/ /
706 名前:132人目の素数さん mailto:sage [2017/11/12(日) 10:33:59.12 ID:VL17AsZ4.net] >>676 バカじゃねーの?野垂れ死ねw
707 名前:132人目の素数さん mailto:sage [2017/11/12(日) 10:39:11.68 ID:VL17AsZ4.net] >>675 この傑作問題を誰か解け
708 名前:132人目の素数さん mailto:sage [2017/11/12(日) 10:43:32.70 ID:+jphTJpC.net] >>687 A 1. f(x)= 1 (x:有理数) = 0 (x:無理数) 「ディリクレの関数」というらしい。 この手のリーマン積分できない関数に対処するために、ルベーグ積分が考案された(?)
709 名前:132人目の素数さん mailto:sage [2017/11/12(日) 11:05:17.75 ID:+jphTJpC.net] >>675 >>690 生姜ねぇ。 a = mCr + nCs, b = 1,
710 名前:132人目の素数さん mailto:sage [2017/11/12(日) 11:18:02.79 ID:+jphTJpC.net] >>675 >>690 チョト変えて a = mCr + nCs, b = a-1
711 名前:132人目の素数さん mailto:sage [2017/11/12(日) 11:28:57.01 ID:VL17AsZ4.net] >>692 それ必要十分か?
712 名前:¥ mailto:sage [2017/11/12(日) 12:08:29.59 ID:AbMINYSr.net] ¥
713 名前:¥ mailto:sage [2017/11/12(日) 12:08:45.67 ID:AbMINYSr.net] ¥
714 名前:¥ mailto:sage [2017/11/12(日) 12:09:01.94 ID:AbMINYSr.net] ¥
715 名前:¥ mailto:sage [2017/11/12(日) 12:09:24.54 ID:AbMINYSr.net] ¥
716 名前:¥ mailto:sage [2017/11/12(日) 12:09:41.08 ID:AbMINYSr.net] ¥
717 名前:¥ mailto:sage [2017/11/12(日) 12:09:58.60 ID:AbMINYSr.net] ¥
718 名前:¥ mailto:sage [2017/11/12(日) 12:10:14.37 ID:AbMINYSr.net] ¥
719 名前:¥ mailto:sage [2017/11/12(日) 12:10:29.70 ID:AbMINYSr.net] ¥
720 名前:¥ mailto:sage [2017/11/12(日) 12:10:46.32 ID:AbMINYSr.net] ¥
721 名前:¥ mailto:sage [2017/11/12(日) 12:11:02.24 ID:AbMINYSr.net] ¥
722 名前:132人目の素数さん mailto:sage [2017/11/12(日) 12:16:56.93 ID:cPywtfD3.net] >>694 mCr+nCs=aCbとなる自然数a,bが存在するために、自然数m,r,n,sが満たすべき必要十分条件は、mCr+nCs=aCbとなる自然数a,bが存在することです
723 名前:132人目の素数さん mailto:sage [2017/11/12(日) 12:23:11.44 ID:VL17AsZ4.net] >>705 無職だろお前w はいNG
724 名前:132人目の素数さん mailto:sage [2017/11/12(日) 12:35:08.20 ID:cPywtfD3.net] >>706 ある無矛盾な公理系τの任意のモデルに対してある論理式φが常に真となるならば、τからφがLKにおいて証明可能となることを示せ、という問題がわかりません
725 名前:132人目の素数さん mailto:sage [2017/11/12(日) 12:48:17.62 ID:cPywtfD3.net] >>675 てか、>>693 これでいいじゃないですか? どんなm,r,n,sを選んでも、aとbを>>693 のように選べば必ず条件を満たすa,bが存在していることになります
726 名前:132人目の素数さん mailto:sage [2017/11/12(日) 12:51:11.01 ID:+jphTJpC.net] >>687 A 2. f(x)= 1/q (x:有理数(x=p/q、p:整数、q:自然数、互いに素)) = 0 (x:無理数) detail.chiebukuro.yahoo.co.jp/qa/question_detail/q1127539791
727 名前:132人目の素数さん mailto:sage [2017/11/12(日) 13:11:50.85 ID:+jphTJpC.net] >>708 いいね♡
728 名前:132人目の素数さん [2017/11/12(日) 13:53:31.67 ID:/Vhh1l7j.net] R^2 の部分距離空間を A とする。 A は互いに共通部部分をもたない R^2 の連結部分集合の和集合としてあらわされることを示せ。
729 名前:132人目の素数さん mailto:sage [2017/11/12(日) 14:23:42.45 ID:cPywtfD3.net]
730 名前:>>711 明らかです [] [ここ壊れてます]
731 名前:¥ mailto:sage [2017/11/12(日) 15:17:55.40 ID:AbMINYSr.net] ¥
732 名前:¥ mailto:sage [2017/11/12(日) 15:18:14.20 ID:AbMINYSr.net] ¥
733 名前:¥ mailto:sage [2017/11/12(日) 15:18:34.89 ID:AbMINYSr.net] ¥
734 名前:¥ mailto:sage [2017/11/12(日) 15:18:53.70 ID:AbMINYSr.net] ¥
735 名前:¥ mailto:sage [2017/11/12(日) 15:19:14.95 ID:AbMINYSr.net] ¥
736 名前:¥ mailto:sage [2017/11/12(日) 15:19:31.76 ID:AbMINYSr.net] ¥
737 名前:¥ mailto:sage [2017/11/12(日) 15:19:48.43 ID:AbMINYSr.net] ¥
738 名前:¥ mailto:sage [2017/11/12(日) 15:20:08.70 ID:AbMINYSr.net] ¥
739 名前:¥ mailto:sage [2017/11/12(日) 15:20:28.96 ID:AbMINYSr.net] ¥
740 名前:¥ mailto:sage [2017/11/12(日) 15:20:47.75 ID:AbMINYSr.net] ¥
741 名前:132人目の素数さん mailto:sage [2017/11/12(日) 17:24:53.81 ID:cTg/FCp5.net] >>687 >Q3. [0,1]上の有理数で不連続、無理数で微分可能(当然連続)な関数を1つ示せ で、Q3の答えはまだ見つからないのかい?(ニヤリ) これ大学数学の常識なんだけどな Q3は、とある有名なテクストに載っている などと、うるさくせかす人がいる(^^
742 名前:132人目の素数さん [2017/11/12(日) 17:25:49.63 ID:MPt+syL4.net] きめえ
743 名前:132人目の素数さん mailto:sage [2017/11/12(日) 17:26:52.72 ID:cPywtfD3.net] >>723 この問題も大学数学の常識なんですが、解いていただけませんか? ある無矛盾な公理系τの任意のモデルに対してある論理式φが常に真となるならば、τからφがLKにおいて証明可能となることを示せ、という問題がわかりません
744 名前:132人目の素数さん mailto:sage [2017/11/12(日) 17:41:41.18 ID:KofdmiCa.net] 日本人を全員死刑にしろよ
745 名前:132人目の素数さん [2017/11/12(日) 17:59:16.52 ID:bcdob+HV.net] >>712 なんで?
746 名前:132人目の素数さん [2017/11/12(日) 17:59:53.48 ID:bcdob+HV.net] >>707 もうやめたら?
747 名前:132人目の素数さん [2017/11/12(日) 18:00:57.88 ID:bcdob+HV.net] >>705 つまんないよ
748 名前:132人目の素数さん mailto:sage [2017/11/12(日) 18:01:04.19 ID:cPywtfD3.net] >>727 わからないんですか(笑)?
749 名前:132人目の素数さん [2017/11/12(日) 18:03:40.04 ID:bcdob+HV.net] >>689 当たり前ということを言いたいのでは?
750 名前:132人目の素数さん [2017/11/12(日) 18:05:09.11 ID:bcdob+HV.net] >>730 わかりません
751 名前:132人目の素数さん mailto:sage [2017/11/12(日) 18:16:33.78 ID:cPywtfD3.net] >>711 Aは相対位相における連結成分Mに直和分解されます MがR^2における連結成分と一致することを示します MがR^2において連結ではないとすると、以下を満たす開集合O1,O2が存在します M=O1∪O2 O1∩O2=φ O1≠φ O2≠φ O1,O2⊂M⊂Aですから、以下が成り立ちます M=A∩(O1∪O2)=(A∩O1)∪(A∩O2) M⊃O1∩O2=A∩(O1∩O2)=(A∩O1)∩(A∩O2)=φ O1=A∩O1≠φ O2=A∩O2≠φ これは、Mが相対位相において連結であることと矛盾します
752 名前:132人目の素数さん mailto:sage [2017/11/12(日) 18:19:08.81 ID:cPywtfD3.net] 全然違いますね 待ってください
753 名前:132人目の素数さん [2017/11/12(日) 18:23:14.06 ID:bcdob+HV.net] >>733 > >>711 > Aは相対位相における連結成分Mに直和分解されます これはなんで?
754 名前:132人目の素数さん mailto:sage [2017/11/12(日) 18:23:54.54 ID:VL17AsZ4.net] >>731 必要十分に述べよって意味も分からんのか ○すぞ
755 名前:132人目の素数さん [2017/11/12(日) 18:27:25.05 ID:bcdob+HV.net] >>736 この人も触っちゃダメな人が まあ問題もアレだし 気づくべきだったか
756 名前:132人目の素数さん mailto:sage [2017/11/12(日) 18:28:36.28 ID:cPywtfD3.net] >>711 Aは、Aの相対位相における連結成分Mに直和分解されます MがR^2における連結成分と一致することを示します MがR^2において連結ではないとすると、以下を満たすR^2の開集合O1,O2が存在します M=(M∩O1)∪(M∩O2) (M∩O1)∩(M∩O2)=φ M∩O1≠φ M∩O2≠φ M⊂Aですから、以下が成り立ちます M=(M∩(A∩O1))∪(M∩(A∩O2)) (M∩(A∩O1))∩(M∩(A∩O2))=φ M∩(A∩O1)≠φ M∩(A∩O2)≠φ これは、Aの相対位相においてMが連結ではないことを意味しますが、これは仮定に反します
757 名前:132人目の素数さん mailto:sage [2017/11/12(日) 18:29:38.15 ID:cPywtfD3.net] >>735 任意の位相空間は連結成分によって直和分解されますよね?
758 名前:132人目の素数さん mailto:sage [2017/11/12(日) 18:30:30.51 ID:VL17AsZ4.net] a,bはa<bなる任意の実数 以下の関数は[a,b]を定義域とする 不連続な関数f(x)があるとき、定数関数でないある連続関数g(x)が存在し、f(x)g(x)を連続関数とできることを示せ。
759 名前:132人目の素数さん mailto:sage [2017/11/12(日) 18:33:01.22 ID:cPywtfD3.net] >>738 ×MがR^2における連結成分と一致することを示します ◯MがR^2において連結となることを示します
760 名前:132人目の素数さん mailto:sage [2017/11/12(日) 18:35:53.37 ID:C0+75xBc.net] 常微分方程式の問題で、λ=±iαのとき、なんで基本解がcosαxとsinαxになるの??どなたか教えてください
761 名前:132人目の素数さん mailto:sage [2017/11/12(日) 18:38:20.55 ID:cPywtfD3.net] >>742 e^iαx=cosαx+isinαxですから、cosとsinでてきますよね
762 名前:132人目の素数さん mailto:sage [2017/11/12(日) 18:45:12.97 ID:cPywtfD3.net] >>740 あるx=aについてg(x)≠0のとき f(a)=f(a)g(a)/g(a)は連続関数÷連続関数となっていますから、x=aにおいてf(x)は連続です 今、f(x)としてディリクレ関数を考えます f(x)は至る所で不連続です 先の対偶を考えると、 x=aにおいてf(x)は不連続→g(a)=0 ですから、任意のxについてg(x)=0とならなければなりません これは、gが定数でないことと反します よって、題意は間違いです
763 名前:132人目の素数さん mailto:sage [2017/11/12(日) 18:45:29.00 ID:C0+75xBc.net] >>743 出したあとに、どうやって基本解に持っていく
764 名前:んでしょうか? [] [ここ壊れてます]
765 名前:132人目の素数さん mailto:sage [2017/11/12(日) 18:48:22.93 ID:cPywtfD3.net] >>745 あなたの知ってる基本解、の定義はなんですか?
766 名前:132人目の素数さん mailto:sage [2017/11/12(日) 18:56:35.69 ID:C0+75xBc.net] >>746 定数係数と組み合わせると一般解になる一次独立な解です。
767 名前:132人目の素数さん mailto:sage [2017/11/12(日) 18:59:26.42 ID:cPywtfD3.net] >>747 cosαxとsinαxは方程式の解ですし、一次独立になってますね
768 名前:132人目の素数さん mailto:sage [2017/11/12(日) 19:09:17.18 ID:VL17AsZ4.net] >>744 チッ
769 名前:132人目の素数さん mailto:sage [2017/11/12(日) 19:09:26.61 ID:C0+75xBc.net] >>748 λ=±iαのとき、基本解X=e^±iαxになると思うのですが、この場合、基本解はcosα+isinαとcosα-isinαではないんでしょうか?
770 名前:132人目の素数さん mailto:sage [2017/11/12(日) 19:11:26.29 ID:cPywtfD3.net] >>750 基本解は1組だけじゃなくていっぱいあるんですよ cosα+isinαとcosα-isinαももちろん基本解です ですが、普通はより簡単にかけるものを選ぶわけですね
771 名前:132人目の素数さん mailto:sage [2017/11/12(日) 19:18:10.93 ID:C0+75xBc.net] >>751 基本解がcosαx+isinαxとcosαx-isinαxのとき、cosαxとsinαxも基本解になるということはどのようにわかるのでしょうか?ここがわかりません。
772 名前:132人目の素数さん mailto:sage [2017/11/12(日) 19:20:32.76 ID:cPywtfD3.net] >>752 y1=cosαx+isinαx y2=cosαx-isinαx とすると、重ね合わせの法則より (y1+y2)/2=cosαx、(y1-y2)/2i=sinαx も解となりますね cosとsinは一次独立ですから、これも基本解です
773 名前:132人目の素数さん mailto:sage [2017/11/12(日) 19:30:02.67 ID:C0+75xBc.net] >>753 ありがとうございます
774 名前:¥ mailto:sage [2017/11/12(日) 20:30:50.96 ID:AbMINYSr.net] ¥
775 名前:¥ mailto:sage [2017/11/12(日) 20:31:08.87 ID:AbMINYSr.net] ¥
776 名前:¥ mailto:sage [2017/11/12(日) 20:31:28.83 ID:AbMINYSr.net] ¥
777 名前:¥ mailto:sage [2017/11/12(日) 20:31:46.99 ID:AbMINYSr.net] ¥
778 名前:¥ mailto:sage [2017/11/12(日) 20:32:04.89 ID:AbMINYSr.net] ¥
779 名前:¥ mailto:sage [2017/11/12(日) 20:32:23.05 ID:AbMINYSr.net] ¥
780 名前:¥ mailto:sage [2017/11/12(日) 20:32:41.43 ID:AbMINYSr.net] ¥
781 名前:¥ mailto:sage [2017/11/12(日) 20:33:01.21 ID:AbMINYSr.net] ¥
782 名前:¥ mailto:sage [2017/11/12(日) 20:33:20.70 ID:AbMINYSr.net] ¥
783 名前:¥ mailto:sage [2017/11/12(日) 20:33:39.14 ID:AbMINYSr.net] ¥
784 名前:132人目の素数さん mailto:sage [2017/11/12(日) 21:09:12.43 ID:VL17AsZ4.net] xに関する以下の方程式が素数の解を持つように自然数nを定めよ。 ただしpは素数の定数である。 (京都大学) px^(p)-npx^(p-1)+n^(p-2)=0
785 名前:132人目の素数さん mailto:sage [2017/11/12(日) 21:29:36.50 ID:dOU8namZ.net] Askeyスキームについて学びたいのですがこれが載ってる本ってありますか?
786 名前:132人目の素数さん mailto:sage [2017/11/12(日) 23:45:56.82 ID:+jphTJpC.net] [前スレ.637] 91歳 竹内外史(1926/01/25〜2017/05/10) 90歳 Atle Selberg(1917/06/14〜2007/08/06) ? 佐藤幹夫(1928/04/18〜) 89 を追加
787 名前:132人目の素数さん [2017/11/12(日) 23:59:49.73 ID:GGaVEi9w.net] >>765 xがその素数解として n^(p-2)=(n-x)px^(p-1) x=pのときは n^(p-2)=(n-p)p^p nはp^2で割れるからn=kp^2と置くと k^(p-2)p^(2(p-2))=(kp-1)p^(p+1) k^(p-2)p^(p-5)=kp-1 p>5はあり得ないから p=5のときは k^3=5k-1 これはあり得ない p=3のときは k=9(3k-1) これもあり得ない p=2のときは 1=8(2k-1) あり得ない x≠pのときは nはpxで割れるからn=kpxと置くと k^(p-2)p^(p-2)x^(p-2)=(kp-1)px^p k^(p-2)p^(p-3)=(kp-1)x^2 p>3はあり得ないから p=3のときは k=(3k-1)x^2 あり得ない p=2のときは 1=(2k-1)px^2 あり得ない
788 名前:132人目の素数さん mailto:sage [2017/11/13(月) 00:08:48.11 ID:abgKGSaf.net] >>331 〔ゲンツェンの基本定理〕 数セミ増刊「数学100の定理」日本評論社 p.206-207 (1983)
789 名前:¥ mailto:sage [2017/11/13(月) 00:53:43.27 ID:tP2A7oah.net] ¥
790 名前:¥ mailto:sage [2017/11/13(月) 00:54:08.94 ID:tP2A7oah.net] ¥
791 名前:¥ mailto:sage [2017/11/13(月) 00:54:29.05 ID:tP2A7oah.net] ¥
792 名前:¥ mailto:sage [2017/11/13(月) 00:54:48.14 ID:tP2A7oah.net] ¥
793 名前:132人目の素数さん mailto:sage [2017/11/13(月) 00:55:03.44 ID:+DyIKn4f.net] しっかし誰も解けない難しい質問ばっかでつまんねえなぁ。 本当に「実際は解いている連中ばっか」状態になったこと一度もねえじゃんw もっと簡単な質問してこい、脳みそウンコまみれの底辺層ども。
794 名前:¥ mailto:sage [2017/11/13(月) 00:55:06.76 ID:tP2A7oah.net] ¥
795 名前:132人目の素数さん mailto:sage [2017/11/13(月) 00:55:23.63 ID:+DyIKn4f.net] しっかし誰も解けない難しい質問ばっかでつまんねえなぁ。 本当に「実際は解いている連中ばっか」状態になったこと一度もねえじゃんw もっと簡単な質問してこい、脳みそウンコまみれの底辺層ども。
796 名前:¥ mailto:sage [2017/11/13(月) 00:55:30.64 ID:tP2A7oah.net] ¥
797 名前:132人目の素数さん mailto:sage [2017/11/13(月) 00:55:38.49 ID:+DyIKn4f.net] しっかし誰も解けない難しい質問ばっかでつまんねえなぁ。 本当に「実際は解いている連中ばっか」状態になったこと一度もねえじゃんw もっと簡単な質問してこい、脳みそウンコまみれの底辺層ども。
798 名前:¥ mailto:sage [2017/11/13(月) 00:55:48.16 ID:tP2A7oah.net] ¥
799 名前:132人目の素数さん mailto:sage [2017/11/13(月) 00:55:54.67 ID:+DyIKn4f.net] しっかし誰も解けない難しい質問ばっかでつまんねえなぁ。 本当に「実際は解いている連中ばっか」状態になったこと一度もねえじゃんw もっと簡単な質問してこい、脳みそウンコまみれの底辺層ども。
800 名前:¥ mailto:sage [2017/11/13(月) 00:56:45.89 ID:tP2A7oah.net] ¥
801 名前:132人目の素数さん mailto:sage [2017/11/13(月) 00:56:53.89 ID:+DyIKn4f.net] しっかし誰も解けない難しい質問ばっかでつまんねえなぁ。 本当に「実際は解いている連中ばっか」状態になったこと一度もねえじゃんw もっと簡単な質問してこい、脳みそウンコまみれの底辺層ども。
802 名前:¥ mailto:sage [2017/11/13(月) 00:57:04.73 ID:tP2A7oah.net] ¥
803 名前:132人目の素数さん mailto:sage [2017/11/13(月) 00:57:13.03 ID:+DyIKn4f.net] しっかし誰も解けない難しい質問ばっかでつまんねえなぁ。 本当に「実際は解いている連中ばっか」状態になったこと一度もねえじゃんw もっと簡単な質問してこい、脳みそウンコまみれの底辺層ども。
804 名前:¥ mailto:sage [2017/11/13(月) 00:57:20.71 ID:tP2A7oah.net] ¥
805 名前:132人目の素数さん mailto:sage [2017/11/13(月) 00:57:28.87 ID:+DyIKn4f.net] しっかし誰も解けない難しい質問ばっかでつまんねえなぁ。 本当に「実際は解いている連中ばっか」状態になったこと一度もねえじゃんw もっと簡単な質問してこい、脳みそウンコまみれの底辺層ども。
806 名前:¥ mailto:sage [2017/11/13(月) 00:57:46.19 ID:tP2A7oah.net] ¥
807 名前:132人目の素数さん mailto:sage [2017/11/13(月) 00:57:51.02 ID:+DyIKn4f.net] しっかし誰も解けない難しい質問ばっかでつまんねえなぁ。 本当に「実際は解いている連中ばっか」状態になったこと一度もねえじゃんw もっと簡単な質問してこい、脳みそウンコまみれの底辺層ども。
808 名前:¥ mailto:sage [2017/11/13(月) 00:58:08.21 ID:tP2A7oah.net] ¥
809 名前:132人目の素数さん mailto:sage [2017/11/13(月) 00:58:22.66 ID:+DyIKn4f.net] しっかし誰も解けない難しい質問ばっかでつまんねえなぁ。 本当に「実際は解いている連中ばっか」状態になったこと一度もねえじゃんw もっと簡単な質問してこい、脳みそウンコまみれの底辺層ども。
810 名前:¥ mailto:sage [2017/11/13(月) 00:58:25.00 ID:tP2A7oah.net] ¥
811 名前:132人目の素数さん mailto:sage [2017/11/13(月) 00:58:50.36 ID:+DyIKn4f.net] しっかし誰も解けない難しい質問ばっかでつまんねえなぁ。 本当に「実際は解いている連中ばっか」状態になったこと一度もねえじゃんw もっと簡単な質問してこい、脳みそウンコまみれの底辺層ども。
812 名前:132人目の素数さん mailto:sage [2017/11/13(月) 01:14:24.65 ID:4IzD8ngY.net] fは実数上において微分可能な関数とする f(b)=0 かつ f'(a)>f(a) (∀a∈R) ならば任意のa>bに対してf(a)>0となることを示せ
813 名前:132人目の素数さん mailto:sage [2017/11/13(月) 01:18:58.30 ID:luyVs4L2.net] >>793 >>363
814 名前:132人目の素数さん mailto:sage [2017/11/13(月) 01:57:00.96 ID:abgKGSaf.net] >>793-794 g(x)= f(x)e^(-x), とおく。題意より g(b)= f(b)e^(-b)= 0, g '(x)={f '(x)- f(x)}e^(-x)> 0, よって a>b ⇒ g(a)= g(b)+∫[b,a]g '(x)dx > 0, ⇒ f(a)> 0, ぢゃね?
815 名前:132人目の素数さん mailto:sage [2017/11/13(月) 02:11:32.56 ID:VgHZBtC8.net] 同じ問題あったのですねすみません あと積分は使えないです
816 名前:132人目の素数さん mailto:sage [2017/11/13(月) 02:26:15.95 ID:+DyIKn4f.net] >>796 ある無矛盾な公理系τの任意のモデルに対してある論理式φが常に真となるならば、τからφがLKにおいて証明可能となることを示せ、という問題がわかりません
817 名前:¥ mailto:sage [2017/11/13(月) 02:28:49.73 ID:tP2A7oah.net] ¥
818 名前:¥ mailto:sage [2017/11/13(月) 02:29:09.39 ID:tP2A7oah.net] ¥
819 名前:¥ mailto:sage [2017/11/13(月) 02:29:29.72 ID:tP2A7oah.net] ¥
820 名前:¥ mailto:sage [2017/11/13(月) 02:29:48.73 ID:tP2A7oah.net] ¥
821 名前:¥ mailto:sage [2017/11/13(月) 02:30:06.04 ID:tP2A7oah.net] ¥
822 名前:¥ mailto:sage [2017/11/13(月) 02:30:24.94 ID:tP2A7oah.net] ¥
823 名前:¥ mailto:sage [2017/11/13(月) 02:30:43.71 ID:tP2A7oah.net] ¥
824 名前:¥ mailto:sage [2017/11/13(月) 02:31:03.97 ID:tP2A7oah.net] ¥
825 名前:¥ mailto:sage [2017/11/13(月) 02:31:22.48 ID:tP2A7oah.net] ¥
826 名前:¥ mailto:sage [2017/11/13(月) 02:31:41.84 ID:tP2A7oah.net] ¥
827 名前:132人目の素数さん mailto:sage [2017/11/13(月) 06:54:16.54 ID:tJWM2i9i.net] 朝から太く低い声で何度も 『完璧ではありませんが』 を聞かされた
828 名前:132人目の素数さん mailto:sage [2017/11/13(月) 07:54:25.69 ID:1qS9TkZH.net] >>711 一点は連結。
829 名前:132人目の素数さん [2017/11/13(月) 09:45:29.00 ID:7L3amaex.net] (sinx)(cosx)(tanx)=sinx+cosx+tanx を満たす実数xは存在しますか?
830 名前:132人目の素数さん mailto:sage [2017/11/13(月) 10:00:21.06 ID:qp7c7UKZ.net] >>810 とりあえずグラフプロットしたら存在するみたいだよ
831 名前:132人目の素数さん mailto:sage [2017/11/13(月) 10:08:07.81 ID:57EPRGac.net] >>711 補題: ある集合族が それぞれ連結でありかつ一点を共有するとき、その和集合は連結である。(証明略) 写像 f: A → P を次のように構成する。(※ PはAの冪集合) x∈A に対して xを含む連結集合の全てを考える。f(x) はその集合族の和集合とする。 一点集合 {x} は連結なので。f(x) は常に空集合ではない。補題よりそれは連結である。 x ∈ f(x) なので A = ∪f(x) は明らか。 y ∈ f(x) の時、 f(y)の定義より f(x) ⊂ f(y) 、よって x ∈ f(y)、f(x)の定義より f(y) ⊂ f(x) つまり f(x) = f(y) Aの同値関係Rを x〜y ⇔ f(x)=f(y) で定義する。(反射/対称/推移律が成り立つ) 同値類別(商集合) A/R が「互いに共通部部分をもたない連結部分集合」による分割を与える。
832 名前:132人目の素数さん mailto:da [2017/11/13(月) 10:16:06.74 ID:57EPRGac.net] >>711 A=∪{x} も「互いに共通部部分をもたない連結部分集合」による分割になってる。(>>809 ) A/Rは「連結成分」による分割って事ですね。
833 名前:132人目の素数さん mailto:sage [2017/11/13(月) 10:53:39.72 ID:abgKGSaf.net] >>793-794 g(x)= f(x)e^(-x), とおく。題意より g(b)= 0, g '(x)> 0, h(x) = g(x) - g(a)(x-b)/(a-b) とおく。題意より h(a)= h(b)= 0, また h(x)は微分可能。 ロルの定理より >>382 h '(c)= 0, b<c<a となるcがある。 ∴ g(a)= g(b)+ g '(c)(c-b)/(a-b)> 0, ∴ f(a)> 0, ぢゃね?
834 名前:132人目の素数さん mailto:sage [2017/11/13(月) 11:36:35.40 ID:abgKGSaf.net] >>810 とりあえず近似値は x = 2nπ - 2.0060382594506053650 x = 2nπ - 0.38657112396120291534 みたいだよ
835 名前:132人目の素数さん [2017/11/13(月) 12:17:13.60 ID:7L3amaex.net] >>811 >>815 グラフによると確かに存在し、近似値をそうなりそうです f(x)=sinxcosxtanx-sinx-cosx-tanx っておいて、微分して単調減少を示し、π/6とか5π/6を代入でいけますかね? でも導関数の符号を調べるのがうまくいかないんですよ いいやり方ないですかね
836 名前:132人目の素数さん mailto:sage [2017/11/13(月) 12:20:27.68 ID:gEvQf1sY.net] >>816 f(0)
837 名前:=-1、f(π)=1ですから、中間値の定理より存在しますね [] [ここ壊れてます]
838 名前:132人目の素数さん mailto:sage [2017/11/13(月) 12:23:56.97 ID:YMnEmf+F.net] >>816 (以下x省略) sin・cos・tan = sin+cos+tan 左辺=sin^2 より sin^2 -sin-cos = tan -π/2<x<π/2に対してy=tanは単調増加 y=sin^2-sin-cosと交点を最低でも1つ持つことを示せば解の存在を言える
839 名前:132人目の素数さん [2017/11/13(月) 12:28:50.83 ID:Lvg//uKF.net] >>817 それだとπ/2で定義されてないとダメでは? 範囲絞ればいいけど
840 名前:132人目の素数さん mailto:sage [2017/11/13(月) 12:50:54.02 ID:tJWM2i9i.net] 舐めた放送をして俺をコケにするのもいい加減にしろよ。 糞NHK、ふざけんな。 一国民を小馬鹿にしたDQN野郎を解雇しろ!
841 名前:132人目の素数さん [2017/11/13(月) 13:06:47.88 ID:tJWM2i9i.net] 私が画面を見ただけで無理とは何事だ。 ふざけんのもいい加減にしろ。 手荒な安否確認か?答えろ、ゴミ!
842 名前:132人目の素数さん [2017/11/13(月) 13:23:09.83 ID:tJWM2i9i.net] 外からワンパターンの「残念でした。」 が聞こえてきましたが、 小学校低学年レベルの日本語能力の糞ガキは 「何が」残念なのか言えるようになってからその言葉を 使いましょうね。
843 名前:132人目の素数さん mailto:sage [2017/11/13(月) 13:31:59.20 ID:qnEbkspS.net] >>822 50歳のホームレスさんですか?
844 名前:132人目の素数さん [2017/11/13(月) 13:35:18.96 ID:tJWM2i9i.net] >>823 いいえ、日本最高峰プログラマーです。 5億、7億、13億、17億と威勢のいい声が聞こえてきますが 誰が何時払うのでしょうか? 当然、期待して待っているわけではありませんが。
845 名前:132人目の素数さん [2017/11/13(月) 14:30:48.53 ID:zSJ2QXAT.net] >>819 arctan咬ませばいいだけ
846 名前:¥ mailto:sage [2017/11/13(月) 15:29:18.17 ID:tP2A7oah.net] ¥
847 名前:¥ mailto:sage [2017/11/13(月) 15:29:34.95 ID:tP2A7oah.net] ¥
848 名前:¥ mailto:sage [2017/11/13(月) 15:29:56.04 ID:tP2A7oah.net] ¥
849 名前:¥ mailto:sage [2017/11/13(月) 15:30:15.51 ID:tP2A7oah.net] ¥
850 名前:¥ mailto:sage [2017/11/13(月) 15:30:33.93 ID:tP2A7oah.net] ¥
851 名前:¥ mailto:sage [2017/11/13(月) 15:30:50.26 ID:tP2A7oah.net] ¥
852 名前:¥ mailto:sage [2017/11/13(月) 15:31:06.85 ID:tP2A7oah.net] ¥
853 名前:¥ mailto:sage [2017/11/13(月) 15:31:24.74 ID:tP2A7oah.net] ¥
854 名前:¥ mailto:sage [2017/11/13(月) 15:31:44.06 ID:tP2A7oah.net] ¥
855 名前:¥ mailto:sage [2017/11/13(月) 15:32:02.66 ID:tP2A7oah.net] ¥
856 名前:132人目の素数さん [2017/11/13(月) 17:01:48.28 ID:DFR7Y7G7.net] 低レベルで、disgustingな言動は不要だ。 頭がおかしいんじゃないのか? いきなり、「無理、無理。」 何が言いたいのかな、おぼっちゃんは?
857 名前:132人目の素数さん [2017/11/13(月) 20:00:08.08 ID:0P2oPhMC.net] Σ【k=1 →∞】1/(t+k)^2 = ∫【0→1】(x^p/1-x) log(1/x) これの示し方を教えて下さいm(_ _)m
858 名前:¥ mailto:sage [2017/11/13(月) 20:33:59.56 ID:tP2A7oah.net] ¥
859 名前:¥ mailto:sage [2017/11/13(月) 20:34:18.10 ID:tP2A7oah.net] ¥
860 名前:¥ mailto:sage [2017/11/13(月) 20:34:35.10 ID:tP2A7oah.net] ¥
861 名前:¥ mailto:sage [2017/11/13(月) 20:34:54.76 ID:tP2A7oah.net] ¥
862 名前:¥ mailto:sage [2017/11/13(月) 20:35:13.27 ID:tP2A7oah.net] ¥
863 名前:¥ mailto:sage [2017/11/13(月) 20:35:34.77 ID:tP2A7oah.net] ¥
864 名前:¥ mailto:sage [2017/11/13(月) 20:35:54.25 ID:tP2A7oah.net] ¥
865 名前:¥ mailto:sage [2017/11/13(月) 20:36:11.64 ID:tP2A7oah.net] ¥
866 名前:¥ mailto:sage [2017/11/13(月) 20:36:28.92 ID:tP2A7oah.net] ¥
867 名前:¥ mailto:sage [2017/11/13(月) 20:36:46.96 ID:tP2A7oah.net] ¥
868 名前:132人目の素数さん mailto:sage [2017/11/13(月) 21:08:04.77 ID:Cl5WReNg.net] (1)log2(3)は無理数であることを示せ。 (2)log2(3)=p√2 となる有理数pは存在しないことを示せ。 (2)が分かりません。(1)がヒントとも思えないのですが…
869 名前:¥ mailto:sage [2017/11/13(月) 21:30:15.49 ID:tP2A7oah.net] ¥
870 名前:¥ mailto:sage [2017/11/13(月) 21:30:37.72 ID:tP2A7oah.net] ¥
871 名前:¥ mailto:sage [2017/11/13(月) 21:30:56.55 ID:tP2A7oah.net] ¥
872 名前:¥ mailto:sage [2017/11/13(月) 21:31:16.52 ID:tP2A7oah.net] ¥
873 名前:¥ mailto:sage [2017/11/13(月) 21:31:38.43 ID:tP2A7oah.net] ¥
874 名前:¥ mailto:sage [2017/11/13(月) 21:31:59.81 ID:tP2A7oah.net] ¥
875 名前:¥ mailto:sage [2017/11/13(月) 21:32:21.66 ID:tP2A7oah.net] ¥
876 名前:¥ mailto:sage [2017/11/13(月) 21:32:47.55 ID:tP2A7oah.net] ¥
877 名前:¥ mailto:sage [2017/11/13(月) 21:33:12.32 ID:tP2A7oah.net] ¥
878 名前:¥ mailto:sage [2017/11/13(月) 21:33:36.50 ID:tP2A7oah.net] ¥
879 名前:132人目の素数さん mailto:ddd [2017/11/13(月) 21:36:11.45 ID:AxkWqEZx.net] (1) 2^x= 3 Suppose x= m/m such that m,n are integers. (2^(m/n))^n = 2^m = 3^n This is impossible, so x is not rational number. (2) With the same procedure that p=m/n, i.e. x= (m/n)Sqrt[2] 2^((m/n)Sqrt[2]) = (2~Sqrt[2])^(m/n)=2^(m/(2n))= 3 this me
880 名前:ans 2^m=3^(2 n) This is contradictory. So p is not rational. [] [ここ壊れてます]
881 名前:132人目の素数さん [2017/11/13(月) 22:50:00.97 ID:K/FirY/9.net] >>837 p?t?
882 名前:132人目の素数さん [2017/11/13(月) 23:06:48.43 ID:fkSmvI97.net] 自殺したい。
883 名前:¥ mailto:sage [2017/11/13(月) 23:23:57.02 ID:tP2A7oah.net] ¥
884 名前:¥ mailto:sage [2017/11/13(月) 23:24:16.20 ID:tP2A7oah.net] ¥
885 名前:¥ mailto:sage [2017/11/13(月) 23:24:37.39 ID:tP2A7oah.net] ¥
886 名前:¥ mailto:sage [2017/11/13(月) 23:24:59.28 ID:tP2A7oah.net] ¥
887 名前:¥ mailto:sage [2017/11/13(月) 23:25:17.36 ID:tP2A7oah.net] ¥
888 名前:¥ mailto:sage [2017/11/13(月) 23:25:34.62 ID:tP2A7oah.net] ¥
889 名前:¥ mailto:sage [2017/11/13(月) 23:25:53.28 ID:tP2A7oah.net] ¥
890 名前:¥ mailto:sage [2017/11/13(月) 23:26:10.27 ID:tP2A7oah.net] ¥
891 名前:¥ mailto:sage [2017/11/13(月) 23:26:27.64 ID:tP2A7oah.net] ¥
892 名前:132人目の素数さん mailto:sage [2017/11/13(月) 23:26:46.58 ID:fkSmvI97.net] 自殺したい。
893 名前:¥ mailto:sage [2017/11/13(月) 23:26:46.58 ID:tP2A7oah.net] ¥
894 名前:¥ mailto:sage [2017/11/13(月) 23:28:21.85 ID:tP2A7oah.net] ¥
895 名前:¥ mailto:sage [2017/11/13(月) 23:28:38.73 ID:tP2A7oah.net] ¥
896 名前:¥ mailto:sage [2017/11/13(月) 23:28:55.10 ID:tP2A7oah.net] ¥
897 名前:¥ mailto:sage [2017/11/13(月) 23:29:13.15 ID:tP2A7oah.net] ¥
898 名前:132人目の素数さん mailto:sage [2017/11/13(月) 23:29:15.21 ID:fkSmvI97.net] 自殺したい。
899 名前:¥ mailto:sage [2017/11/13(月) 23:29:30.08 ID:tP2A7oah.net] ¥
900 名前:¥ mailto:sage [2017/11/13(月) 23:29:47.37 ID:tP2A7oah.net] ¥
901 名前:¥ mailto:sage [2017/11/13(月) 23:30:03.43 ID:tP2A7oah.net] ¥
902 名前:¥ mailto:sage [2017/11/13(月) 23:30:21.75 ID:tP2A7oah.net] ¥
903 名前:¥ mailto:sage [2017/11/13(月) 23:31:05.12 ID:tP2A7oah.net] ¥
904 名前:132人目の素数さん mailto:sage [2017/11/13(月) 23:31:18.88 ID:+DyIKn4f.net] 自殺したい
905 名前:¥ mailto:sage [2017/11/13(月) 23:31:27.68 ID:tP2A7oah.net] ¥
906 名前:¥ mailto:sage [2017/11/13(月) 23:31:47.52 ID:tP2A7oah.net] ¥
907 名前:¥ mailto:sage [2017/11/13(月) 23:32:05.17 ID:tP2A7oah.net] ¥
908 名前:¥ mailto:sage [2017/11/13(月) 23:32:21.41 ID:tP2A7oah.net] ¥
909 名前:132人目の素数さん mailto:sage [2017/11/13(月) 23:57:32.66 ID:qZZBzQyT.net] <再投稿> >>687 Q3. [0,1]上の有理数で不連続、無理数で微分可能(当然連続)な関数を1つ示せ これ、なんか、難しい問題なんかね? はて?
910 名前:132人目の素数さん mailto:sage [2017/11/13(月) 23:59:10.75 ID:+DyIKn4f.net] >>888 ある無矛盾な公理系τの任意のモデルに対してある論理式φが常に真となるならば、τからφがLKにおいて証明可能となることを示せ、という問題がわかりません
911 名前:132人目の素数さん mailto:sage [2017/11/14(火) 03:33:19.09 ID:8GRq7eAg.net] [前スレ.637] 91歳 50日 吉田洋一(1898/07/11〜1989/08/30) を追加
912 名前:¥ mailto:sage [2017/11/14(火) 03:48:28.31 ID:DKMYn3HH.net] ¥
913 名前:¥ mailto:sage [2017/11/14(火) 03:48:45.02 ID:DKMYn3HH.net] ¥
914 名前:¥ mailto:sage [2017/11/14(火) 03:49:00.06 ID:DKMYn3HH.net] ¥
915 名前:¥ mailto:sage [2017/11/14(火) 03:49:17.63 ID:DKMYn3HH.net] ¥
916 名前:¥ mailto:sage [2017/11/14(火) 03:49:38.96 ID:DKMYn3HH.net] ¥
917 名前:¥ mailto:sage [2017/11/14(火) 03:49:55.91 ID:DKMYn3HH.net] ¥
918 名前:¥ mailto:sage [2017/11/14(火) 03:50:14.35 ID:DKMYn3HH.net] ¥
919 名前:¥ mailto:sage [2017/11/14(火) 03:50:32.72 ID:DKMYn3HH.net] ¥
920 名前:¥ mailto:sage [2017/11/14(火) 03:50:50.33 ID:DKMYn3HH.net] ¥
921 名前:¥ mailto:sage [2017/11/14(火) 03:51:08.58 ID:DKMYn3HH.net] ¥
922 名前:132人目の素数さん mailto:sage [2017/11/14(火) 04:23:02.57 ID:8GRq7eAg.net] [前スレ.637] 93歳 64日 伊藤 清(1915/09/07〜2008/11/10)確率微分方程式 [前スレ.643] 98歳 吉田秀和 (1913/09/23〜2012/05/22)音楽評論家、随筆家。 93歳 鈴木清順 (1923/05/24〜2017/02/13)映画監督、俳優。 を追加
923 名前:132人目の素数さん mailto:sage [2017/11/14(火) 04:40:25.17 ID:8GRq7eAg.net] [前スレ.643] 100歳 3世 井上八千代(1838/02/01〜1938/09/07)京舞 98歳 4世 井上八千代(1905/05/14〜2004/03/19)京舞 97歳 2世 井上八千代(1770〜1868/03/24)京舞 を追加。
924 名前:¥ mailto:sage [2017/11/14(火) 05:08:27.98 ID:DKMYn3HH.net] ¥
925 名前:¥ mailto:sage [2017/11/14(火) 05:08:46.60 ID:DKMYn3HH.net] ¥
926 名前:¥ mailto:sage [2017/11/14(火) 05:09:03.04 ID:DKMYn3HH.net] ¥
927 名前:¥ mailto:sage [2017/11/14(火) 05:09:24.27 ID:DKMYn3HH.net] ¥
928 名前:¥ mailto:sage [2017/11/14(火) 05:09:43.75 ID:DKMYn3HH.net] ¥
929 名前:¥ mailto:sage [2017/11/14(火) 05:10:02.35 ID:DKMYn3HH.net] ¥
930 名前:¥ mailto:sage [2017/11/14(火) 05:10:21.44 ID:DKMYn3HH.net] ¥
931 名前:¥ mailto:sage [2017/11/14(火) 05:10:42.79 ID:DKMYn3HH.net] ¥
932 名前:¥ mailto:sage [2017/11/14(火) 05:11:05.44 ID:DKMYn3HH.net] ¥
933 名前:¥ mailto:sage [2017/11/14(火) 05:11:22.58 ID:DKMYn3HH.net] ¥
934 名前:132人目の素数さん mailto:sage [2017/11/14(火) 07:13:44.10 ID:agSxZaXK.net] >>888 <転載> 146 返信:132人目の素数さん[sage] 投稿日:2017/11/14(火) 06:31:02.20 ID:IDi6PSmH [1/2] >>142 なんだ、結局分からないんだw ところで >>75 >Q3. [0,1]上の有理数で不連続、無理数で微分可能(当然連続)な関数を1つ示せ >>77 >Q3は、とある有名なテクストに載っている ハイラー、ヴァンナーの「解析教程」下に 有理数rが既約分数p/qで表されるとき、1/q^2 無理数か整数で0 という関数がx=0(より一般にはxが整数のとき)で微分可能 という証明が出ているが、無理数の箇所については言及してない
935 名前:132人目の素数さん mailto:sage [2017/11/14(火) 07:54:43.69 ID:waTphX1J.net] >>913 無理数でも微分可能っぽいけどな
936 名前:132人目の素数さん [2017/11/14(火) 12:21:44.92 ID:5GwueLvD.net] 区間 [a, b) と R は同相でないことの証明ですが、 [a, b) から1点 a を除いた集合は連結 一方、 R から1点を除いた集合は非連結 という証明があります。 [a, b) から R への同相写像が存在するとし、それを f をおく。 (a, b) と R - {f(a)} は同じ構造をしているから一方が連結であれば他方も連結である。 (a, b) は連結である。 R - {f(a)} は非連結である。 これは矛盾。 ということでしょうが、同じ構造というのはどういうことでしょうか?
937 名前:132人目の素数さん [2017/11/14(火) 12:35:15.89 ID:5GwueLvD.net] (a, b) と R - {f(a)} も同相である。 R - {f(a)} は連結でないから、 R - {f(a)} の空でない共通部分をもたない開集合 O1, O2 により、 R - {f(a)} = O1 ∪ O2 とかける。 f^(-1)(O1 ∪ O2) = f^(-1)(O1) ∪ f^(-1)(O2) f は連続だから、 f^(-1)(O1) f^(-1)(O2) は (a, b) の空でない開集合である。 これは矛盾。
938 名前:132人目の素数さん mailto:sage [2017/11/14(火) 12:53:07.20 ID:K8vpabbC.net] >>915 同相であるとは、それぞれの集合の各元と各開集合の間に一対一対応をつけることができることを言います
939 名前:132人目の素数さん mailto:sage [2017/11/14(火) 13:00:09.14 ID:Xl0QoX6y.net] >>916 (a, b) が連結も証明しなきゃ
940 名前:132人目の素数さん mailto:sage [2017/11/14(火) 13:10:00.05 ID:5GwueLvD.net] R と R^2 は同相でないことを証明せよ。 証明してください。
941 名前:132人目の素数さん mailto:sage [2017/11/14(火) 13:34:41.89 ID:aH7mb9a4.net] >>917 同相(位相同型) は 一対一だけじゃなくて 連続かつ逆写像も連続である写像が存在することをいいます。
942 名前:132人目の素数さん mailto:sage [2017/11/14(火) 13:35:50.96 ID:K8vpabbC.net] >>920 その条件は、元だけでなく開集合も一対一対応する、と言い換えることができます
943 名前:132人目の素数さん mailto:sage [2017/11/14(火) 13:43:46.54 ID:aH7mb9a4.net] >>919 同相写像 f: R^2 → R が存在すると仮定する。 R^2-{(0.0)} は f により R- f(0,0) = (-∞,f(0,0)) ∪ (f(0,0),+∞) に写される。 前者は連結(弧状連結なのは明らか)、後者は連結ではない。 連結性は連続写像で不変であるのでこれは矛盾である。
944 名前:132人目の素数さん mailto:sage [2017/11/14(火) 13:46:07.71 ID:aH7mb9a4.net] >>921 そうですが そもそも「それぞれの集合の各元と各開集合の間に一対一対応」 って何かおかしな記述でしたね。
945 名前:132人目の素数さん mailto:sage [2017/11/14(火) 14:11:24.55 ID:0VJm9RHG.net] ttp://www.geocities.jp/masuokun_2004/math/money/index.html 個人サイトをリンクして良いものかわからないのですが、前に大阪大学の類題(砂田赤チャート) を質問した者です。 ここのページの [B] 5 円玉以下に両替するとき 10 × n 円を,5 円玉のみを使って支払う方法は,10 × n ÷ 5 = 2n これと [A] より,10 × n 円を,5 円玉以下に両替する方法は 2n + 1 通りです。 なんですが、これだと五円玉と1円玉が両方混在する時の場合の数が含まれないと思うのですが、 これで正しいのでしょうか? リンク先の方はかなりの数学達者な方のようなので私のほうが間違えている可能性が高いと思うのですが、 疑問に思ったので質問させていただきます。
946 名前:132人目の素数さん [2017/11/14(火) 14:27:13.55 ID:GD1DjxVU.net] >>924 疑問に思ったらまず実験
947 名前:132人目の素数さん mailto:sage [2017/11/14(火) 14:31:01.05 ID:aH7mb9a4.net] >>924 10 × n 円に対して,5 円玉の使える余地は最低 0枚から最大 2n 枚って事だね。 5 円玉使った以外の金額は 1円玉で埋め尽くせってだけの話。 だから 2n+1 通りになる。 (結論: 日本語のおかしい個人サイトを参考に勉強するのが悪い)
948 名前:132人目の素数さん mailto:sage [2017/11/14(火) 14:32:50.98 ID:QC9cj7qM.net] >>924 結果は合ってるけど日本語がおかしい 5円玉のみ使ったら1通りしかない
949 名前:924 mailto:sage [2017/11/14(火) 15:38:16.60 ID:0VJm9RHG.net] 皆さんレスありがとうございます。日本語がおかしいだけですか。
950 名前:924 mailto:sage [2017/11/14(火) 16:02:47.08 ID:0VJm9RHG.net] よくわからないのは[D]なんです。 [C]の(n+1)~2通りというのまではいいんですが、(赤チャートや私の計算と同じです) 50円を使える硬貨として追加した時、 (n+1)~2のnに代入するのは5n-5i (iは50円玉の枚数i=1,2,3.....n)ではないのですか? 砂田赤チャート流の答えとこの方の答えが同じにならないのですが。 (赤チャートの方は10円50円100円500円で総額3000円で使う硬貨に違いがありますが)
951 名前:924 mailto:sage [2017/11/14(火) 16:07:39.58 ID:0VJm9RHG.net] 1円、5円の場合があるこのリンク先の問題のほうが場合の数が大きくなるのは当然なのかもしれませんが、 それならば
952 名前:なぜ(n+1)~2が赤チャートの場合とこの場合の両方に現れるのかわからないのです。 [] [ここ壊れてます]
953 名前:132人目の素数さん mailto:sage [2017/11/14(火) 16:12:31.51 ID:32YNxmqA.net] p,qを有理数とする(0も有理数とする)。 放物線y=x^2+px+q上にちょうどk個(kは非負整数)の格子点があるとする。 p,qが色々と変わるとき、kの取りうる値をすべて求めよ。
954 名前:132人目の素数さん mailto:sage [2017/11/14(火) 16:17:46.84 ID:0VJm9RHG.net] 砂田版赤チャートの問題と解説です。 問 10円玉、50円玉、100円玉、500円玉を組合わせて合計3000円のするには何通りの方法があるか? 答(略解) {1}10円玉と50円玉で、50*n円(nは自然数)とするには、50円玉をi個(i=0,1,2......,n)とすると、、10円玉は5(n-i)個と決 まるから、(n+1)通り {2}10円玉、50円玉、100円玉で、100:n円(nは自然数)にするには、100円玉をi個(i=0,1,....,n)とすると、残りは100(n-i), すなわち50(2n-2i)円。 10円玉と50円玉の組み合わせは{1}により(2n-2i+1)通り。したがって (2n+1-2*0)+(2n+1-2*1)+...............+(2n+1-2*n) =(2n+1)*(n+1)-2*1/2n(n+1)=(n+1)~2(通り) [3}10円玉、50円玉、100円玉、500円玉で3000円とする。500円玉がk個(K=0,1,......,6)とすると、 残りは100(30-5k)円である。10円玉、50円玉、100円玉の組み合わせは[2]により(30-5k+1)通り。(31-5k)~2=961-310k+25k~2であるから、 961*7-310(0+1+........+6)+25(0~2+1~2+.......+6~2)=6727-310*21+25*91=2492(通り) です。 こっちの解答の方は一見理解しやすいのですが、前述の通りリンク先の類題(1円、5円が加わって500円がなくて総額が100n円なバージョン)の[d]がよくわかりません。ご教示ください。
955 名前:132人目の素数さん mailto:sage [2017/11/14(火) 17:00:18.38 ID:mF5uWx0A.net] >>929 >(n+1)~2のnに代入するのは5n-5i (iは50円玉の枚数i=1,2,3.....n)ではないのですか? nに代入するの意味がわかりません >>932 の問題で言えば、3000円=100(円)×30の30に相当する部分です 932で言えば、3000円で両替の通りを考えている時に、突然、n=30の3000円でなくn=20を代入にして2000円の両替を考え出すようなものです
956 名前:132人目の素数さん mailto:sage [2017/11/14(火) 17:03:34.23 ID:mF5uWx0A.net] ちなみにDにおけるkは、50円玉の枚数ではなく、50円玉にせずに10円以下だけで両替する50円のくくりの数でしょう 両替して50円玉にする数は、kではなく2n-k
957 名前:929 mailto:sage [2017/11/14(火) 17:54:53.07 ID:0VJm9RHG.net] >>933 代入という言葉は適切ではなかったかもしれません。 赤チャートの解答を参考に、 1円 5円の組み合わせはn+1通り 1円 5円 10円の組み合わせは(n+1)~2通り 1円 5円 10円 50円の組み合わせは??? ???のところは50(n-i)(iは50円玉の枚数i=1.2......n)で10(5n-5i)となり、 Σ[i=0]-n(5n-5i+1)~2になると思うのですが、何か私は根本的に勘違いしているような気がします・・・
958 名前:132人目の素数さん mailto:age [2017/11/14(火) 18:15:25.38 ID:pIGnxfeb.net] 地方のチンピラ(公務員風情のゴミ)は防災無線のマイクを使って つまらない個人攻撃を繰り返しています。 鹿児島県はこれを放置しておいていいのでしょうか? 鹿児島県警には迷惑野郎の捜査をお願いしたいと思います。 どうぞ、よろしくお願いいたします。
959 名前:132人目の素数さん mailto:age [2017/11/14(火) 18:16:43.06 ID:pIGnxfeb.net] 誹謗中傷はしょうっちゅう外から聞こえてきますが。 マイクを使ったものは、初めてです。
960 名前:132人目の素数さん mailto:sage [2017/11/14(火) 18:17:46.76 ID:kvt/80v9.net] >>937 あなたは本当に50歳のホームレスではないんですか?
961 名前:132人目の素数さん mailto:age [2017/11/14(火) 18:33:12.44 ID:pIGnxfeb.net] 安倍政権もこんな嫌がらせをしていることを放置している あるいは許可している あるいは率先してやらせているなんてことになったら 支持率が下がるのではないのでしょうか? いいんですか、それで。
962 名前:132人目の素数さん [2017/11/14(火) 18:52:42.01 ID:5GwueLvD.net] [D] 100*n - 50*i = 10*(10*n - 5*i) 両替に50円玉が i 枚含まれる両替の仕方の数は、 [C]より [(10*n - 5*i) + 1]^2 通り Σ [(10*n - 5*i) + 1]^2 from i = 0 to i = 2*n = Σ [(10*n - 5*i) + 1]^2 from i = 0 to i = 2*n = Σ [5*(2*n - i) + 1]^2 from i = 0 to i = 2*n = Σ [5*i + 1]^2 from i = 0 to i = 2*n
963 名前:132人目の素数さん [2017/11/14(火) 18:58:23.96 ID:GD1DjxVU.net] 1円と5円の組でnを表す総数a(n)は 10円と50円で10nを表す総数と同じ xの中で10円と50円で表してる部分を10yとしたら xを1から50円で表す総数は Σ[y=0,x/10]a(y)a(x-10y) 100円と500円も使うと Σ[z=0,x/100][y=0,x/10-10z]a(z)a(y)a(x-100z-10y)
964 名前:132人目の素数さん mailto:age [2017/11/14(火) 19:21:10.39 ID:pIGnxfeb.net] それから、「とどきませんでした。」 と何度も外から聞こえてくるのですが 私が一度たりとも、「届けてくれ。」 なりの言葉を発したことがあるのでしょうか? そう言っているうるさい連中の頭はおかしいのではないのでしょうか。 私に対する嫌がらせで外から、聞こえてくるのは 要約すると 「天皇陛下(今上)を馬鹿にしやがって。」 「アメリカを馬鹿にしやがって。」 「マイクロを馬鹿にしやがって。」 「自民党の政治家を馬鹿にしやがって。」 「毎日定時で帰りやがって。」 「先輩を無視しやがって。」 「しはく(意味不明)はごみ。」 「トランプがお前に挨拶をするわけがないだろう。」 など、荒唐無稽なチンピラの誹謗が繰り返されています。 夜中の3時4時に絶叫する人間も現れます。 チンピラのみなさまにおかれましては以後、ド田舎であっても 騒音被害で大迷惑の極みですので チンピラ発言を叫ばないようにしていただきたく思います。 どうぞ、よろしくお願いいたします。
965 名前:132人目の素数さん mailto:sage [2017/11/14(火) 19:49:25.04 ID:mF5uWx0A.net] >>935 どこでどういう勘違いしてるのかわからないので、確実に間違ってるとこだけ指摘するね >(iは50円玉の枚数i=1.2......n) 50円玉は0〜2n個までね nまでじゃない
966 名前:132人目の素数さん mailto:sage [2017/11/14(火) 19:55:09.73 ID:0VJm9RHG.net] >>940 詳細な解説ありがとうございます。 100*n-50*i=10(10*n-5*i)が 50*nとならず、100*nとなるのはなぜなのでしょうか? 質問ばかりですみません。 >>941 そちらの解説は私の頭では理解できませんでした。 せっかく書いていただいたのに申し訳ない。
967 名前:132人目の素数さん mailto:sage [2017/11/14(火) 19:57:34.64 ID:0VJm9RHG.net] >>943 10*nまでなので2nなのですね。それはわかったのですが、 なぜ100*nまでなのかがわかりません。 50*nではまずい理由がわからないのです。
968 名前:132人目の素数さん mailto:sage [2017/11/14(火) 20:17:51.49 ID:mF5uWx0A.net] >>945 ・・・サイトも読み直した方がいいぞ 馬鹿にしてるつもりはないが、誤解ないように丁寧に書くね 100*n円を50円の区切り(50円玉ではない)で何分割できるかを考える それは100*n/50で求まり、2n 2nはここからきてること 次に50円玉の数を考える これは50円の区切りから、50円玉にする個数なので、これをiとおくと0〜2nまで値をとる 次に50円玉にしない50円の区切り、つまり更に10円玉、5円玉、1円玉で両替する区切りの数を考える これをkとおくと、50円玉と同じように0〜2nまで値をとる *k=2n‐iとも表記できるが、どうでもいい
969 名前:945 mailto:sage [2017/11/14(火) 20:26:42.84 ID:0VJm9RHG.net] >>946 馬鹿なのでバカにされても何も思いませんが、 50円の区切りだったんですか。赤チャの解答には全くそんなこと書いてなかったので 気が付かなかったです。もうちょっと自分で考えてみます。 >>940 氏の解説だと赤チャの説明に近くてわかりやすいんですが初っ端の 100*n - 50*i = 10*(10*n - 5*i) がわかりません。これがk=2n-iというものなんですか。 これまで50(2n-2i)のように2n-2iだったのになぜ[D]から2n-iになってるんでしょうか?
970 名前:132人目の素数さん mailto:sage [2017/11/14(火) 20:27:06.11 ID:mF5uWx0A.net] 「50円玉にしない50円の区切り」を、10円玉、5円玉、1円玉で両替する通りを考える これは、自明だが50円玉を10円玉、5円玉、1円玉で両替する通りと同じである わかりやすくするために、「50円玉にしない50円の区切り」を「その後、10円玉、5円玉、1円玉で両替する50円玉」とする 略して「両替予約済み50円玉」 サイトの解放のアプローチはこう A 「両替予約済み50円玉」の個数を変数kとする B kに応じた10円玉、5円玉、1円玉で両替する通りを求める C AとBを総和で表記する D k=0、つまりのちに両替をする50円玉が0個の場合は例外で処理(全てが50円玉の1通り) Bについては、前問で求めれている また、100円玉、500百円玉と増えていっても、同じようなアプローチで取り組める
971 名前:132人目の素数さん mailto:sage [2017/11/14(火) 20:35:08.76 ID:xcb1oScZ.net] https://www.sci.hokudai.ac.jp/~inaz/lecture/butsurisuugaku2/html/model/node24.html これの式5.47が直ちに解けると書いてあるのですが、どうやって解くんでしょうか?
972 名前:132人目の素数さん [2017/11/14(火) 20:37:13.34 ID:5GwueLvD.net] >>922 解答、ありがとうございました。
973 名前:132人目の素数さん mailto:sage [2017/11/14(火) 20:38:12.47 ID:kvt/80v9.net] >>949 なんでもクソもないですよね 微分したら元に戻る、一番簡単な微分方程式ですよね dy/dx+◯y=0 こういうの解いたことないですか?
974 名前:132人目の素数さん mailto:sage [2017/11/14(火) 20:43:38.45 ID:mF5uWx0A.net] Aで「両替予約済み50円玉」の個数を基準にするのではなく、「両替をしない最終確定50円玉」の個数を基準にするのも可能 互いに2nから引けばその個数になるので 個人的には、「両替予約済み50円玉」を選ぶ→それを両替する、という思考のがやりやすいが 最初に「両替をしない最終確定50円玉」の個数を確定させる方が自然な人もいるかな どちらを基準にしても最後の結果は同じだけど、どちらでやってるかは意識すべき
975 名前:945 mailto:sage [2017/11/14(火) 20:43:42.51 ID:0VJm9RHG.net] >>948 どうも解説ありがとうございます。もう一度いちから考え直してみます。
976 名前:132人目の素数さん mailto:sage [2017/11/14(火) 20:44:58.75 ID:xcb1oScZ.net] >>951 ありがとうございます
977 名前:132人目の素数さん [2017/11/14(火) 21:46:57.65 ID:5GwueLvD.net] [E] 100*n 円を1円玉と5円玉と10円玉と50円玉と100円玉のみを使って両替する仕方の数を 100円玉を使用する枚数によって場合分けする: (Case 0) 0 枚の100円玉を使う場合に 100*n 円を1円玉と5円玉と10円玉と50円玉と100円玉のみを使って両替する仕方の数は、 100*n - 100*0 = 100*(n - 0) 円を1円玉と5円玉と10円玉と50円玉のみを使って両替する仕方の数に等しい。 (Case 1) 1 枚の100円玉を使う場合に 100*n 円を1円玉と5円玉と10円玉と50円玉と100円玉のみを使って両替する仕方の数は、 100*n - 100*1 = 100*(n - 1) 円を1円玉と5円玉と10円玉と50円玉のみを使って両替する仕方の数に等しい。 … (Case i) i 枚の100円玉を使う場合に 100*n 円を1円玉と5円玉と10円玉と50円玉と100円玉のみを使って両替する仕方の数は、 100*n - 100*i = 100*(n - i) 円を1円玉と5円玉と10円玉と50円玉のみを使って両替する仕方の数に等しい。 … (Case n) n 枚の100円玉を使う場合に 100*n 円を1円玉と5円玉と10円玉と50円玉と100円玉のみを使って両替する仕方の数は、 100*n - 100*n = 100*(n - n) 円を1円玉と5円玉と10円玉と50円玉のみを使って両替する仕方の数に等しい。 以上より、100*n 円を1円玉と5円玉と10円玉と50円玉と100円玉のみを使って両替する仕方の数を知るには、 100*整数 円を1円玉と5円玉と10円玉と50円玉のみを使って両替する仕方の数が分かればよい。
978 名前:132人目の素数さん [2017/11/14(火) 21:47:21.88 ID:5GwueLvD.net] [D] 100*m 円を1円玉と5円玉と10円玉と50円玉のみを使って両替する仕方の数を 50円玉を使用する枚数によって場合分けする: (Case 0) 0 枚の50円玉を使う場合に 100*m 円を1円玉と5円玉と10円玉と50円玉のみを使って両替する仕方の数は、 100*m - 50*0 = 50*(2*m - 0) 円を1円玉と5円玉と10円玉のみを使って両替する仕方の数に等しい。 (Case 1) 1 枚の50円玉を使う場合に 100*m 円を1円玉と5円玉と10円玉と50円玉のみを使って両替する仕方の数は、 100*m - 50*1 = 50*(2*m - 1) 円を1円玉と5円玉と10円玉のみを使って両替する仕方の数に等しい。 … (Case i) i 枚の50円玉を使う場合に 100*m 円を1円玉と5円玉と10円玉と50円玉のみを使って両替する仕方の数は、 100*m - 50*i = 50*(2*m - i) 円を1円玉と5円玉と10円玉のみを使って両替する仕方の数に等しい。 … (Case 2*m) 2*m 枚の50円玉を使う場合に 100*m 円を1円玉と5円玉と10円玉と50円玉のみを使って両替する仕方の数は、 100*m - 50*2*m = 50*(2*m - 2*m) 円を1円玉と5円玉と10円玉のみを使って両替する仕方の数に等しい。 以上より、100*m 円を1円玉と5円玉と10円玉と50円玉のみを使って両替する仕方の数を知るには、 50*整数 円を1円玉と5円玉と10円玉のみを使って両替する仕方の数が分かればよい。
979 名前:132人目の素数さん [2017/11/14(火) 21:47:46.34 ID:5GwueLvD.net] [C] 50*l 円を1円玉と5円玉と10円玉のみを使って両替する仕方の数を 10円玉を使用する枚数によって場合分けする: (Case 0) 0 枚の10円玉を使う場合に 50*l 円を1円玉と5円玉と10円玉のみを使って両替する仕方の数は、 50*l - 10*0 = 10*(5*l - 0) 円を1円玉と5円玉のみを使って両替する仕方の数に等しい。 (Case 1) 1 枚の10円玉を使う場合に 50*l 円を1円玉と5円玉と10円玉のみを使って両替する仕方の数は、 50*l - 10*1 = 10*(5*l - 1) 円を1円玉と5円玉のみを使って両替する仕方の数に等しい。 … (Case i) i 枚の10円玉を使う場合に 50*l 円を1円玉と5円玉と10円玉のみを使って両替する仕方の数は、 50*l - 10*i = 10*(5*l - i) 円を1円玉と5円玉のみを使って両替する仕方の数に等しい。 … (Case 5*l) 5*l 枚の10円玉を使う場合に 50*l 円を1円玉と5円玉と10円玉のみを使って両替する仕方の数は、 50*l - 10*5*l = 10*(5*l - 5*l) 円を1円玉と5円玉のみを使って両替する仕方の数に等しい。 以上より、50*l 円を1円玉と5円玉と10円玉のみを使って両替する仕方の数を知るには、 10*整数 円を1円玉と5円玉のみを使って両替する仕方の数が分かればよい。
980 名前:132人目の素数さん [2017/11/14(火) 21:48:11.66 ID:5GwueLvD.net] [B] 10*k 円を1円玉と5円玉のみを使って両替する仕方の数を 5円玉を使用する枚数によって場合分けする: (Case 0) 0 枚の5円玉を使う場合に 10*k 円を1円玉と5円玉のみを使って両替する仕方の数は、 10*k - 5*0 = 5*(2*k - 0) 円を1円玉のみを使って両替する仕方の数に等しい。 (Case 1) 1 枚の5円玉を使う場合に 10*k 円を1円玉と5円玉のみを使って両替する仕方の数は、 10*k - 5*1 = 5*(2*k - 1) 円を1円玉のみを使って両替する仕方の数に等しい。 … (Case i) i 枚の5円玉を使う場合に 10*k 円を1円玉と5円玉のみを使って両替する仕方の数は、 10*k - 5*i = 5*(2*k - i) 円を1円玉のみを使って両替する仕方の数に等しい。 … (Case 2*k) 2*k 枚の5円玉を使う場合に 10*k 円を1円玉と5円玉のみを使って両替する仕方の数は、 10*k - 5*2*k = 5*(2*k - 2*k) 円を1円玉のみを使って両替する仕方の数に等しい。 以上より、10*k 円を1円玉と5円玉のみを使って両替する仕方の数を知るには、 5*整数 円を1円玉のみを使って両替する仕方の数が分かればよい。
981 名前:132人目の素数さん [2017/11/14(火) 21:48:29.53 ID:5GwueLvD.net] [A] 5*j 円を1円玉のみを使って両替する仕方の数は明らかに1通りである。
982 名前:132人目の素数さん [2017/11/14(火) 21:48:50.75 ID:5GwueLvD.net] [B] 10*k 円を1円玉と5円玉のみを使って両替する仕方の数を 5円玉を使用する枚数によって場合分けする: (Case 0) 0 枚の5円玉を使う場合に 10*k 円を1円玉と5円玉のみを使って両替する仕方の数は、 10*k - 5*0 = 5*(2*k - 0) 円を1円玉のみを使って両替する仕方の数に等しい。 [A]よりそれは 1 通りである。 (Case 1) 1 枚の5円玉を使う場合に 10*k 円を1円玉と5円玉のみを使って両替する仕方の数は、 10*k - 5*1 = 5*(2*k - 1) 円を1円玉のみを使って両替する仕方の数に等しい。 [A]よりそれは 1 通りである。 … (Case i) i 枚の5円玉を使う場合に 10*k 円を1円玉と5円玉のみを使って両替する仕方の数は、 10*k - 5*i = 5*(2*k - i) 円を1円玉のみを使って両替する仕方の数に等しい。 [A]よりそれは 1 通りである。 … (Case 2*k) 2*k 枚の5円玉を使う場合に 10*k 円を1円玉と5円玉のみを使って両替する仕方の数は、 10*k - 5*2*k = 5*(2*k - 2*k) 円を1円玉のみを使って両替する仕方の数に等しい。 [A]よりそれは 1 通りである。 以上より 10*k 円を1円玉と5円玉のみを使って両替する仕方の数は Σ 1 from i = 0 to i = 2*k = 2*k + 1 である。
983 名前:132人目の素数さん [2017/11/14(火) 21:49:18.13 ID:5GwueLvD.net] [C] 50*l 円を1円玉と5円玉と10円玉のみを使って両替する仕方の数を 10円玉を使用する枚数によって場合分けする: (Case 0) 0 枚の10円玉を使う場合に 50*l 円を1円玉と5円玉と10円玉のみを使って両替する仕方の数は、 50*l - 10*0 = 10*(5*l - 0) 円を1円玉と5円玉のみを使って両替する仕方の数に等しい。 [B]よりそれは、2*(5*l - 0) + 1 通りである。 (Case 1) 1 枚の10円玉を使う場合に 50*l 円を1円玉と5円玉と10円玉のみを使って両替する仕方の数は、 50*l - 10*1 = 10*(5*l - 1) 円を1円玉と5円玉のみを使って両替する仕方の数に等しい。 [B]よりそれは、2*(5*l - 1) + 1 通りである。 … (Case i) i 枚の10円玉を使う場合に 50*l 円を1円玉と5円玉と10円玉のみを使って両替する仕方の数は、 50*l - 10*i = 10*(5*l - i) 円を1円玉と5円玉のみを使って両替する仕方の数に等しい。 [B]よりそれは、2*(5*l - i) + 1 通りである。 … (Case
984 名前:5*l) 5*l 枚の10円玉を使う場合に 50*l 円を1円玉と5円玉と10円玉のみを使って両替する仕方の数は、 50*l - 10*5*l = 10*(5*l - 5*l) 円を1円玉と5円玉のみを使って両替する仕方の数に等しい。 [B]よりそれは、2*(5*l - 5*l) + 1 通りである。 以上より 50*l 円を1円玉と5円玉と10円玉のみを使って両替する仕方の数は Σ 2*(5*l - i) + 1 from i = 0 to i = 5*l = Σ 2*i + 1 from i = 0 to i = 5*l = 25*l^2 + 10*l + 1 である。 [] [ここ壊れてます]
985 名前:132人目の素数さん [2017/11/14(火) 21:49:38.86 ID:5GwueLvD.net] [D] 100*m 円を1円玉と5円玉と10円玉と50円玉のみを使って両替する仕方の数を 50円玉を使用する枚数によって場合分けする: (Case 0) 0 枚の50円玉を使う場合に 100*m 円を1円玉と5円玉と10円玉と50円玉のみを使って両替する仕方の数は、 100*m - 50*0 = 50*(2*m - 0) 円を1円玉と5円玉と10円玉のみを使って両替する仕方の数に等しい。 [C]よりそれは、 25*(2*m - 0)^2 + 10*(2*m - 0) + 1 通りである。 (Case 1) 1 枚の50円玉を使う場合に 100*m 円を1円玉と5円玉と10円玉と50円玉のみを使って両替する仕方の数は、 100*m - 50*1 = 50*(2*m - 1) 円を1円玉と5円玉と10円玉のみを使って両替する仕方の数に等しい。 [C]よりそれは、 25*(2*m - 1)^2 + 10*(2*m - 1) + 1 通りである。 … (Case i) i 枚の50円玉を使う場合に 100*m 円を1円玉と5円玉と10円玉と50円玉のみを使って両替する仕方の数は、 100*m - 50*i = 50*(2*m - i) 円を1円玉と5円玉と10円玉のみを使って両替する仕方の数に等しい。 [C]よりそれは、 25*(2*m - i)^2 + 10*(2*m - i) + 1 通りである。 … (Case 2*m) 2*m 枚の50円玉を使う場合に 100*m 円を1円玉と5円玉と10円玉と50円玉のみを使って両替する仕方の数は、 100*m - 50*2*m = 50*(2*m - 2*m) 円を1円玉と5円玉と10円玉のみを使って両替する仕方の数に等しい。 [C]よりそれは、 25*(2*m - 2*m)^2 + 10*(2*m - 2*m) + 1 通りである。 以上より 100*m 円を1円玉と5円玉と10円玉と50円玉のみを使って両替する仕方の数は Σ 25*(2*m - i)^2 + 10*(2*m - i) + 1 from i = 0 to i = 2*m = Σ 25*i^2 + 10*i + 1 from i = 0 to i = 2*m = (200/3)*m^3 + 70*m^2 + (61/3)*m + 1 である。
986 名前:132人目の素数さん [2017/11/14(火) 21:51:26.20 ID:e6LNzgOj.net] [E] 100*n 円を1円玉と5円玉と10円玉と50円玉と100円玉のみを使って両替する仕方の数を 100円玉を使用する枚数によって場合分けする: (Case 0) 0 枚の100円玉を使う場合に 100*n 円を1円玉と5円玉と10円玉と50円玉と100円玉のみを使って両替する仕方の数は、 100*n - 100*0 = 100*(n - 0) 円を1円玉と5円玉と10円玉と50円玉のみを使って両替する仕方の数に等しい。 [D]よりそれは、 (200/3)*(n - 0)^3 + 70*(n - 0)^2 + (61/3)*(n - 0) + 1 通りである。 (Case 1) 1 枚の100円玉を使う場合に 100*n 円を1円玉と5円玉と10円玉と50円玉と100円玉のみを使って両替する仕方の数は、 100*n - 100*1 = 100*(n - 1) 円を1円玉と5円玉と10円玉と50円玉のみを使って両替する仕方の数に等しい。 [D]よりそれは、 (200/3)*(n - 1)^3 + 70*(n - 1)^2 + (61/3)*(n - 1) + 1 通りである。 … (Case i) i 枚の100円玉を使う場合に 100*n 円を1円玉と5円玉と10円玉と50円玉と100円玉のみを使って両替する仕方の数は、 100*n - 100*i = 100*(n - i) 円を1円玉と5円玉と10円玉と50円玉のみを使って両替する仕方の数に等しい。 [D]よりそれは、 (200/3)*(n - i)^3 + 70*(n - i)^2 + (61/3)*(n - i) + 1 通りである。 … (Case n) n 枚の100円玉を使う場合に 100*n 円を1円玉と5円玉と10円玉と50円玉と100円玉のみを使って両替する仕方の数は、 100*n - 100*n = 100*(n - n) 円を1円玉と5円玉と10円玉と50円玉のみを使って両替する仕方の数に等しい。 [D]よりそれは、 (200/3)*(n - n)^3 + 70*(n - n)^2 + (61/3)*(n - n) + 1 通りである。 以上より 100*n 円を1円玉と5円玉と10円玉と50円玉と100円玉のみを使って両替する仕方の数は Σ (200/3)*(n - i)^3 + 70*(n - i)^2 + (61/3)*(n - i) + 1 from i = 0 to i = n = Σ (200/3)*i^3 + 70*i^2 + (61/3)*i + 1 from i = 0 to i = n = (1/6)*(n + 1)*(100*n^3 + 240*n^2 + 131*n + 6) である。
987 名前:132人目の素数さん mailto:sage [2017/11/14(火) 21:53:06.17 ID:kvt/80v9.net] 誰もそんな長文読みませんよw?
988 名前:132人目の素数さん [2017/11/14(火) 21:59:11.43 ID:0okTRPqG.net] 自殺をするか東大を目指すか迷う。
989 名前:132人目の素数さん mailto:sage [2017/11/14(火) 22:05:47.09 ID:vUTrW3BJ.net] ヒマラヤさんって今何歳なんですか?
990 名前:132人目の素数さん mailto:sage [2017/11/14(火) 22:11:18.48 ID:0VJm9RHG.net] >>955 詳細なレスありがとうございます。 一般化するとなるとものすごい量になりますね。 本当に参考になりました。ありがとうございます。
991 名前:132人目の素数さん mailto:sage [2017/11/14(火) 22:14:56.92 ID:vUTrW3BJ.net] 質問者にすら読んでもらえてないんですね(笑)
992 名前:132人目の素数さん mailto:sage [2017/11/15(水) 00:04:07.96 ID:JnSkgPie.net] 分からない問題はここに書いてね437 https://rio2016.5ch.net/test/read.cgi/math/1510671832/
993 名前:132人目の素数さん [2017/11/15(水) 00:09:02.98 ID:Sb2FJtYo.net] >>965 白石容疑者にお願いしてください
994 名前:¥ mailto:sage [2017/11/15(水) 00:49:52.83 ID:WZuPK5Ir.net] ¥
995 名前:¥ mailto:sage [2017/11/15(水) 00:50:12.04 ID:WZuPK5Ir.net] ¥
996 名前:¥ mailto:sage [2017/11/15(水) 00:50:30.86 ID:WZuPK5Ir.net] ¥
997 名前:¥ mailto:sage [2017/11/15(水) 00:50:54.60 ID:WZuPK5Ir.net] ¥
998 名前:¥ mailto:sage [2017/11/15(水) 00:51:14.74 ID:WZuPK5Ir.net] ¥
999 名前:¥ mailto:sage [2017/11/15(水) 00:51:32.91 ID:WZuPK5Ir.net] ¥
1000 名前:¥ mailto:sage [2017/11/15(水) 00:51:53.25 ID:WZuPK5Ir.net] ¥
1001 名前:¥ mailto:sage [2017/11/15(水) 00:52:13.01 ID:WZuPK5Ir.net] ¥
1002 名前:¥ mailto:sage [2017/11/15(水) 00:52:34.89 ID:WZuPK5Ir.net] ¥
1003 名前:¥ mailto:sage [2017/11/15(水) 00:52:55.32 ID:WZuPK5Ir.net] ¥
1004 名前:132人目の素数さん [2017/11/15(水) 03:52:44.48 ID:Z7b8EnyO.net] やっぱり自殺してえ。
1005 名前:132人目の素数さん mailto:sage [2017/11/15(水) 04:54:27.25 ID:+vKpWV6S.net] >>981 誰も止めないから死ね せめて死ぬ前に大量殺人して日本史に名を残せよw
1006 名前:967 mailto:sage [2017/11/15(水) 05:05:42.62 ID:Rs4MO3Z3.net] 質問ばかりで申し訳ないのですが砂田版の答えを一般の場合に拡張するとどうなりますか?
1007 名前:132人目の素数さん mailto:sage [2017/11/15(水) 08:43:06.36 ID:KRJCfOte.net] >>983 ある無矛盾な公理系τの任意のモデルに対してある論理式φが常に真となるならば、τからφがLKにおいて証明可能となることを示せ、という問題がわかりません
1008 名前:132人目の素数さん mailto:sage [2017/11/15(水) 09:38:46.81 ID:LbNkD/hZ.net] >>966 信州在住の40代のニートの少林寺拳法一級のおっさん
1009 名前:¥ mailto:sage [2017/11/15(水) 10:52:36.06 ID:WZuPK5Ir.net] ¥
1010 名前:¥ mailto:sage [2017/11/15(水) 10:52:51.27 ID:WZuPK5Ir.net] ¥
1011 名前:¥ mailto:sage [2017/11/15(水) 10:53:08.11 ID:WZuPK5Ir.net] ¥
1012 名前:¥ mailto:sage [2017/11/15(水) 10:53:24.75 ID:WZuPK5Ir.net] ¥
1013 名前:¥ mailto:sage [2017/11/15(水) 10:53:40.82 ID:WZuPK5Ir.net] ¥
1014 名前:¥ mailto:sage [2017/11/15(水) 10:53:57.06 ID:WZuPK5Ir.net] ¥
1015 名前:¥ mailto:sage [2017/11/15(水) 10:54:12.55 ID:WZuPK5Ir.net] ¥
1016 名前:¥ mailto:sage [2017/11/15(水) 10:54:29.61 ID:WZuPK5Ir.net] ¥
1017 名前:¥ mailto:sage [2017/11/15(水) 10:54:46.44 ID:WZuPK5Ir.net] ¥
1018 名前:¥ mailto:sage [2017/11/15(水) 10:55:02.78 ID:WZuPK5Ir.net] ¥
1019 名前:132人目の素数さん mailto:sage [2017/11/15(水) 11:47:18.45 ID:bRQyF7b9.net] >>315 >>609 >>707 >>725 >>797 >>889 >>984 〔ゲーデルの完全性定理〕(1929) 数セミ増刊「数学100の定理」日本評論社 p.210-212 (1983)
1020 名前:132人目の素数さん mailto:sage [2017/11/15(水) 13:17:51.27 ID:8vSvkmEq.net] 読めるわけねーな
1021 名前:132人目の素数さん mailto:sage [2017/11/15(水) 13:43:02.56 ID:bRQyF7b9.net] 図書館 池
1022 名前:¥ mailto:sage [2017/11/15(水) 14:50:30.37 ID:WZuPK5Ir.net] 数学徒は馬鹿板を『しない』生活を送るべき。 ¥
1023 名前:円 mailto:sage [2017/11/15(水) 14:51:04.01 ID:WZuPK5Ir.net] 円
1024 名前:1001 [Over 1000 Thread.net] このスレッドは1000を超えました。 新しいスレッドを立ててください。 life time: 13日 16時間 26分 2秒
1025 名前:過去ログ ★ [[過去ログ]] ■ このスレッドは過去ログ倉庫に格納されています